Dahiliye Tus Deneme Sınavı Soruları [2021 ed.]

TUSDATA 1-2-3-4-5-6-7 TUSEM 1-2-3-4-5 TUSTİME 85-86-87-88 TUSWORLD 61-62-63-64-65 2021 TUS DENEME SINAVLARI DAHİLİYE SO

2,838 200 27MB

Turkish Pages 261 Year 2021

Report DMCA / Copyright

DOWNLOAD FILE

Polecaj historie

Dahiliye Tus Deneme Sınavı Soruları [2021 ed.]

Table of contents :
dahiliye.pdf......Page 46
Tusdata 1.Deneme dahiliye 121-162.pdf......Page 2
Tusdata 2.klinik.dahiliye......Page 8
Tusdata 3.Deneme.klinik.dahiliye......Page 14
Tusdata 4.Deneme klinik.dahiliye......Page 20
Tusdata 5.Deneme - Klinik.dahiliye......Page 26
Tusdata 6.Deneme klinik.dahiliye......Page 33
Tusdata 7.Deneme klinik.dahiliye......Page 40
Deneme-1 açıklamalar.dahiliye.pdf......Page 47
Deneme-2 açıklamalar klinik.dahiliye......Page 63
Deneme-3 açıklamalar klinik.dahiliye......Page 82
Deneme-4 açıklamalar klinik.dahiliye......Page 107
Deneme-5 açıklamalar klinik.dahiliye......Page 126
Tustime 85.pdf dahiliye.pdf......Page 148
Tustime 86.pdf dahiliye......Page 161
Tustime 87.pdf dahiliye......Page 173
Tustime 88.pdf dahiliye......Page 185
61. DENEME AÇIKLAMALI.pdf dahiliye.pdf......Page 201
62. DENEME AÇIKLAMALI klinik.pdf dahiliye......Page 215
63. DENEME AÇIKLAMALI klinik.pdf dahiliye......Page 228
64. DENEME AÇIKLAMALI klinik.pdf dahiliye......Page 239
65. DENEME AÇIKLAMALI klinik.pdf dahiliye......Page 252

Citation preview

İÇİNDEKİLER NİSAN 2021 TUSDATA / 1-2-3-4-5-6-7 TUSEM / 1-2-3-4-5 TUSTIME / 85-86-87-88 TUSWORLD / 61-62-63-64-65

1758

2021 DÖNEMİ 1. DENEME SINAVI KLİNİK BİLİMLER TESTİ SORULARI Bu metinde sırasıyla Dahili Bilimler, Pediatri, Cerrahi Bilimler, Kadın Doğum soruları bulunmaktadır.

124.

121.Kırk yaşında erkek hasta halsizlik ve çabuk yorulma şikâyetleri nedeniyle başvuruyor. Laboratuvar testlerinde hemoglobin 6.7 gr/dL, lökosit 5.400/mm3, trombosit 250.000/mm3 ve MCV 73 fL olarak bulunuyor. Serum demiri yüksek, demir bağlama kapasitesi düşük, ferritin düzeyi İse yüksek saptanıyor. Hastanın öyküsünden yoğun alkol kullanımı olduğu ve tüberküloz profllaksl amacıyla Izonlazid kullandığı öğreniliyor.

I. Lenfadenomegali II. Testis tutulumu III. Kloroma IV. Auer rod pozitifliği Akut lösemi ön tanısıyla tetkik edilen bir hastada yukarıdakilerden hangisi ya da hangileri daha çok akut lenfoblastik lösemi lehinedir?

Bu hastanın anemisine yönelik kesin tanı testi aşağıdakilerden hangisidir?

A) I ve II C) II ve IV

B) I, II ve III D) I, II, III ve IV E) III ve IV

A) Kemik iliği İncelemesi B) Sitogenetik analiz C) Hemoglobin elektroforezl D) Akım sitometri E) Folat düzeyi

125.Yetmiş iki yaşında kadın hasta nefes darlığı, yüzünde şişme, morarma ve öksürük şikâyeti İle acil servise başvuruyor. Hastanın yapılan fizik muayenesinde boyun venöz dolgunluk ve yüzünde ödem saptanıyor. Çekilen akciğer graflslnde üst medlastende genişleme olduğu görülen hastaya toraks bilgisayarlı tomografi İsteniyor. Bilgisayarlı tomografisinde vena kava superlora bası yapan konglomere lenfadenopatller İzleniyor.

122.EIII sekiz yaşında erkek hasta kulak çınlaması, baş dönmesi ve özellikle sıcak su İle duş sonrası olan kaşıntı şikâyetleri ile başvuruyor. Fizik muayenesinde pletorlkyüz görünümü saptanıyor ve dalak 5 cm palpabl tespit ediliyor. Laboratuvar testlerinde hemoglobin 19 gr/dL, lökosit 13.000/mm3 ve trombosit 460.000/mm3 bulunuyor. Bu hastada olası tanıya yönelik değerlendirmede aşağıdakilerden hangisi majör tanı kriterlerinden biridir?

Bu hastada tanı için bundan sonraki aşamada en uvaun yöntem aşağıdakilerden hangisidir?

A) Eritrosit sedlmentasyon hızı düşüklüğü B) Hiperselüler kemik İliği C) Splenomegali D) Düşük erltropoletln düzeyi E) Normal parslyel oksijen basıncı

A) Pozitron emisyon tomografisi İle görüntüleme B) Mediastendeki lenfadenopatilerden İnce İğne aspirasyonu C) Toraks manyetik rezonans görüntüleme D) Bronkoskopik İnceleme E) Mamografi

123.İmmünglobulin A eksikliği olduğu bilinen bir hastaya eritrosit transfüzyonu planlandığında aşağıdakilerden hangisi mutlaka uygulanmalıdır?

126.Altmış yaşındaki sağlıklı kadında kanser taraması için aşağıdaki testlerden hangisini istemeye gerek yoktur?

A) Tam kan verilmesi düşünülmelidir. B) Yıkanmış eritrosit süspansiyonu kullanılmalıdır. C) Banka kanı değil taze kan verilmelidir. D) Lökosit filtreleri kullanılmalıdır. E) Işınlanmış eritrosit süspansiyonu kullanılmalıdır.

A) Mammografi B) Akciğer grafisl C) Dışkıda gizil kan D) Pap smear E) Cilt muayenesi

17

131.Yetmiş iki yaşında erkek hasta eforla gelen nefes darlığı şikâyetiyle başvuruyor. Hastanın daha önceden bilinen kalp yetmezliği olduğu öğreniliyor. Çekilen elektrokardiyografisinde ritmin sinüs olduğu, ileti kusurunun olmadığı ve kalp hızı 95/dakika saptanıyor. Ekokardiyografisinde sol kalpte dilatasyon ve ejeksiyon fraksiyonu %25 saptanıyor. Kalp yetmezliği tanısı ile hastaya perindopril, karvedilol ve spironolakton başlanıyor, ancak takiplerinde klinik şikâyetlerinin devam ettiği görülüyor.

127.Beş yıldırtip 2 diyabeti olan, 2 yıl önce miyokart infarktüsü geçiren ve son 1 yıldır hipertansiyonu olan ve düzenli antihipertansif ilaç kullanmayan 65 yaşında erkek hasta şiddetli baş ağrısı nedeniyle başvuruyor. Fizik muayenede tansiyon arteriyel 190/120 mmHg ve bilateral 2 pozitif pretibial ödem saptanıyor. Yapılan tetkiklerinde üre 20 mg/dL, kreatinin 1,2 mg/dL, glukoz 160 mg/dL, albümin 2.8 gr/dL saptanıyor. Diğer biyokimyasal tetkikleri normal bulunuyor. Antihipertansif tedavi başlanan hastanın kan basıncı düşmesine rağmen, 3-4 gün içinde nefes darlığı ve idrar miktarında azalma saptanıyor. Bu sırada yapılan tetkiklerinde üre 150 mg/dL, kreatinin 8 mg/dL ve potasyum düzeyi 7.2 mEq/L saptanıyor ve hasta diyalize alınıyor.

Bu hastada aşağıdaki tedavi ajanlarından hangisi eklendiğinde hastanın kalp yetmezliğinde semptomatik iyileşme sağlanır?

Bu hastada kullanılan en olası antihipertansif ilaç aşağıdakilerden hangisidir?

A) Trimetazidin C) Valsartan

A) Diüretik B) Beta blokör C) ACE inhibitörü D) Kalsiyum kanal blokörü E) Alfa blokör

128. Membranöz nefropati ile ilgili olarak aşağıdakilerden hangisi yanlıştır?

B) Verapamil D) Ivabradin E) Ranolozin

132. I. Sekonder hipertansiyon yapan nedenlerden biridir. II. Gençlerde en sık nedeni aterosklerozdur. III. Periumblikal oskültasyonda sistolik üfürüm duyulur. IV. Kesin tanısı renal anjiyografi ile konur.

A) Sistemik bir hastalık sonucu ya da primer olarak açığa çıkabilir. B) Yaşlı birinde tespit edilir ise malignité yönünden araştırılmalıdır. C) Tromboz ilişkisi en fazla olan glomerülonefrittir. D) Serum kompleman düzeylerinde azalma beklenir. E) Daha çok nefrotik sendrom kliniğine neden olur.

Yukarıdakilerden hangileri renovasküler hipertansiyon için doğrudur? A) I ve II C) III ve IV

B) I, II ve IV D) I, III ve IV E) I,II,III ve IV

129. Yetmiş beş yaşında erkek hasta acil servise bilinç bulanıklığı ve genel durum bozukluğu şikâyetiyle getiriliyor. Bakılan laboratuvar değerlerinde sodyum 131 mEq/L, klor 92 mEq/ L, potasyum 2,5 mEq/L, arteriyel kan gazı örneğinde pH 7.53, bikarbonat 35 mEq/L, parsiyel karbondioksit basıncı 48 mm/Hg saptanıyor.

133.Elli yedi yaşında diyabet, hiperlipidemi ve hipertansiyon tanıları olan erkek hasta acil servise son beş saattir eforla ilişkisiz sıkıştırıcı tarzda göğüs ağrısı şikâyetiyle başvuruyor. Hastanın tansiyonu 150/95 mmHg, nabız 90/dakika saptanıyor. Fizik muaynesinde kardiyak açıdan bir özellik saptanmıyor. Elektrokardiyografisinde, sinüs ritminde, ileti kusuru yok, V4 - V5 ve V6’da ST depresyonu saptanıyor. Bakılan CK-MB ve troponin değerlerinin yüksek olduğu saptanıyor.

Bu hastada mevcut en olası asit-baz bozukluğu aşağıdakilerden hangisidir? A) Dekompanze metabolik alkaloz B) Akut respiratuvar alkaloz C) Kompanze metabolik asidoz D) Kompanze metabolik alkaloz E) Kronik respiratuvar alkaloz

Bu hastada en olası tanı aşağıdakilerden hangisidir? A) Kararlı anjina pektoris B) Kararsız anjina pektoris C) ST elevasyonsuz miyokart infaktüsü D) Akut perikardit E) ST elevasyonlu miyokart infarktüsü

130. I. Ölümün en önemli sebebi enfeksiyonlardır. II. Siklosporin böbrekte glomerülonefrit yapar. III. Akut tübüler nekroz iskemik ya da toksik nedenlerle gelişebilir. IV. Prerenal böbrek yetmezliğinde fraksiyone sodyum atılımı (FENa) >%1 beklenir. V. Prerenal böbrek yetmezliğinde idrarda sodyum atılımı azalmıştır. Akut böbrek yetmezliği ile ilgili olarak yukarıdaki ifadelerden hangileri doğrudur? A) I, II, ve III C) I, II, III ve V

B) III, IV, ve V D) I, III ve V E) I, III ve IV

18

139.

134.Sarkoidoz ve pulmoner hipertansiyon tanıları ile takip edilen 62 yaşında erkek hasta nefes darlığı ve bacaklarda ödem şikâyetleri ile başvuruyor. Fizik muayenesinde sol alt sternal kenarda 3/6 şiddetinde holosistolik üfürüm duyuluyor. Üfürümün nefes alma ile şiddetlendiği tespit ediliyor. Boyun ven dalgalarından v dalgasının belirginleştiği saptanıyor. Elektrokardiyografisi sinüs ritminde ve p pulmonale saptanıyor.

I. Tipik pnömoninin en sık nedeni Streptococcus pneumoniae’dır. II. Atipik pnömonide kuru öksürük ve ronküs vardır. III. Palpasyonda vibrasyon torasik azalmıştır. IV. Perküsyonda timpanik ses alınır. Yukarıdakilerden hangileri pnömoni için doğrudur? A) C)

Bu hasta için en olası tanı aşağıdakilerden hangisidir?

I ve II III ve IV E)

B) I, II ve IV D) I, III ve IV I, II, III ve IV

A) Triküspit yetmezliği B) Aort yetmezliği C) Aort stenozu D) Mitral stenozu E) Pulmoner stenozu

140.

135.Aşağıdaki “ hastalık - nabız tipi” eşleştirmelerden hangisi doğru değildir?

I. Hiponatremi II. Hiperkalemi III. Anemi IV. Guatr

A) Aort stenozu - Pulse defisit B) Sol kalp yetmezliği - Pulsus alternans C) Septik şok - Pulsus filiformis D) Hipertrofik kardiyomiyopati - Pulsus bisferiens E) Aort yetmezliği - Pulsus çeler et altus

136.

Yukarıdakilerden hangisi ya da hangileri hipofizer yetmezlikte görülebilir?

Aşağıdaki “ akciğer hastalıkları ve beklenen fizik muayene bulgusu” eşleştirmelerinden hangisi yanlıştır?

A) I ve III C) I ve IV

B) II ve IV D) III ve IV E) I ve II

A) Bronşiektazi - kaba rai B) Astım - Wheezing C) Pnömoni - İnce rai D) Laringomalazi - Stridor E) Interstisyel akciğer hastalıkları - tuber sufl

141.Kırk beş yaşında kadın hasta ses kısıklığı nedeni ile başvuruyor. Hastanın yapılan boyun ultrasonografisinde 2 cm çapında hipoekoik görünümde nodül saptanıyor. Laboratuvar testlerinde ötiroid olduğu anlaşılan hastaya yapılan tiroid ince iğne aspirasyon biyopsi sonucu folliküler neoplazi olarak rapor ediliyor.

137.Yetmiş bir yaşında erkek hasta nefes darlığı, öksürük ve balgam şikâyetleri ile başvuruyor. Fizik muayenesinde uzamış ekspiryum ve ronküs duyuluyor. Öyküsünde 50 paket/yıl sigara öyküsü olan hastaya amfizem ağırlıklı kronik obstrüktif akciğer hastalığı (KOAH) tanısı konuyor. Bu hastada aşağıdaki solunum fonksiyon testi sonuçlarından hangisinin saptanması en olasıdır?

Bu hasta için bir sonraki aşamada en uvaun yaklaşım aşağıdakilerden hangisidir?

A) Rezidüel volüm azalır. B) Total akciğer kapasitesi azalır. C) FVC (zorlu vital kapasite) artar. D) FEV1/FVC artar. E) Difüzyon kapasitesinde azalma görülür.

138.

A) Biyopsi tekrarı B) Takip C) Tiroid sintigrafisi D) Moleküler test incelemesi E) Tirotropin reseptör antikoru bakılması

Elli sekiz yaşında erkek hasta büyük hücreli akciğer kanseri ve hipertansiyon tanıları ile takip ediliyor. Bu hastada özellikle aşağıdaki paraneoplastik sendromlardan hangisi beklenir? A) Uygunsuz ADH sendromu B) Jinekomasti C) Hipertrofik osteoartropati D) Eaton Lambert sendromu E) Hiperkalsemi

142.Elli beş yaşında erkek hastanın yaklaşık 4 yıl önce tip 2 diabetes mellitus tanısı aldığı öğreniliyor. Kan şekeri regülasyonu için metformin tedavisi aldığı öğrenilen hastanın yapılan kontrollerinde HbA1c düzeyleri %8,5 olarak saptanıyor. Kilo alma endişesi taşıyan bu hastanın tedavisine aşağıdaki ajanlardan hangisinin eklenmesi uygun olur? A) Pioglitazon C) Glimepirid

19

B) Gliklazid D) Ekzenatid E) Bazal insülin

143.

147.Aşağıdakilerden hangisi intrahepatik hipertansiyon sebeplerinden biri değildir?

I. 24 saatlik idrarda vanil mandelik asit II. Fludrokortizon testi III. Tuz infüzyon testi IV. Synacten testi

A) Şistozoma B) Sarkoidoz C) Siroz D) Portal ven trombozu E) Peliozis hepatis

Primer hiperaldosteronizm ön tanısı ile takip edilen bir hastada yukarıdakilerden hangisi/hangileri doğrulama testi olarak kullanılır? A) I ve II C) I ve IV

B) II ve III D) III ve IV E)

I ve III

148. Elli beş yaşında erkek hasta halsizlik, yorgunluk, kilo kaybı, karın ağrısı ve eklemlerde ağrı şikâyeti ile başvuruyor. Özgeçmişinde hipertansiyon ve kronik hepatit B öyküsü olan hastanın yapılan fizik muayenesinde tansiyon arteriyel: 220/120 mmHg olarak saptanıyor. Ayrıca sistemik muayenesinde sağ alt ekstremitede düşük ayak ve duyusal nöropati saptanıyor. Laboratuvar değerlendirmesinde sedimantasyon değeri 80 mm/s, kreatinin: 2.5 mg/dL, ALT: 120 U/L olarak saptanıyor.

144.EIIİ yaşında erkek hasta yemeklerden sonra ağzına acı su gelmesi ve göğüs bölgesinde yanma şikâyeti olması nedeniyle başvuruyor. Özgeçmişinde obezite dışında özellik saptanamayan hastanın bakılan laboratuvar testlerinde hiperglisemi ve hiperlipidemi dışında bir özellik saptanmıyor. Hastaya yapılan üst gastrointestinal sistem endoskopide alt özofajial bölgede mide asit kaçağı gözleniyor. Bu hastanın tedavisine aşağıdaki ajanlardan hangisinin eklenmesi uygundur?

Bu hasta için en olası tanı aşağıdakilerden hangisidir?

A) Alginik asit B) Izosorbid nitrat C) Metilen mavisi D) Diazepam E) Prinzepin

A) VVegener granülomatozu B) Churge strauss sendromu C) Poliarteritis nodoza (PAN) D) Behçet hastalığı E) Ailesel akdeniz ateşi (FMF)

145.

149.Kırk iki yaşında kadın hasta her iki el eklemlerinde özellikle sabahları olan en az 1 saat süren ağrı tutukluk hissi ile başvuruyor. Yaplan fizik muayenesinde her iki el eklemlerinde hassasiyet saptanıyor. Özgeçmişinde özellik saptanmayan bu hasta için bakılan laboratuvar testlerinde normokrom normositer anemi, sedimantasyon ve C-reaktif protein düzeylerinde yükseklik saptanıyor. Hastanın bakılan anti-CCP testi ise yüksek titrede pozitif izleniyor.

I. Periferik artrit II. Pyoderma gangrenosum III. Eritema nodosum IV. Üveit Yukarıdakilerden hangileri ülseratif kolit seyrinde bağırsak aktivitesi ile ilişkili olarak açığa çıkmaktadır?

Bu hastada tüm yaşam boyunca mortalitenin en önemli sebebi aşağıdakilerden hangisidir?

A) I ve III B) II ve IV C) II ve III D) I ve IV E) I ve II

146.

portal

A) Kardiyovasküler tutulum B) Böbrek tutulumu C) Akciğer tutulumu D) Pulmoner arter anevrizması E) Santral sinir sistemi tutulumu

Akut viral hepatit tanısı alan hastada fulminant hepatite gidişatı gösteren en deöerli parametre aşağıdakilerden hangisidir? A) Alanin transaminaz düzeyi B) Aspartat transaminaz düzeyi C) Protrombin zamanı D) Albümin düzeyi E) Gama glutamil transferaz düzeyi

150.Kırk yaşında kadın hasta başının sol yarısında olan genelde 12-24 saat süren ayda birkaç kez olan tekrarlayan tarzda baş ağrısı olduğunu söylüyor. Hastanın ağrısı olduğu zaman işine devam edemediğini özellikle ışıktan çok rahatsız olduğunu söylüyor. Bu hastada profilatiktedavide aşağıdaki yaklaşımlardan hangisi en doğrudur? A) Sumatriptan C) Kodein E)

20

B) Parasetamol D) Propranolol Asprin

151.Yirmi bir yaşında kadın hasta iki gün önce başlayan ve gittikçe artan sol kolunda ve bacağında karıncalanma, uyuşma şikâyeti ile polikliniğe başvuruyor. Hastanın öyküsünden bir hafta önce grip olduğu öğreniliyor. Uyuşma şikâyetinin sol bacağından başladığı ve son iki günde sol kol ve bacağının tamamının uyuştuğunu söylüyor.

154.Yirmi sekiz yaşında kadın hasta sürekli olarak kötü bir şey olacakmış hissi, gergin hissetme, yaygın vücut ağrısı, çarpıntı, çabuk yorulma ve uykuya dalamama yakınmalarıyla başvuruyor. Hasta bir yıldır devam eden bu yakınmalarının sosyal ve mesleki başarısını olumsuz etkilediğini ifade ediyor. Bu hasta için en olası tanı ve uzun süre tek başına kullanılabilecek Mk ilaç seçeneği aşağıdakilerden hangisinde birlikte doğru verilmiştir? A) Panik bozukluğu - Alprazolam B) Panik bozukluğu - Essitalopram C) Yaygın anksiyete bozukluğu - Diazepam D) Yaygın anksiyete bozukluğu - Sitalopram E) Yaygın anksiyete bozukluğu - Pregabalin

155.“Atak hızı” nm aşağıdaki hastalıklardan hangisi için kullanılması en uygundur?

Yukarıda manyetik rezonans görüntüsü olan hastada en olası tanı aşağıdakilerden hangisidir? A) B) C) D) E)

İskemik serebrovasküler hastalık Hemorajik serebrovasküler hastalık Nöromiyelitis optika Multipl skleroz Nörosistiserkozis

A) Gastroenterit B) Farenjit C) Rinit D) Konjunktivit E) Kızamık

152.0n aylık erkek bebek annesi tarafından kollarında ve bacaklarında tekrarlayan kasılıp gevşemeleri olduğu için polikliniğe getiriliyor. Annesi bebeğin kendisiyle göz teması kurmadığını ve desteksiz oturamadığını söylüyor. Hastanın yapılan EEG incelemesinde multipl odak yaygın diken yavaş dalga aktivitesi ve hipsaritmi saptanıyor.

156.Deneysel bir çalışmada, kontrolsüz diyabeti olan 100 hastanın kan şekeri düzeyleri, yeni bir oral anti-diyabetik ajanın uygulanması öncesi ve sonrası yapılan ölçümlerle mg/dL cinsinden değerlendiriliyor.

Bu hastada “ en olası tanı - ilk tedavi yaklaşımı” eşleştirmelerinden hangisi en doârudur?

Bu araştırmanın verilerinin değerlendirilmesinde aşağıdaki istatistiksel testlerden hangisi uygulanmalıdır?

A) Lenox- Gestaut sendromu - Valproat B) Dravet sendromu - Levetirasetam C) Selim süt çocuğu miyoklonisi - Karbamezapin D) West sendromu - ACTH E) Jeneralize tonik klonik nöbet - Valproat

A) B) C) D) E)

153.0tuz yaşında kadın hasta kendisini takip eden ve öldürmek isteyen insanlar olduğundan şüphelenme, evlerinin telsizlerle dinlendiğini, televizyonda kendisiyle ilgili yayınlar yapıldığını söyleme ve kendisini yönlendiren sesler duyma şikâyetleri ile yakınları tarafından psikiyatri polikliniğine getiriliyor. Alınan psikiyatrik öyküsünde hastanın geçmişte iki kez enerjisinin ve hareketliliğinin çok yüksek olduğu, uyku ihtiyacının azaldığı, çok fazla para harcadığı, çok fazla ve hızlı konuştuğu, kendisine ve etrafa zararı dokunacak davranışlarda bulunabildiği dönemler yaşadığı ve bu dönemlerin yaklaşık 3 hafta sürerek kendiliğinden gerilediği belirtiliyor. Bununla birlikte hastanın gerçek dışı alınganlık ve şüphecilik düşünceleri ile ses duyma yakınmalarının enerjisinin normal olduğu dönemlerde de görüldüğü öğreniliyor.

Mann-Whitney U testi İki bağımsız ortalama arasındaki farkın önemlilik testi İki eş arasındaki ortalama farkının önemlilik testi İki bağımsız yüzde arasındaki farkın önemlilik testi Ki-kare uyum testi

157.0tuz iki yaşında erkek hasta son iki yıldır her iki taraf diz ve dirseklerinde ortaya çıkan hafif kaşıntılı gümüş renkli, eritematöz plaklar nedeniyle başvuruyor. Dermatolojik muayenesinde gümüş renkli plaklar kaldırıldığında noktasal kanama odaklarıyla karşılaşıyor. Bu hastada aşağıdaki klinik ve patolojik bulgulardan hangisinin görülmesi beklenmez? A) Akantoz B) Parakeratoz C) Mum lekesi belirtisi D) Koebner fenomeni E) Civatte hücreleri

Güncel ruhsal durum muayenesinde duygudurumunun ötimik olduğu değerlendirilen bu hasta için en olası tanı aşağıdakilerden hangisidir? A) Şizofreni B) Şizoaffektif bozukluk C) Bipolar bozukluk, manik dönem D) Sanrılı bozukluk E) Paranoid kişilik bozukluğu

21

161 .Serebral perfüzyon bozukluğu düşünülen yenidoğanda germinal matriks kanaması ve hipoksik iskemik ensefalopati ekartasyonları için en ivi tetkikler sırasıyla aşağıdakilerden hangileridir?

158.Altmış yaşında kadın hasta son iki aydır giderek artan kaşıntılı büllerle başvuruyor. Fizik muayenesinde ürtikeryal plakların yanı sıra abdomen ve uylukta gergin büller saptanıyor Deri biyopsisinin direkt immünofloresan incelemesinde bazal membrana karşı gelişmiş IgG tipinde antikorlar saptanıyor.

A) Ultrasonografi - Bilgisayarlı tomografi B) Ultrasonografi - Manyetik rezonans görüntüleme C) Bilgisayarlı tomografi - Manyetik rezonans görüntüleme D) Manyetik rezonans görüntüleme - Ultrasonografi E) Bilgisayarlı tomografi - Ultrasonografi

Bu hasta için en olası tanı aşağıdakilerden hangisidir? A) Alerjik kontakt dermatit B) Büllöz pemfigoid C) Dermatitis herpetiformis D) Epidermolizis bülloza E) Pemfigus vulgaris

159.0tuz sekiz yaşında erkek hasta sağ el başparmak hizasında olan aşırı ağrı sebebiyle doktora başvuruyor, öyküsünden tornavidacı olduğu ve bu ağrılarının son dört aydırşiddetlendiği öğreniliyor. Fizik muayenesinde aşağıda görülen testi pozitif saptanan hastanın sedimantasyon ve diğer tüm kan tahlilleri normal olarak saptanıyor.

162.Aşağıdakilerden hangisi eritrosit yaşam süresini değerlendirme için kullanılan nükleer tıp ajanıdır? A) Cr-51 C) 1-123 E) P-32

Bu hastada en cok etkilenmesi beklenen kaslar aşağıdakilerden hangileridir? I. Abduktor pollicis longus II. Ekstensör pollicis longus III. Ekstensör pollicis brevis IV. Ekstensör carpi radialis longus A) I ve II C) II ve IV

B) I ve III D) II ve III E) III ve IV

160.Kırk sekiz yaşında kadın hasta özellikle geceleri artan elindeki uyuşukluk ve ağrı şikâyetiyle doktora başvuruyor. Öyküsünden bilinen romatoid artriti olan hastanın şikâyetinin son bir yıldır giderek şiddetlendiği ve koluna doğru yayıldığı öğreniliyor. Fizik muayenesinde tenar kaslarda atrofi saptanıyor. Bu hastada en olası tanı ile ilgili olarak aşağıda verilen testlerden hangisi veya hangilerinin pozitif olması beklenir? I. Tinel testi II. Phalen testi III. Yergason testi A) Yalmzl C) I ve II

B) 1-131 D) Ga-67

B)Yalnız II D)II ve III E) I, II ve III

22

2021 DÖNEMİ 2. DENEME SINAVI KLİNİK BİLİMLER TESTİ SORULARI t me/tuskitabı Bu metinde sırasıyla Dahili Bilimler, Pediatri, Cerrahi Bilimler, Kadın Doğum soruları bulunmaktadır.

125. Aşağıdaki “paraneoplastlk sendrom - en sık neden olan kanser" eşleştirmelerinden hangisi doğrudur?

121.Aşağıdaki periferik yayma bulgularından hangisinin beta talasemi majör hastasında görülme ihtimali en düşüktür?

A) Tromboemboli - Non-Hodgkin lenfoma B) Hipoglisemi - Glukagonoma C) Ektopik Cushing sendromu - Küçük hücreli akciğer kanseri D) Hipotiroidl - Mol hidatiform E) Uygunsuz ADH sendromu - Büyük hücreli akciğer kanseri

A) Anizositoz B) Şistosit C) Hedef hücre D) Bazofilik noktalanma E) Hipokromi

122.Yirmi altı yaşında erkek hasta halsizlik, yorgunluk ve sol servikal bölgede hızlı büyüyen şişlik şikâyeti ile başvuruyor. Fizik muayenede sol servikal 5x4 cm boyutlarında hassas olmayan lenf bezi palpe ediliyor. Aksilla ve inguinal bölgede de boyutları artmış lenf nodları saptanıyor. Servikal lenf nodundan yapılan biyopsi dlffüz büyük B hücreli lenfoma olarak raporlanıyor.

126.Elli sekiz yaşında kadın hasta sol memede kitle ve sırt ağrısı şikâyetleri İle başvuruyor. Yirmi yıldır ortalama günde 2 paket sigara kullanımı olduğu öğrenilen hastanın yapılan kitle biyopsisi, meme kanseri ile uyumlu bulunuyor. Laboratuvar testlerinde serum kalsiyum düzeyi 14 mg/dL ve diğer parametreler normal bulunuyor.

Bu hastanın prognoz değerlendirilm esinde kullanılabilecek kriterler aşağıdakilerden hangisinde doğru olarak verilmiştir?

Bu hastada hiperkalsemiye yönelik RANKL (receptor activator of nuclear factor-kB ligand) etkinliğini nötralize edici etkisi nedeniyle verilebilecek en uvaun ilaç aşağıdakilerden hangisidir?

A) Bcl-2, Bcl-6 ve c-MYC pozitifliği B) t(8; 14) pozitifliği C) SOX11 pozitifliği D) Anaplastlk lenfoma klnaz (ALK) pozitifliği E) t(14;18) pozitifliği

A) Mitramisin B) Zolendronik asit C) Kalsitonin D) Denosumab E) Ofatumumab

123.Aşağıdaki seçeneklerden hangisinde aktive parsiyel tromboplastin zamanı (aPTT) uzaması ile birlikte tromboz riskinde de artış beklenir?

127.Altmış beş yaşında erkek hasta tip 2 diabetes mellitusa bağlı kronik böbrek hastalığı nedeni ile takip ediliyor. İdrar miktarında azalma ve ciddi halsizlik nedeniyle başvuran hastanın bilinci açık, oryante ve koopere bulunuyor. Laboratuvar tetkiklerinde; serum kreatinln düzeyi 6 mg/ dL, kan üre azotu (BUN) 70 mg/dL, potasyum 6.1 mEq/L, hemoglobin 9 g/dL, glomerüler filtrasyon değeri 9 mL/dk olarak saptanıyor.

A) Immün trombositopenik purpura (ITP) B) Antifosfollpid antikor sendromu C) Faktör VIII eksikliği D) von Willebrand hastalığı E) Faktör XIII eksikliği

Bu hastada aşağıdakilerden endikasyonlarından birisidir?

124.Otuz üç yaşında kadın hasta halsizlik ve adet kanamalarının uzun sürmesi şikâyetleri İle başvuruyor. Fizik muayenede ateş 38°C, konjunktivalarında solukluk ve ekstremltelerlnde peteşiler saptanıyor. Organomegall tespit edilmiyor. Laboratuvar incelemelerinde hemoglobin 7 g/dL, ortalama eritrosit hacmi (MCV) 106 fL, lökoslt 1.100/ mm3, trombosit 16.000/mm3, düzeltilmiş retikülosit %0,2 total bilirubin 0,8 mg/dL, indirekt bilirubin 0,3 mg/dL ve laktat dehidrogenaz 110 U/L olarak saptanıyor.

A) B) C) D) E)

hangisi diyaliz

Kan üre azotu (BUN) 70 mg/dL olması Kreatininin 6 mg/dL olması Potasyum 6.1 mEq/L olması Hemoglobin 9 g/dL olması Glomerüler filtrasyon düzeyi 9 mL/daklka olması

128.Serum potasyum düzeyi 2.8 mEq/L olan bir hastada aşağıdaki bulgulardan hangisinin görülme olasılığı en düşüktür?

Bu hastada en olası tanı aşağıdakilerden hangisidir? A) Saf erltrold aplazi B) Paroksismal noktürnal hemoglobinüri C) Vitamin B12 eksikliği D) Aplastik anemi E) Kronik mlyelold lösemi

A) Kısa PR mesafesi B) Poliüri C) T dalgasının düzleşmesi D) Kas güçsüzlüğü E) QT uzaması

17

133.Yetmlş iki yaşında prostat kanseri tan ısı ile takip edilen erkek hasta nefes darlığı şikâyeti İle acil servise başvuruyor. Fizik muayenesinde boyun venöz dolgunluğu ve hafif prétibial ödem saptanıyor. Kardlyak oskültasyonda kalp sesleri derinden geliyor. Ölçülen kan basıncı 80/60 mmHg, nabız hızı 149/dakika saptanıyor. Elektrokardiyografisinde ritmin sinüs ritmi olduğu görülüyor ve voltaj kaybı saptanıyor. Telekardlyografide kalp gölgesinin büyüdüğü İzleniyor.

129.Yirmi beş yaşında erkek hasta ateş bulantı, kusma ve bacaklarında döküntü olması nedeniyle başvuruyor. Özgeçmişinde daha öncesinde bilinen bir hastalığı olmadığı öğrenilen hastanın 10 gün önce akut sinüzit nedeniyle antibiyotik kullandığı öğreniliyor. Laboratuvar değerlerinde kreatlnln 2.5 mg/dL, üre 130 mg/dL olarak saptanıyor, hemogramında lökositoz ve eozlnoflli tespit ediliyor.

Bu hastada en olası tanı aşağıdakilerden hangisidir? Bu hastanın idrar analizinde özellikle aşağıdaki bulgulardan hangisinin görülmesi beklenir? A) B) C) D) E)

A) Atriyal miksoma B) Akut mlyokardlt C) Akut perikardit D) Varyant anjlna E) Kardiyak tamponad

Eritrosit sllendlrl Lökosit sllendlrl Proteinüri Hiyalen silendir Kahverengi granüler silendir

130.Aşağıdakilerde hangisi gebelikte ilk trimesterde üriner sistemde görülebilen fizyolojik değişikliklerden birisidir?

134.Resimdeki elektrokardiyografiye göre en olası tanı aşağıdakilerden hangisidir?

A) İdrarla bikarbonat atılımında azalma B) Kan basıncı ve periferik damar direncinde artış C) Serum kreatlnln, üre ve ürik asit değerlerinde hafif artış D) Hafif solunumsal asidoz E) Glukozüri

| ffa j i l J

l -AİZi

=P _

V

lİ Ö fi 1- A

“ —— !

A) Atriyal fibrilasyon (AF) B) Wolf - Parkinson White sendromu (WPW) C) Atriyal flutter D) Torsades de pointes E) Ventriküler fibrilasyon (VF)

131.Dental girişim planlanan hastada enfektif endokardit profilaksisi gerektirmeyen durum aşağıdakilerden hangisidir? A) Kalp yetmezliği olan hasta B) Protez aort kapak replasmam olan hasta C) Enfektif endokardit öyküsü olan hasta D) Düzeltilmemiş siyanotik kalp hastalığı olan hasta E) Ventriküler septal defektl olan hastada düzeltme sonrası rezidüel defekt

135.

Aşağıdakilerden hangisi periferik arter hastalığı için doğru değildir? A) Özellikle sigara ile ilişkilidir. B) Öyküde kladikasyo intermlttans varlığı tipiktir. C) Kadınlarda daha sık görülür. D) Tedavide antlagregan ve vazodllatör İlaçlar verilir. E) Klinik olarak tutulan ekstremitede ağrı, karakteristiktir.

132.EIN altı yaşında kadın hasta göğüs ağrısı şikâyeti ile acil servise başvuruyor. Şikâyetinin erkek arkadaşıyla yaptığı tartışma sonrasında başladığı öğreniliyor. Fizik muayenesinde kardlyak açıdan bir özellik saptanmıyor. Kan basıncı 120/85 mmHg, nabız 90/dakika ölçülüyor. Elektrokardiyografisinde prekordlyal derivasyonlarda 1 mm üzerinde ST segment elevasyonu saptanarak akut anterior miyokard Infarktüsü ön tanısı İle koroner anjiyografi yapılıyor. Hastanın anjiografisinde patolojik bulguya rastlanmıyor, ancak yapılan ekokardlyografide apikal bölgede balonlaşma saptanıyor.

136. I. Nonkazeifiye granülomlar İle karakterize multisistemik hastalıktır. II. En sık cilt bulgusu “lupus pernio”dur. III. Göz bulgusu olarak özellikle anterior üvelt görülür. IV. Kemik İliği tutulumu olarak en sık hematolojik bulgusu lenfopenldlr. Yukarıdakilerden hangileri sarkoidoz için doğrudur?

Bu vakada en olası tanı aşağıdakilerden hangisidir?

A) B) C) D) E)

A) Hlpertrofik kardiyomiyopati B) Dilate kardiyomiyopati C) Takotsubo kardiyomiyopatisi D) Akut miyokardit E) Akut perikardit

18

I ve II I, II ve IV III ve IV I, III ve IV I, II, III ve IV

137.Elli beş yaşında obez kadın hasta ani başlayan nefes darlığı, göğüs ağrısı ve genel durum bozukluğu sebebiyle acil servise getiriliyor. Hastanın öyküsünden, daha önce derin ven trombozu geçirdiği, 2 hafta önce kalça operasyonu yapıldığı ve bir süredir kronik böbrek hastalığı nedeniyle takip edildiği öğreniliyor. Fizik muayenesinde taşipneik olan hastada S3 duyuluyor. Kan basıncı 110/70 mmHg, solunum sayısı 32/daklka ve kalp hızı 102/dakika olarak saptanıyor.

140.Son dönem böbrek yetmezliği nedeni ile diyaliz tedavisi uygulanan 55 yaşındaki kadın hastanın geçmiş kayıtları incelendiğinde son 1 yıldır fosfat ve parathormon değerlerinin kontrol edilemediği anlaşılıyor. Bu hastanın son laboratuvar değerlerinde kalsiyum 12 mg/dL, fosfat 8 mg/dL, parathormon düzeyi 1100 pg/mL saptanıyor. Bu hastada en olası tanı aşağıdakilerden hangisidir? A) Primer hiperparatiroidi B) Sekonder hiperparatiroidi C) Tersiyer hiperparatiroidi D) Psödohipoparatlroldi E) Aç kemik sendromu

Bu hastada pulmoneremboli tanısını doğrulamak için İlk yapılacak tetkik aşağıdakilerden hangisidir? A) Toraks BT anjiyografi B) Ventilasyon perfüzyon sintigrafisi C) D-dimer düzeyi D) Ekokardiyografi E) Elektrokardiyografi

141.Hipoglisemiye yol açan aşağıdaki sebeplerden hangisinde C - peptid düzeyinde artış beklenmez?

138.Aşağıdakilerden hangisi tüberküloz için doğru

değildir?

A) B) C) D) E)

A) Bakterinin hücre duvarı lipitten zengin olduğu İçin aside dirençli boyanma olur. B) En sık bulaş yolu damlacık Inhalasyonu İle olur. C) Bulaştırıcılık riski en yüksek olan şekil, kavlter tüberkülozdur. D) Postprimer tüberkülozda en sık lenf nodu tutulumu beklenir. E) Postprimer tüberküloz özellikle apekstekl Simon odağından kaynaklanır.

139.Kırk dört yaşında amfizem tanısı ile takip edilen kadın hasta ani başlayan nefes darlığı ve göğüs ağrısı şikâyeti ile acil servise getiriliyor. Fizik muayenesinde; sağ hemitoraksta vibrasyon toraslk azalmış, perküsyonda hipersonarite alınıyor. Oskültasyonda sağ tarafta solunum sesleri alınmıyor. Hastanın posterlor anterior akciğer grafisi aşağıdaki gibidir.

Sülfonilüre kullanımı İnsülinoma Ekzojen insülin kullanımı Nesidoblastozis Beta hücre antikoru varlığı

142. I. Flushlng II. Llpemla retinalis III. Gut atağı IV. Hepatotoksislte Yukandakilerden hangisi/hangileri niasinin yan etkileri arasında yer almaktadır? A) Yalnız III B) I ve III C) I ve IV D) II ve III E) II ve IV

143.0tuz İki yaşında kadın hasta aşırı terleme, sinirlilik, çarpıntı ve boynunda ağrı şikâyetleri ile başvuruyor, öyküsünden, bir hafta önce üst solunum yolu enfeksiyonu geçirdiği öğreniliyor.

Bu bulgularla hastada en olası tanı aşağıdakilerden hangisidir?

Bu hastada aşağıdaki laboratuvar bulgularından hangisinin saptanması en olasıdır?

A) Pnömoni B) Atelektazi C) Pnömotoraks D) Ampiyem E) Akciğer kanseri

A) Eritrosit sedimentasyon hızı 10 mm/saat B) T4 ve T3 düşük, TSİH yüksek C) Radyoaktif iyot alımı artmış D) Anti-tiroid peroksidaz antikoru negatif E) Anti-tiroglobUI in antikoru pozitif

19

144.

Aşağıdaki seçeneklerden hangisi fonksiyonel dispepsi kriterleri içerisinde ver almaz? A) Altı ay önce başlayan karında şişkinlik hissi B) Üç aydır devam eden midede yanma C) Üç aydır devam eden epigastrik ağrı D) Üst gastrointestinal endoskopide erozif gastrit bulgusu E) Altı aydır devam eden çabuk doyma ve hazımsızlık hissi

148.0tuz yaşında kadın hasta halsizlik, yorgunluk ve ateş şikâyeti İle başvuruyor. Öyküsünden, güneşe çıkınca cildinde kızarıklıklar olduğu, zaman zaman elde küçük eklemlerde ağrı ve şişlik olduğu, ancak kendiliğinden geçtiği öğreniliyor. Hastanın laboratuvar testlerinde normokrom normositer anemi, lenfopenl, kreatinin yüksekliği ve tam İdrar tetkikinde protelnüri saptanıyor. Bu hastada böbrek tutulumu ile en yakından ilişkili olan antikor aşağıdakilerden hangisidir? A) Antinükleer antikor B) Anti ds-DNA C) Anti-sentromer D) U1RNP E) Anti-histon

149.Otuz yaşında erkek hasta sırt ve bel ağrısı şikâyeti İle başvuruyor. Ağrısının özellikle geceleri uykudan uyandırdığını ve hareket etmekle azaldığını belirtiyor, özgeçm işinde özellik saptanmayan hastanın fizik muayenesinde Schober testi pozitif bulunuyor. Laboratuvar testlerinde normositer anemi dışında özellik belirlenmeyen hastanın HLA-B27 testi pozitif olarak saptanıyor.

145.Yirmi iki yaşında erkek hasta yaklaşık 6 aydır devam eden İshal ve kaşıntı şikâyeti İle başvuruyor. Hastanın özgeçmişinde bir özellik bulunmamakta olup yapılan fizik muayenesinde ekstremltelerln ekstansör yüzlerinde ve sırtta yaygın kaşıntılı vezlkülopapüler lezyonlar görülüyor. Hastanın laboratuvar testlerinde mlkroslter anemi ve hipoalbümineml saptanıyor, lenfosit sayısı normal bulunuyor.

Bu hastada hastalığın seyrinde aşağıdaki bulgulardan hangisi beklenmez?

Bu hastada saptanması beklenen en olası HLA gen varyasyonu aşağıdakilerden hangisidir?

A) Anterior üveit B) Osteoporoz C) Akciğer aplkal lob fibrozu D) Aort koarktasyonu E) IgA nefropatisi

A) B51 B) B27 C) DR2 D) DQ2 ve DQ8 E) DR3

146.

150.Seksen beş yaşında kadın hasta sağ tarafını görememe şikâyeti İle polikliniğe başvuruyor. Hastanın öyküsünden daha önce periferlk arter hastalığı sebebiyle sol bacağından ameliyat olduğu ve 10 yıl önce kardlyak by-pass olduğu öğreniliyor. Hastanın muayenesinde sağ homonim hemianopsl dışında patoloji saptanmıyor.

I. Entekavir II. Sofosbuvir III. Lamlvudin IV. Boseprevir Yukarıdaki ilaçlardan hangileri kronik hepatit C tedavisinde kullanılır? A) Yalnız II B) I ve IV C) II ve III D) II ve IV E) III ve IV

Kranyal tomografik görünümü yukarıda verilen hastada en olası tanı aşağıdakilerden hangisidir?

147.Hepatik ensefalopati tedavisinde aşağıdakilerden hangisinin yeri yoktur?

A) Anterior serebral arter Iskemlsl B) Orta serebral arter Iskemlsi C) Sağ posterlor serebral arter Iskemlsl D) Sol posterlor serebral arter Iskemlsi E) Sol posterlor serebellar arter Iskemlsl

A) Laktüloz verilmesi B) Metronidazol verilmesi C) Neomisin verilmesi D) Tuz kısıtlaması E) L ornitin - L asparagin (LOLA) verilmesi

20

151 .Yirmi dört yaşında kadın hasta baş ağrısı, bulantı, kusma şikâyetleri ile acil servise başvuruyor. Üç yıldır oral kontraseptif kullanan hastada sistemik hastalık öyküsü saptanmıyor. Hastanın göz dibi muayenesinde bilateral papiller ödem saptanıyor.

154.Majör depresyon tanısıyla takip edilen elli sekiz yaşında kadın hasta acil servise kardiyak aritmi ile getiriliyor. Acil servisteki izlemi sırasında hastada epileptik nöbet gelişiyor. Eşinden alınan öyküden, hastanın kullandığı psikiyatri ilacını intihar amacıyla yüksek dozda almış olabileceği öğreniliyor. Bu hastadaki tabloya yol açma olasılığı en vüksek olan ilaç aşağıdakilerden hangisidir? A) Sitalopram B) Klomipramin C) Fluoksetin D) Venlafaksin E) Atomoksetin

Yukarıda radyolojik görüntülemesi verilen hastada sq olası tanı aşağıdakilerden hangisidir?

155.Aşağıdakilerden hangisi ana ve çocuk sağlığı hizmetlerini değerlendirmede diğerlerine göre daha yaygın ve güvenilir bir ölçüt olarak kullanılabilir?

A) Kavernöz sinüs trombozu B) Transvers sinüs trombozu C) Süperior sagital sinüs trombozu D) Menengiom E) Ependimom

A) Yaşa özel ölüm hızı B) Postneonatal ölüm hızı C) Fatalite hızı D) Kaba ölüm hızı E) Perinatal ölüm hızı

156.Bir araştırmacı, hastane kayıtlarından 150 kolon kanserli ve 300 kolon kanseri olmayan hastayı çalışmaya alıyor. Hastaları yaşam boyu kabızlık şikâyeti varlığı yönünden değerlendiriyor. Kolon kanseri olanların 90’ında, kolon kanseri olmayanların ise 60’ında kabızlık şikâyeti var olarak saptanıyor.

152.Epilepsi tanısı olduğu öğrenilen 22 yaşında kadın hasta 112 tarafından acil servise getiriliyor. Hastanın 20 dakikadan uzun süredir nöbet geçirmekte olduğu öğreniliyor. I. Fenition intravenöz II. Hasta stabilizasyonu ve A-B-C kontrolü III. Lorazepam intravenöz IV. Propofol anestezisi

Bu araştırmada tahmini rölatif risk aşağıdakilerden hangisidir?

Bu hastaya yaklaşımda yapılacak uygulamaların en doöru sıralaması aşağıdakilerden hangisidir?

A) 0,3 B) 2 C) 6 D) 0,4 E) 4

A) I - II - III - IV B) I - I I I - I I - I V C) I I - I - I I I - I V D) II - III - I - IV E) III - II - I - IV

157.Yirmi yaşında kadın hasta el ve ayak parmakları ile kol ve bacaklarında hedef tahtası şeklinde deri döküntüleriyle başvuruyor. Hastanın yapılan muayenesinde makül, papül ve veziküllerden oluşan farklı döküntülerinin olduğu saptanıyor.

153. I. Hastalığın erken yaşta başlaması II. Depresyon öyküsünün varlığı III. Katatoni Yukarıdakilerden hangisi ya da hangileri şizofrenide kötü prognostik faktörlerdendir?

Bu hastadaki lezyonun gelişiminde rol oynayan eo olası neden aşağıdakilerden hangisidir?

A) Yalnız I B) I ve II C) I ve III D) II ve III E) I, II ve III

A) B) C) D) E)

21

Herpes simpleks virüs enfeksiyonu Staphylococcus aureus enfeksiyonu Besin alerjisi Maligniteler İlaç reaksiyonu

158.Aşağıdaki sinyal iletim proteinlerinden hangisinin genetik mutasyonunda malign melanom riski en fazla artmıştır?

162.Nükleer tıpta tanı ve tedavi için kullanılan maddelere ne ad verilir? A) B) C) D) E)

A) NOTCH-1 B) ABL C) JAK2 D) GNAS E) BRAF

159.Elli sekiz yaşında erkek hasta elinde olan hareket kısıtlılığı ve şekil bozukluğu sebebiyle doktora başvuruyor, öyküsünden bilinen bir hastalığı olmadığı öğrenilen hastanın bisiklet sürücüsü olduğu öğreniliyor. Fizik muayenesinde aşağıdaki resimde görüldüğü gibi dördüncü parmağının fleksiyonda kaldığı görülüyor.

Bu hastalık için aşağıda verilenlerden hangisi veya hangileri risk faktörüdür? I. Kadın cinsiyet II. Miyokard infarktüsü III. Siroz IV. Diabetes mellitus A) Yalnız I B) I ve III C) I, II ve III D) II, III ve IV E) I, II, III ve IV

160. I. Kısa dalga diatermi II. Mikrodalga diatermi III. Ultrason IV. Girdap banyoları Yukarıdakilerden hangisi/hangileri fizik tedavide kullanılan derin ısıtıcılar arasında yer alır? A) C)

Yalnız I I ve III E)

B) I, II ve III D) II, III ve IV I, II, III ve IV

161.Vezikoüreteral reflünün gösterilmesi için aşağıdaki tetkiklerden hangisi yapılmalıdır? A) Manyetik rezonans ürografi B) Ultrasonografi C) Voiding sistoüretrografi D) Retrograd üretrografi E) Bilgisayarlı tomografi

22

İzotop Radyoizotop Biyoaktif ajan Radyofarmasi Radyofarmasötik

2021 DÖNEMİ 3. DENEME SINAVI KLİNİK BİLİMLER TESTİ SORULARI

https://t.me/tusldtabi

Bu metinde sırasıyla Dahili Bilimler, Pediatri, Cerrahi Bilimler, Kadın Doğum soruları bulunmaktadır.

124.Yetmiş iki yaşında erkek hasta kilo kaybı ve halsizlik nedeniyle başvuruyor. Laboratuvar İncelemelerinde, lökosit 38.600/mm3, monosit 5100/mm3, hemoglobin 7.8 g/dL, trombosit: 64.000/mm3 saptanıyor. Hastada son iki yıldırtekrarlayan üst ve alt solunum yolları enfeksiyonlarına ek olarak daha önceden tam kan sayımında anormal bulgu yokken son altı ay İçinde giderek artış gösteren monosltoz, anemi ve trombosltopeni olduğu öğreniliyor. Perlferlk yaymada %20-25 civarında olgun görünümlü monosltler, seyrek normoblastlar, m iyelosit ve promlyelosltler gözleniyor. Kemik İliği İncelemesinde; hlpersellülerlte, %28 oranında monositer seri elemanları, %3 oranında blastlk hücre İzleniyor. Erltroslter ve granüloslter seride hafif dismorfizm ve grade II retlkülln lif varlığı gözleniyor.

121. Elli altı yaşında erkek hasta halsizlik, yorgunluk ve solukluk nedeniyle başvuruyor. Fizik muayenede konjunktiva ve palmar çizgilerde solukluk haricinde anormal bulgu saptanmıyor. Hemogramda hemoglobin 9.2 g/dL, MCV 85 fL, MCH 30 pg, MCHC %25, retikülosit %0,5, trombosit ve beyaz küre sayıları normal saptanıyor. Bu hastada anemi etiyolojisine yönelik en az olası tanı aşağıdakilerden hangisidir? A) Kronik böbrek yetmezliği B) Romatoid artrit C) Hipotiroidizm D) Renal hücreli kanser E) Soğuk tip otolmmün hemolitik anemi

Bu hastada en olası tanı aşağıdakilerden hangisidir? A) Primer miyelofibroz B) Kronik lenfosltlk lösemi C) Polisitemia vera D) Mlyelodisplastlk sendrom E) Kronik mlyelomonositer lösemi

122. Akut lenfoblastik lösemi (ALL) tanısıyla allojenik kemik iliği nakli yapılan bir hastada aşağıdaki aşılardan hangisinin transplantasyondan sonraki ilk bir yıl içinde yapılması uygun değildir?

125. I. Erlotinib - Akciğer kanseri II. Trastuzumab - Mide kanseri III. Lapatlnlb - Pankreas kanseri IV. Setukslmab - Kolon kanseri

A) Difteri-Tetanoz B) Hepatit A C) Hepatit B D) Kızamık E) Pnömokok

Yukarıdaki “malignité - kullanılan ilaç” eşleştirmelerinden hangisi/hangileri doğrudur? A) I ve II B) I, II ve IV C) I ve IV D) II, III ve IV E) II ve IV

123.Yetmiş yaşında kadın hasta halsizlik, yorgunluk, baş dönmesi, kulak çınlaması ve bulanık görme şikâyeti ile başvuruyor. Fizik muayenesinde yaygın lenfadenopatiler saptanıyor ve dalak 8 cm palpe ediliyor. Kan sayımı normal olarak sonuçlanan hastanın sedlmentasyon hızı 88 mm/ saat ölçülüyor. Serum protein elektroforezinde monoklonal gamopatl olduğu görülüyor.

126.Nötropenik ateş tanısı konulan bir hastada risk belirlenmesinde kullanılan MASCC (Multinational Association for Supportive Care in Cancer) skorlamasinda aşağıdaki parametrelerden hangisi ver almaz? A) Hastanın yaşı B) Kan basıncı C) Solunum sayısı D) Fungal enfeksiyon öyküsü E) Kronik obtrüktlf akciğer hastalığı varlığı

Bu hastanın kemik iliği incelemesinde özellikle aşağıdaki hücre tiplerinden hangisinde artış beklenir? A) Plazma hücresi B) Miyeloblast C) Lenfoblast D) Olgun lenfosit E) Lenfoplazmositer hücre

18

131.

127.Otuz yedi yaşında kadın hasta üç gündür devam eden kusma ve ishal nedeniyle acil servise getiriliyor. Son 6 saatte idrar miktarında azalma olan hastanın kan üre azotu (BUN): 73 mg/dL, serum kreatinin 2,1 mg/dL, spot idrar sodyumu 5 mEq/dL olarak bulunuyor.

I. Birinci kalp sesi (S1) atriyoventriküler kapakların kapanması İle oluşur. II. Mitral darlığında S1 şiddetlenmesi görülür. III. İleri sol kalp yetmezliğinde geniş S2 çiftleşmesi görülür. IV. Opening snap diyastolün erken fazında duyulmaktadır.

Bu hastada aşağıdaki laboratuvar bulgularından hangisinin bulunması beklenmez? A) BUN/kreatinin oranı >20 olması B) İdrar tetkikinde eritrosit silendirlerinin görülmesi C) İdrar danslteslnln >1020 olması D) Fraksiyone sodyum 100 mEq/l D) Serum kreatinin > 1.5 mg/dl E) Normal tiroid fonksiyonu

65

www.tusem.com.tr

TUSEM

Klinik Bilimler / T 47 - 1

TIPTA UZMANLIKSINAVI EŞİTİM MERKEZİ

10 - D Uygunsuz ADH sendromunda, hipovolemi veya hiperozmolarite olmadan artmış ADH salınımı mevcuttur. Artan ADH salınımı ile hiponatremi pahasına su tutulumu olur. Su tutulumu sonucunda artan volüm nedeni ile Renin - Anjiotensin - Aldosteron sistemi baskılanır ve ANP salınımı artar. ANP salınımı ile sodium atılımı ve övolemi sağlanır. Uygunsuz ADH salınımı laboratuvar bulguları; •

Serum ozmolaritesi 40 mEq/l



İdrar ozmolaritesi > 100 mEq/l



Serum ürik asit, kreatinin, BUN düzey düşüklüğü



Normal tiroid, adrenal ve böbrek fonksiyonu

Uygunsuz ADH salınımında serum kreatinin artışı saptanmaz.

11.

Aşağıdakilerden hangisi tiroid nodülünde m alignite lehine risk faktörü değildir ? A) Boyuna radyasyon öyküsü B) > 4 cm büyük nodül C) Kadın cinsiyet D) Tiroid dışı yayılım E) Servikal lenfadenopati

11 - C Tiroid nodülleri sıklıkla benign olmakla birlikte bir takım risk faktörleri varlığında malignite riskleri artar. Tiroid nodülü malignite risk faktörleri; •

Erkek cinsiyet



Baş - Boyuna radyasyon öyküsü



4 cm büyük nodül



Aile öyküsü



Tiroid dışı yayılım



Servikal lenfadenopati



Vokal kord paralizi

12.

Aşağıdakilerden hangisi subakut granülom atöz tiroiditin özelliklerinden biri değildir? A) Sıklıkla viral bir üst solunum yolu enfeksiyonunu takip eder. B) Tiroid palpasyonla çok ağrılıdır. C) Akut faz yanıtı yüksek saptanır. D) Otoantikorlar yüksek titrededir. E) Tedavide antiinflamatuvar ilaçlar kullanılır.

12 - D Subakut granülomatöz tiroidit (De quarvein tiroiditi); sıklıkla viral bir üst solunum yolu enfeksiyonunu takiben oluşur. Tiroid çok ağrılıdır. Akut faz yanıtı yüksek saptanır. Otoantikor sıklıkla saptanmazken antiinflamatuvar tedavi faydalıdır.

www.tusem.com.tr

66

TUSEIUT

Klinik Bilimler / T 47 - 1

TIPTA UZMANLIK SINAVI EĞİTİM MERKEZİ

13.

Üriner sistem infeksiyonlarında aşağıdaki antibiyotiklerden hangisi kullanılam az? A) Levofloksasin B) Fosfomisin C) Sefuroksim D) Moksifloksasin LU

Sefiksim

13 - D Moksifloksasin karaciğerde metabolize olur ve başlıca hepatik yolla atılır, renal yolla atılmadığından idrara geçmez bu nedenle üriner infeksiyonların tedavisinde kullanılmaz. Moksifloksasin hepatik yolla atıldığındna böbrek yetmezliğinde doz ayarı yapılmasına gerek yoktur. Antianaerob etkinliğinin olması diğer kinoon grubu antibiyotiklerden en önemli farkıdır Bu nedenle toplum kaynaklı sekonder peritonitlerde (apandisit perforasyonu vb.) penisilin alerjisi olan hastalarda tedavide kulla­ nılabilir (Sekonder peritonit etkenleri olan E.coli ve bacteroides fragilis'e karşı etkili olduğu için)

14.

55 yaşında alkolik erkek hasta hematez ve melena şikayetleriyle acile başvurduğunda, fizik incelemede bilinci açık, hipotansif-taşikardik, gözlerinde sararma, ellerinde Dupuytren kontraktürü, batında asit ve pretibial ödem saptanıyor. Genel destek tedavisi başlanan ve acil endoskopiye hazırlanan bu hastada aşağıdakilerden hangisinin mutlaka yapılması gerekir? A) Oral PPİ B) Beta-bloker tedavisi C) Sangstaken-Blackmore tüpü takılması D) Oral NSAİİ E) iv terlipressin veya oktreotid başlanması

14 - E Alkol öyküsü, fizik inceleme ipuçları sirozu düşündürmektedir, varis kanamasında endoskopi öncesi stabilizasyon aşamasında por­ tal basıncı azaltan ilaçlardan da başlanır. Beta-bloker tedavi akut kanama döneminde hipotansiyon riskinden dolayı kullanılmaz (varislerin primer ve sekonder profilaksisinde uygulanır). Sangstaken-Blackmore tüpü ise geçici mekanik tampon yaparak kana­ mayı durudurabilir, diğer yöntemlere köprü olarak seçilebilir (endoskopik tedavi için uzak bir merkeze sevkedilirken, endoskopik tedavi başarısız olmuş-TİPS hazırlıkları yapılırken gibi...).

15.

Aşağıdaki durum ların hangisi steatore şeklinde yağlı ishal nedenlerinden biridir? A) Kronik pankreatit B) Amebik kolit C) Ülseratif kolit D) Laktoz intoleransı E) Laktüloz kullanımı

15 - A Kronik pankreatit sık görülen malabsorpsiyon nedenlerinden biridir, dolayısıyla yağlı diyare yapar. Amebik kolit ve ÜK inflamatuvar diyare; laktoz intoleransı ve laktüloz kullanımı ise osmotik diyare yapar.

67

www.tusem.com.tr

TUSEM

Klinik Bilimler / T 47 - 1

TIPTA UZMANLIK SINAVI EŞİTİM MERKEZİ

16.

50 yaşında bayan hastanın kolonoskopisinde çok sayıda ham artom atöz polip izleniyor. Fizik incelem ede alopesi ve trırnak distrofisi saptanan, ailesinde polipozis öyküsü olm ayan hastada hangisi en olası tanıdır? A) Peutz-Jeghers sendormu B) Gardner sendromu C) Lynch sendromu D) Cowden hastalığı E) Cronkhite-Canada sendromu

16 - E Cronkhite-Canada sendromu tarif edilmektedir, son dönemde bu sendromun otoimmün olduğu iddia edilemktedir, herediter değildir.

17.

60 yaşında erkek hasta, halsizlik, iştahsızlık, efor dispnesi, karın şişliği ve kilo kaybı nedeniyle başvurduğunda; hemoglobin 9 gr/dl, KCFT hafif artmış, USG'de yaygın asit, BT'de asite ek olarak omentumda ve barsak duvarında kalınlaşma saptanırken; serum-asit albümin gradienti 0.8 saptanıyor. En olası asit nedeni aşağdakilerden hangisidir? A) Malign asit (GİS tm-peritoneal karsinomatozis) B) Siroz C) Sekonder bakteryel peritonit D) Tüberküloz peritoniti E) Konjestif kalp yetmezliği

17 - A Anemi, kilo kaybı ve BT'de barsak duvarı-omentum kalınlaşması ileri evre kolon tümörüne bağlı peritoneal karsinomatzis ve malign asit lehine bulgulardır. Sirotik asitin tersine serum-asit albümin gradienti düşüktür (60



>1 ekstranodal tutulum



Ann-arbor evre 3-4



Yüksek LDH



ECOG performans statüsü >1

Nöropatisi olan bir kanser hastasında aşağıdaki ilaçlardan hangisi nöropati üzerine olumsuz etkisi en azdır? A) Brentuximab vedotin B) Dosetaksel C) Vinkristin D) Sisplatin E) Topotekan

23 - E Nöropati yapan kemoterapi ilaçları hem TUS hem de yurtdışı sınavlarda popüler bir sorudur. •

Genel olarak mikrotübül zehirlerinin hepsinin ortak yan etkisidir (taksanalr, vinka alkoloidler gibi). Aynı zamanda hodgkin len­ foma tedavisinde kullandığımız CD30 monoklonal antikoru brentuximab vedotin ve platinler içerisinde sisplatinde de çok sık görülen bir yan etkidir.



Şıklarda olmamakla beraber bu konuda asla bortezomib de unutulmamalıdır.



Topotekan bir topoizomeraz-1 inhibitörüdür ve diğerleri kadar nöropati yapması beklenmez.

79

www.tusem.com.tr

TUSEM

Klinik Bilimler / T 47 - 5

TIPTA UZMANLIK SINAVI E6İTİM MERKEZİ

24.

30 yaşında bayan hasta alti aydır olan bacaklarında şişlik nedeniyle başvurduğunda 6 gr/gün proteinüri saptanıyor. BUN, kreatinin normal; Vücut kitle indeksi 38, H bA lc % 5.5, viral-otoimmün seroloji negatif, komplemanları normal olan hasta­ da nefrotik sendromun en olası nedeni hangisidir? A) Ailevi akdeniz ateşi B) Membranöz nefropati C) FSGS D) IgA nefropatisi E) Diyabetik nefropati

24 - C • •

FSGS, primer veya sekonder olabilir. İnfeksiyon (HİV, parvovirüs, HBV), eroin kullanımı, orak hücreli anemi, obezite, veziko üreteral reflü, malignite, soliter böbrek ve kronik transplant rejeksiyonu gibi çok sayıda sekonder nedeni vardır.



25.

Obezite ilişkili glomerulopatide denilen özellikle ciddi-morbid obezlerdeki FSGS tablosu ilgi çeken güncel bir konudur.

HBV sirozu nedeniyle takipli 50 yaşında erkek hasta bilinç bulanıklığı geliştiği için getirildi. Asit tedavisi nedeniyle kul­ landığı ilaçların dozlarının 1 hafta önce artirıldığı öğrenildi. Hb 11 gr/dl, BK 3000/mm3 Plt 130.000/mm3; BUN 18 mg/dl, kre 1 mg/dl, Na 134 mEq/L, K 2.4 mEq/l, Ca 9 mg/dl saptanan hastada, hangi metabolik bozukluk hepatik ensefalopatiyi tetiklemiştir? A) Hipernatremi B) Hiperkalsemi C) Hipokalemi D) Hiperkalemi E) Hiponatremi

25 - C Asit tedavisinde kullanılan furasemid aşırı dozda hipokalemi yapar, hipokalemi ensefalopatiyi tetikleyen faktörlerden biridir.

www.tusem.com.tr

80

TUSEIUT

Klinik Bilimler / T 47 - 5

TIPTA UZMANLIK SINAVI EŞİTİM MERKEZİ

26.

Aşağıdakilerden hangisi romatoid artiritin (RA) güncel tanısında kullanılan tanı kriterlerinden biri değildir? A) Eklem tutulumu B) Böbrek tutulumu C) Sedimantasyon / CRP artışı D) Semptom süresi E) RF/ CCP pozitifliği

26 - B •

RA, böbrekleri genelde etkilemez. Dolayısıyla tanı kriterleri arasında böbrek tutulumu yoktur.



B şıkkı haricindeki bulgular, RA'nın 2010 ACR-EULAR kriterleri arasındadır.



Böbrek tutulumu SLE'nin tanı kriterlerinden biridir.

27.

Sjögren sendromunun en sık extra-glandüler tutulumu aşağıdakilerden hangisidir? A) Kas-iskelet sistemi bulguları B) Derin ven trombozu C) Raynound sendromu D) Böbrek tutulumu E) Periton tutulumu

27 - A • Sjögren sendromunun en sık extra-glandüler tutulumu kas iskelet sistemi bulgularıdır (artrit-artalji). • Aynı zamanda SLE'de de cilt tutulumundan sonra en sık tutulum artirt-artalji şeklindedir. Derin ven trombozu, antifosfolipid sendromunun en sık vasküler tutulum şeklidir (B şıkkı). • Raynound sendromu, sistemik sklerozun en sık cilt dışı tutulumudur (C şıkkı). • Gut'da eklemlerden sonra en sık böbrekler tutulur (D şıkkı). •

28.

Periton tutulumuna (peritonit) bağlı karın ağrısı, tipik olarak ailevi akdeniz ateşinde görülür ve ateşten sonra en sık semptomdur.

Vankomisine dirençli enterokok infeksiyonlarında aşağıdaki antibiyotiklerden hangisi kullanılabilir? A) Ampisilin Cû

Seftriakson

C) Daptomisin D) Meropenem LU

Aztreonam

28 - C Vankomisine dirençli enterokok infeksiyonlarının tedavisinde; daptomisin, tigesiklin, linezolid kullanılabilir.

81

www.tusem.com.tr

TUSEM

Klinik Bilimler / T 47 - 5

TIPTA UZMANLIK SINAVI E6İTİM MERKEZİ

29.

Çift görme yakınmasıyla acil servise başvuran 35 yaşında kadın hasta bu yakınmasının daha önce de birkaç kez olup geçtiğini ifade etmiştir. Muayenesi normal bulunmuş ve diffüzyon MRG'sinde patolojik bulgu saptanmamışttır. Gelişinden üç saat sonra tekrarlanan muayenesinde sağ gözün primer pozisyonda dışa deviye olduğu görülmüştür. Bu olguda en olası tanı nedir? A) 4.kraniyal sinir paralizisi B) 6. kraniyal sinir paralizisi C) Lateral meduller sendrom D) İnternükleer oftalmopleji E) Myastenia gravis

29 - E Bu soruda hastada tekrarlayan şikayetler olması ve bir süre sonrası şikayetlerin olması MG düşündürmektedir. Myastenia Gravis •

Myastenia gravis (MG), hareketle artan kas güçsüzlüğü (bitkinliği) ile karakterize, öncelikle okülobulber kasları tutan, postsinaptik yerleşimli nikotinik asetilkolin reseptörlerinin (AChR) hedef alındığı otoimmün kökenli bir hastalıktır.



Hastalığın en önemli özelliği yorulmakla artan ve dinlenmekle en azından kısmen düzelen kas güçsüzlüğüdür. Hastalar sabah­ ları düzeldiklerini, belirtilerin akşama doğru ya da yorulunca arttığını ifade ederler.



Hastalık remisyon ve alevlenmelerle seyreder.



Hastalık çoğu zaman oküler belirtilerle, en sık da pitozla başlar. Buna çift görme eşlik eder. Hastaların çoğunda oküler bulgulara kısa zamanda bulber (orofarengeal) kaslara ve ektremite kaslarına ait belirtiler eklenir. Nazone konuşma dikkat çekicidir.

30.

Altmış iki yaşında erkek hasta dört ay önce sol humerusta travmatik bir kırığı izleyerek sol kolunda el parmaklarına kadar yayılan keskin, derin ve sıcak bir ağrı ve soğuğa aşırı duyarlılıktan yakınmaktadır. Elde distonik bir postür ile birlikte şişlik, kızarıklık ve sıcaklık izlenen bu hastada en olası tanı nedir? A) Monomelik amiyotrofi B) Servikal 6-7 disk hernisi C) Kompleks bölgesel ağrı sendromu D) Servikal 5-8 kök avülsiyonu E) Nevraljik amiyotrofi

30 - C Fizyopatolojide olay, sempatik sinir sisteminin hiperaktivasyonu sonucu gelişir. (otonomik disrefleksi gibi) KBAS Tip 1 •

Başlattıcı zarar veren bir olayı takip eder.



Tek periferik sinir alanının ötesine yayılan, başlangıçtaki olayla oranttısız spontan ağrı veya allodini / hiperaljezi görülür.



Ağrı bölgesinde ödem, deri kan akımında bozukluk ve anormal sudomotor aktivite görülür.



Ağrının ve disfonksiyonun derecesini açıklayacak başka durumların varlığı tanıyı geçersiz kılar.

www.tusem.com.tr

82

TUSEIUT

Klinik Bilimler | T 47 - S

TIPTA UZMANLIK SINAVI EŞİTİM MERKEZİ

31.

Otuz iki yaşında kadın hasta, son bir haftadır süren şiddetli ve sürekli başağrısından yakınmaktadır. Son 6 aydır tüp bebek uygulaması için östrojen kullanan ve 20 paket-yıl sigara öyküsü olan hastanın muayenesinde TA:120/80 mmHg, BMI:33 kg/m2, bilateral papil ödemi ve uykuya meyil saptanmıştır. Valsalva manevrası ile başağrısı artan hastanın tam kan sayımı, PT ve INR değerleri normal bulunmuştur. Kontrastsız beyin MRG'si de normal olan bu hastaya tanı için bu aşamada hangi diyagnostik test yapılmalıdır? A) MR perfüzyon B) Lomber ponksiyon C) MR venografi D) MR anjiografi E) Transkraniyal Doppler Ultrasonografi

31 - B Psödotümör Serebri (İdiyopatik İntrakranyal Hipertansiyon) • İntrakranial yer kaplayan lezyon olmadan kafa içi basıncının yükselmesidir. •

Kliniğinde yeni başlamış baş ağrısı, görme bozukluğu, diplopi, pulsatil tinnitus, bulantı-kusma, baş dönmesi, göz hareketleri ile ortaya çıkan retrobulber ağrı vardır.

• En sık idiopatik olarak gelişir. • PTS'nin patogenezinde BOS emilim defekti mevcuttur. • Çoğu hasta obezdir. • Hemen daima diffuz baş ağrısı başlangıç semptomudur. • Görme semptomları ilk olarak göz kararmaları ile başlar sonra ilerleyerek santral görme kaybına kadar gidebilir. • Özellikle LP yapılmadan BT/MRI ile kafa içi yer kaplayan lezyon mutlaka ekarte edilmelidir. • MR görüntülemede perinöral subaraknoid mesafede genişlemesi boş sella ve nadiren Chiari tip 1malformasyonu görülebilir. • Kesin tanı LP de BOS basıncının 200 mmHg üstünde olması ile konulur.

32.

On beş yaşında erkek çocukta retinada multipl anjiomatoz görüntü yanı sıra böbrek ve pankreasta kistler saptanmıştır. Bu olguda hangi tümör öncelikle düşünülmelidir? A) Glioblastoma multiforme B) Meningioma C) Hemanjioblastoma ru £ o E ~o c QJ Q.

LU

D)

E) Pinealoma

S3

www.tusem.com.tr

TUSEM

Klinik Bilimler | T 47 - S

TIPTA UZMANLIK SINAVI E6İTİM MERKEZİ

32 - C AİLESEL MSS TÜMÖRLERİ En sık rastlanan ilişki fakomatozlardır. •

Nörofibromatozis tip 1

Optik gliom ve menengiom •

Tuberoskleroz

Ependimom, astrositom •

Von-Hippel-Lindau Hastalığı

Hemanjioblastom, feokromasitoma ve retinal tümör

33.

Altmış iki yaşında kadın hasta üç saat önce başlayan vücudun sağ tarafında hissizlik ve yürümede dengesizlik yakınması ile başvurdu. Nörolojik muayenede sağ yarıda yüzü de içine alan, yüzeyel ve derin duyu kaybı saptanıyor. Motor defisiti olmayan bu hastada lezyon nerededir? A) Pons B) Kaudat nükleus C) Frontal korteks D) İnternal kapsül genu E) Ventral posterior talamus

33 - E Talamus: Beyinsapının rostralinde, serebral hemisferlerin derinliğinde yer alır. Talamus somato-sensoriyel duyular, (koku duyusu ha­ riç) görme ve işitme impulslarının kortekse ulaşmadan önceki durağıdır. Lezyonunda kişinin uyanıklık durumunda bozukluk görülür. Talamik sendrom; kontralateral vücut yarısında tüm duyuların kaybına ve daha sonra spontan veya dokunmakla şiddetli ağrıya neden olan bir patolojidir. Hastada motor defisitinin olmaması ve lateralizasyon bulgusu talamus lezyonunu düşündürmektedir.

34.

Akut gelişen parapleji, seviye veren duyu kusuru ve sfinkter kusuru saptanan derin duyuları sağlam bulunan 66 yaşındaki kadın hastada en olası tanı aşağıdakilerden hangisidir? A) Anterior spinal arter tıkanması B) Vertebra metastazı C) Kauda ekina sendromu D) Foramen magnum sendromu E) Anterior serebral arterin iki yanlı tıkanması

www.tusem.com.tr

84

TUSEIUT

Klinik Bilimler / T 47 - 5

TIPTA UZMANLIK SINAVI EŞİTİM MERKEZİ

34 - A Anterior Serebral Arter •

Frontal lob ve serebral hemisferin medial kısmını besler.



Karşı bacakta güçsüzlük ve sensorial kayıp.



İki taraflı anterior serebral arter tutulumunda frontal lobların bilateral etkilenmesi olur ve akinetik mutizm gelişebilir.



İnme hastalarında motor ve duyu defisitler gelişebilmektedir.



Anterior serebral arter altekstremitelerin innervasyonunu sağlayan korteks kısmını beslemektedir. Kanama veya iskemi du­ rumlarında paraplejiye neden olabilmektedir.

35.

Kırk beş yaşında kadın hasta bir yıldır var olan, başının hep aynı tarafında, günde 5-6 kez olan, 20-30 dakika süren şiddetli başağrılarından yakınmaktadır. Ağrı geldiğinde hemen bir yere oturma ya da yatma ihtiyacı duyan indometazin kullanımı sonrası 3 günde ağrıları tamamen geçen hastada tanı nedir? A) Paroksizmal hemikranya B) Kronik küme C) Epizodik küme D) Kronik gerilim tipi başağrısı E) Hemikranya kontinua

35 - A Paroksismal Hemikranya •

Çok seyrek görülen bir primer başağrısıdır.



Kadınlarda daha sık görülür ve 25-40 yaş arasındaki kişilerde rastlanmaktadır.



Ağrı kesinlikle hep aynı tarafta şiddetli olarak ortaya çıkar, kısa süren ve remisyonsuz baş ağrılarıdır. Sıklıkla kroniktir, ancak epizodik formu da tanımlanmışfır.



İlişkili otonom değişiklikler küme başağrısındakine çok benzer ancak tipik özellik indometazinle 3 gün içinde tam kontrol edi­ lebilmesidir.

36.

Optik trakt ile gelen görme duyusu beynin hangi bölgesine iletilir? A) Kaudat nükleus B) Putamen C) Lateral genikulat cisim D) Globus pallidus eksterna E) Globus pallidus interna

85

www.tusem.com.tr

TUSEM

Klinik Bilimler / T 47 - 5

TIPTA UZMANLIK SINAVI E6İTİM MERKEZİ

36 - C N. Opticus • Retinanın nazal ve temporal yarısından gelen görsel impulsları taşıyan optik sinir telleri sella turcica bölgesine kadar gelir. • Burada, her iki gözün nazal retinasından gelen lifler çaprazlaşıp karşıya geçer. • Çaprazlaşan sinir tellerinin oluşturduğu yapıya chiasma opticum denir. • Kiyazmadan sonra görme lifleri tractus opticus adını alır. •

Tractus opticus'taki lifler talamusun corpus geniculatum laterale adı verilen çekirdeğinde sonlanır.



Yani, retinadan başlayarak talamusa kadar kesintiye uğramadan uzanan görme yolları burada sinaps yapar ve radiatio optici adını alarak temporal ve parietal lobların derinliklerinden geçip oksipital lobların iç yüzler bnindeki primer görme korteksine (kalkarin korteks) ulaşır.



Işık refleksiyle ilgili lifler tractus opticus içinde seyreder. Görme liflerinden farklı olarak, bunlar talamusa uğramazlar ve doğru­ dan pretektal bölgeye ulaşır.

37.

Ağrının zamanı, ağrının tipi ve ağrı kaynağı olan hastalık göz önüne alındığında, aşağıdakilerden hangisi sempatik sinir sistemi blokajı uygulanması için en uygundur? A) Akut - Nosiseptif - Miyofasyal Ağrı Sendromu B) Kronik - Nöropatik - Kompleks rejyonel ağrı sendromu C) Kronik - Nosiseptif - Anjina pektoris D) Kronik - Nöropatik - Başarısız bel cerrahisi sendromu E) Akut - Nöropatik - Başarısız bel cerrahisi sendromu

37 - B Nöropatik Ağrı •

Sinir sistemindeki primer bir lezyon veya disfonksiyon sonucu ortaya çıkan ağrı olarak tanımlanmaktadır.



Fizyopatolojide olay, sempatik sinir sisteminin hiperaktivasyonu sonucu gelişir. (otonomik disrefleksi gibi)

Tedavi 1. Fizik tedavi ve rehabilitasyon: İlk ve en önemli yapılması gereken erken egzersiz programıdır. 2. Bilişsel-davranışsal tedavi 3. Girişimsel tedaviler: Sempatik gangliyon blokajları. 4. Farmakolojik tedavi

38.

Lomber spinal stenoz ile ilgili aşağıdaki ifadelerden hangisi yanlıştır? A) Konjenital olabilir. B) Dejeneratif değişiklikler sonucu gelişebilir. C) Belde fleksiyonla artan ağrı tipiktir. D) Cerrahi sonrası gelişebilir. E) En sık alt üç lomber seviyede gözlenir.

www.tusem.com.tr

86

TUSElUr

Klinik Bilimler / T 47 - 5

TIPTA UZMANLIK SINAVI EŞİTİM MERKEZİ

38 - C Spinal Stenoz •

Spinal stenoz genellike omurga dejenerasyonu patofizyolojisinin terminal döneminde görülen sorundur.



Anteriordan diskin basısı, posteriordan lig. flavumun hipertrofisi ve içe kıvrılması, faset hipertrofisi ve osteofitlerin basısı ile kanal daralır.



Klinik olarak en önemli semptom nörolojik kladikasyodur.



Hastaların ağrısı fleksiyonda azalırken ekstansiyonda artmaktadır.



Radyolojik tanıda MR en önemli tetkiktir. Aksiyal görüntülemede kanal çapının 10 mm'den düşük olması tanı koydurur.

39.

Aşağıdakilerden hangisi prolaktinoma cerrahi endikasyonlarından biri değildir? A) Hipofizer apopleksi B) Medikal tedaviye direnç C) Görme problemi D) İlaç yan etkisi E) Mikroadenom

39 - E Prolaktinoma • •

Semptomlar genel olarak hiperprolaktinemi nedeniyle ortaya çıkmaktadır. Hiperprolaktinemi, kadınlarda sekonder amenore veya oligomenore, galaktore ve infertiliteliye, erkeklerde en sık libido azal­ ması ve/veya erektil disfonksiyona neden olur.



Mikroprolaktinomada semptom yok ise tedavi şart değildir izlenirler.



Makroadenomlar mutlaka tedavi edilmelidir.



Tedavi endikasyonları tümörün basısına bağlı veya hiperprolaktineminin sebep olduğu infertilite, amenore/oligomenore, osteoporoz, rahatsız eden hirsutism veya galaktore gibi semptomlardır.



Dopamin agonistleriyle tedavi prolaktinomalarda birinci seçenek tedavidir.

Cerrahi Tedavi •

Makroadenomlarda nörolojik bulgularla birlikte olan apopleksi



Nörolojik semptomlara neden olan kistik makroprolaktinomalar (genellikle dopamin agonistleri ile küçülmezler)



Medikal tedavi ile görme alanı defektinin düzelmemesi



Makroadenomda gebelik istenmesi



Medikal tedavi altındaki gebelikte tümörün büyümesi

87

www.tusem.com.tr

TUSEM

Klinik Bilimler / T 47 - 5

TIPTA UZMANLIK SINAVI E6İTİM MERKEZİ

40.

65 yaşında erkek hasta, son 6 ayda artan múltiple seboreik keratozları nedeniyle dermatoloji polikliniğine başvuruştur. Kilo kaybı şikayeti de olan hastanın endoskopisinde alınan mide -biyopisisi adenokarsinom saptanmıştır. Hastada saptanan mevcut tablonun adı nedir? A) Muir-Torre Sendromu B) Leser-Trelat belirtisi C) Borst-Jadasshon fenomeni D) Wickham fenomeni E) Auspitz fenomeni

40 - B Seboreik Keratoz •

En sık göğüs ve sırtta yerleşen, keskin sınırlı, deriden kabarık, papül, plak ve nodül şeklinde olabilen, pigmente lezyonlardır.



Genellikle 30-50 yaşlarında başlar.



Lezyonlarda folliküler uzantıları olması tipiktir.



Internal malignitelerle ilişkili multiple seboreik keratozlar tarafından oluşturulan erüptif görüntü "leser-trelat" belirtisi olarak adlandırılır.

41.

Skuamöz hücreli deri karsinomunda aşağıdakilerden hangisinin prognoza etkisi yoktur? A) Mitoz sayısı B) Akantolitik tip C) Horizontal yerleşim D) Perinöral tutulum E) Tümör lokalizasyonu

41 - B

42.

Aşağıdaki hastalıkların hangisinde Eritema nodozum görülmez? A) Behçet hastalığı B) El-ayak-ağız hastalığı C) Tüberküloz enfeksiyonu D) Streptokok enfeksiyonu E) İnflamatuar bağırsak hastalıkları

www.tusem.com.tr

88

TUSEIUT

Klinik B ilim le r/T 4 7 - 5

TIPTA UZMANLIK SINAVI EĞİTİM MERKEZİ

42- B Eritema nodozum (EN) Septal pannikülitlerin prototipidir. Değişik antijenik uyaranlara karşı gecikmiş hipersensitivite cevabı olduğu kabul edilir. Kadınlarda 3-6 kat daha fazla görülür ve 20-30 yaş arasında daha sıktır. Klinik olarak prétibial alanlarda yerleşen bilatéral ağrılı hassas nodüllerle karakterizedir. 3-6 hafta sonunda skar bırakmadan spontan iyileşirler. Çoğu zaman klinik görünüm hastalığın tanısında yeterlidir; ancak diğer pannikülitlerden ayırmak için bazen subkutan dokuyu da içine alan derin biyopsilerin yapılması gerekmektedir. Tetikleyici faktörler arasında ilaçlar, enfeksiyonlar, tümörler ve inflamatuvar hastalıklar yer alırken, olguların %32-72'si idyopatiktir. Eritema nodozum etyolojisi bölgelere ve ülkelere göre değişiklik göstermektedir; •

idiyopatik



Behçet Hastalığı



ilaç



Sarkoidoz



ÜSYE



Otoimmun Hastalık



Gebelik



Tüberküloz

89

www.tusem.com.tr

KLİNİK BİLİMLER AÇIKLAMALI CEVAP ANAHTARI 3 Bu testte sırasıyla Dahiliye Grubu, Pediatri, Cerrahi Grubu ve Kadın Hastalıkları ve Doğum Bilgisi soruları ve açıklamaları bulunmaktadır. 1. Aşağıdakilerden hangisi akut romatizmal ateşte mitral kapak tutulumuna bağlı middiyastolik üfürüm olarak adlandırılan özel üfürümün adıdır? A) Graham Steel B) Austin Flint C) Carvello D) Carey Coombs E) Gallaverdin Herhangi bir şikayeti olmayan hipertansiyon tanısı ile izlenen

Cevap D

hastanın EKG'si yukarıdaki gibidir.

ÖZEL ÜFÜRÜMLER

Bu hastada en muhtemel tanı aşağıdakilerden hangisidir?

Graham-Steel üfürümü: Pulmoner HT'a bağlı PY'ye bağlı

A) Sağ dal bloğu

erken diastolik üfürümdür. (Not: MD'ın Graham Steel üfürümü pandiastoliktir)

B) Sol dal bloğu

Austin-Flint üfürümü: AY'de duyulan rölatif MD üfürümü (AY'de

C) Sağ ventrikül hipertrofisi

geri kaçan kan akımının m itral kapağı mid-diastolik ve pre-

D) Sol ventrikül hipertrofisi

sistolik dönemde erken kapatması sonucu oluşur)

E) Wolf Parkinson White Cevap D

Carvello belirtisi: Triküspit yetmezliği üfürümünün inspirasyonda şiddetlenmesidir.

Hipertrofilerde EKG:

Carey-Coombs üfürümü: Akut Romatizmal Ateş'de mitral

Sol ventrikül hipertrofisi: Göğüs derivasyonlarında büyük voltaj

valvülite bağlı kısa mid-diastolik üfürümdür.

(büyük R) dalgaları görülür. Kriterleri şunlardır;

Gallaverdin fenomeni: AD'de üfürümünün apekse yayılması (en

V - g'da: R'ın 25 m m'den büyük olması

yaygın olarak yaşlılarda görülür) MY ile karışır.

Vı _ 2'de: S'in 25 mm'den büyük olması

Rytand üfürümü: AV tam blokta duyalan diyastolik üfürümdür.

Sokolow-Lyon index (V1S + V5R > 35 mm)

5

Dock üfürümü: Sol anterıor desendan koroner arterin darlığına

Cornell voltaj kriterleri (SV3 + RaVL > 28 mm (erkek), SV3 + RaVL

bağlı duyulan diyastolik üfürümdür.

> 20 mm (kadın) Sağ ventrikül hipertrofisi kriterleri; Sağ aks sapması Vl'de R'ın S'den büyük olması ve R dalgasının 5 mm'den büyük olması Vl'de 7 mm'den büyük R dalgası V5 derivasyonunda7 mm'den derin S dalgaları Sol Dal Bloğu • QRS aralığı > 0, 12 s • Vı ve V2'de Geniş S dalgası • Dı, AVL, V 5 ve V6'da Geniş çentikli R dalgası Sağ Dal Bloğu • QRS aralığı > 0, 12 s • Dı, AVL, V5, Vg'da Geniş S dalgası •

T U Sİj TIME Kazavuim r...

V 1( V2 'de çentikli R dalgası

DENEME S IN A V I- 85

4

3.

Aşağıdakilerden hangisi antiaritm ik ilaç

İnce fibröz başlığı bulunur. Fibröz başlık; düz kas hücresi

sınıflandırmasında grup 1 c olarak sınıflandırılır?

ve kollajenden fakirdir.

A) Lidokain

Lipid içeriği %40'dan fazladır.

B) Propafenon

İnflamatuvar hücre (makrofaj, T lenfosit) sayısı çoktur.

C) Amiodaron D) Verapamil E) Kinidin Cevap B 5. Mitral yetmezliği ile ilgili aşağıda verilenlerden hangisi SINIF 1

SODYUM KANAL BLOKAJI (Faz 0)

yanlıştır?

1a: kinidin, dlsoplramld, prokalnamld

A) Sol ventrikülün volüm yükü artar.

1b: lidokain, fenitoin, meksiletin, tokainid

B) Kardiyak debi azalmıştır. C) Pulmoner hipertansiyon gelişebilir.

1c: flekainid, propafenon, morisizin

D) Kronik dönemde atriyal fibrilasyon gelişebilir.

SINIF 2

BETABLOKERLER (Faz 4)

propranolol, metoprolol...

SINIF 3

K KANAL BLOKAJI (Faz 3)

amiodaron, sotalol, bretilyum, ibutilid

SINIF 4

CA KANAL BLOKAJI

verapamil, diltiazem, adenozin

E) En sık semptomu çarpıntıdır.

Cevap E

MY'de hem sol ventrikül hem sol atriyumda öncelikle volüm

Sınıf 1b sadece ventriküler aritmilerde kullanılır.

yükü artar ve sistolik bir yüklenme görülür. MY'de sol atriuma

Supraventriküler etkili değil.

olan kaçak tekrar sol ventriküle volüm yükü olarak geri döndüğü

Sınıf 1a ve sınıf 3 antiaritmikler QT'yi uzatır.

için ventrikülü dilate ederek MY miktarını uzun dönemde daha fazla artırır (MY'nin MY'yi artırması). Uzun dönemde kontraktilitenin bozulmasıyla düşük debi bulguları ortaya çıkar. 4.

Aşağıdaki aterom plak özelliklerinden hangisine sahip olan hastanın miyokard infarktüsü geçirme olasılığı daha yüksektir?

Geç dönemde kalbin gerisindeki yükün artışıyla pulmoner konjesyon ve sağ kalp yetersizliği kliniği eklenebilir. Klinik:

A) Kalın fibröz başlığın bulunması

En sık semptomu yorgunluktur. Çünkü erken dönemde henüz

B) Düz kas hücresinin yoğun olması

pulmoner konjesyon yoktur. Yorgunluk sebebi kardiyak debinin

C) Makrofaj hücre sayısının fazla olması

azalmasıdır. Hastalık ilerledikçe sol ventrikül diastol sonu basınç

D) Lipid içeriğinin %50>den az olması

ve sol atriyal basınç artar, pulmoner konjesyon ve dispne gelişir.

E) T lenfosit sayısının az olması

Akut MY'de ise sol atriyum genişleyemediği için kaçak volüm direkt olarak akciğere ile tilir ve akut pulmoner ödem olarak Cevap C

kendini gösterir. Kronik dönemde atriyal fibrilasyon sıktır. Sol atrium dev boyutlara ulaşabilir. Klasik konjestif kalp yetersizliği

Vulnerable plağa sahip kişilerde miyokard infarktüsü geçirme olasılığı daha yüksektir.

kliniği gelişebilir. Sol atriumun büyümesine bağlı çevre organlara bası arazları görülebilir.

Fibröz (Stabil) Plak •

Makroskopik olarak beyaz renklidir.



Lümene doğru büyür, lümeni daraltır.



Kalın fibröz başlığı bulunur. Fibröz başlık düz kas hücresi ve kollajenden zengindir.



Lipid içeriği %40'dan azdır.



İnflamatuvar hücre (makrofaj, T lenfosit) sayısı azdır.

Vulnerable Plak •

Büyük reaktif bir çekirdek içerir.



Trombojenitesi yüksektir.

TUS'U KAZANDIRAN DOĞRU ADRES K a z a l ı rtr...

6.

Aşağıdakilerden hangisi akciğer tüberkülozu olarak tanımlanır? (Şıklarda verilenlere ek tutulum olmadığı varsayılacaktır) A) Larinks tüberkülozu B) Bronş tutulum u olan tüberküloz C) Hilusta lenf nodu tüberkülozu D) Mediastende lenf nodu tüberkülozu E) Plevra tüberkülozu Cevap B

Akciğer tüberkülozu: Akciğer parankimini ya da trakea bronş ağacını tutan TB için kullanılır. Akciğer parankiminde tutulum yoksa, plevra efüzyonu ya da toraks içinde (hilusta, mediastende) lenf bezi büyümesi ile olan TB, akciğer dışı TB kabul edilir. Akciğer dışı tüberküloz (AD-TB): Akciğer parankimi dışındaki organlardan alınan örneklerde ARB gösterilebilen ya da TB ile uyumlu histolojik ve klinik bulgusu olan hastalar bu gruba girmektedir. Miliyer TB: Akciğer ve akciğer dışı TB olarak kabul edilir. Larinks TB: Akciğer tutulum u yoksa akciğer dışı TB kabul edilir. Plevra TB: Akciğer dışı TB olarak kabul edilir.

7.

Aşağıdakilerden hangisi tip 1 solunum yetersizliğinin ön planda olduğu hastalıklardan biri değildir? A) Pnömoni B) ARDS C) Pulmoner hemoraji D) Perioperatif atelektazi E) Göğüs travması Cevap D

Tip

I Hipoksemik

II Hiperkapnik

III Perioperatif

IV Şok

Mekanizma

Şant

Ventilasyon

Atelektazi

Hipoperfüzyon

Etyoloji

Hava yollarında

Solunum merkezi

FRK

Kardiyojenik

sıvı

Nöromusküler ileti

Kapanma volümü

Ölü boşluk solunumu Klinik

• Pnömoni

• İlaç / Hasar

Örnekler

• Pulmoner ödem

• Miyastenia gravis poliradikülit,

• Kardiyojenik ARDS • Pulmoner hemoraji • Göğüs travması

ALS, botulism, kürar • Astma, KOAH, kifoskolyoz, pulmoner fibrozis

Hipovolemik Septik

• Obesite, asit, peritonit, üst abdomen cerrahisi, anestezi • İleri yaş, sigara, bronkospazm, sekresyon, sıvı yüklenmesi

• İnfarktüs • Kanama • Dehidrasyon • Tamponad • Endotoksemi

8 . Aşağıdakilerden hangisi astım tanısını desteklemekte

İnhale kortikosteroidler

kullanılm az?



Çoğu hasta yan etki yaşamamaktadır.

A) TanısaL süreçte en az bir kez FEV1/FVC < %75-80 olması



Lokal yan etkiler arasında orofaringeal kandidiazis (inha-

B) Pozitif reversibilite testinde PEF'de >%20'lik azalma

lasyonun ardından ağzın çalkalanmasıyla azaltılabilir),

C) 4 haftalık antiinflam atuvar tedavi sonrası PEF'de >%20'lik

disfoni ve öksürük vardır. Beta agonistler

artış D) Pozitif bronş provokasyon testi sonrası FEVİ'de > %20 düşme



Taşikardi, Baş ağrısı, Kramplar, Taşiflaksi

Lökotrien düzenleyiciler

E) Egzersiz provokasyon testi sonrası FEVİ 'de bazaline göre •

>% 10 ve 200 m l düşme

Karaciğer fonksiyon testlerinde yükselme

Antikolinerjikler •

Cevap B

Yaygın değildir; ağız kuruluğu görülebilir.

Teofilin

Değişken Hava Akımı Kısıtlaması •

Tanısal süreçte en az bir kez FEV1/FVC < %75-80 olması



Bulantı, kusma



Pozitif reversibilite testi (Bronkodilatör reversibilitesi)



Yumuşak dışkılama

(Eğer hasta kullanıyorsa kısa etkili beta agonist >4 sa,



QT uzaması (aritmi)

uzun etkili >12 sa kesilmelidir)(200-400 pg salbutamol-



Ototoksisite



Yüksek dozlarda ölüm

den 10-15 dk sonra): FEVİ'de başlangıç değerinde göre >200 ml ve >%12 artış veya PEF'de >%20'lik artış •

İki hafta boyunca günde iki kez yapılan PEF tetkikinde değişkenlik: ortalama günlük PEF değişkenliği >%10



4 haftalık antiinflamatuvar tedavi sonrası solunum yolu enfeksiyonu olmadan FEVİ'de bazal değerine göre >200 ml ve %12 artış veya PEF'de >%20'lik artış







Pozitif egzersiz provokasyon testi: FEVİ ‘de bazaline

10 .

İlaca bağlı lupus için aşağıda verilen bilgilerden hangisi

göre >%10 ve 200 ml düşme

doğrudur?

Klinik kontroller sırasında solunum fonksiyonlarında

A) Anti dsDNA pozitiftir.

değişkenlik: solunum yolu enfeksiyonu olmadan FEV1'de

B) Kadınlarda daha sık görülür.

>200 ml ve > % 12 değişkenlik

C) Kompleman düzeyi düşer.

Pozitif bronş provokasyon testi: Standart metakolin veya

D) Böbrek tutulum u sık görülür.

histamin dozları ile FEV1'de > %20 veya standardize hi-

E) Anti histon antikor pozitiftir.

perventilasyon, hipertonik salin veya mannitol sonrasında >%15 düşme

Cevap E İlaca bağlı lupus: Başta Prokainamid ve Hidralazin olmak üzere çeşitli ilaçlarla tetiklenir. SSS, renal ve hematolojik tutulum görülmez. Anti dsDNA negatif olup hipokomplemantemi görülmez. Her iki cinste eşit görülür.

9.

KOAH'ta kullanılan aşağıdaki ilaçların karşısındaki yan etki yanlış verilmiştir? A) Beta agonistler - taşiflaksi B) Zileuton - karaciğer fonksiyon testlerinde yükselme C) Antikolinerjikler - ağız kuruluğu D) Teofilin - kabızlık E) İnhale kortikosterodler - disfoni ve öksürük Cevap D

11.

Helikobakter pylori enfeksiyonlarına en sık eşlik eden

14. 53 yaşında 3 aydır kilo kaybı ve yutm a güçlüğü saptanan

hastalık aşağıdakilerden hangisidir?

hastada yapılan üst GIS endoskopide orta ösefagusta kitle

A) İmmün trombositopenik purpura

lezyon izleniyor. Yapılan biyopside ösefagus squamoz hücreli

B) Sjögren sendromu

karsinom saptanıyor.

C) Akne rosesea

Hastada aşağıdakilerden hangisinin bu hastadaki malignitenin gelişiminden sorumlu olma olasılığı daha

D) Migren

azdır?

E) Hepatit C Cevap A

A) Sigara B) Alkol

HP enfeksiyonuna en sık eşlik eden hasatlık ITP'dir. Bunun dışında migren, akne rosesea gibi ektragastrik hastalıklar eşlik

C) Fankoni anemisi D) Plummer Winson sendromu E) Schatzki halkası

edebilir.

Cevap E

Schatzki halkası, alt ösefagusta reflüye bağlı fibroz şeklinde kendini gösterir. Özellikle et alımı sonrası disfaji gelişmesi 12.

Aşağıdaki İnflamatuar barsak hastalığı tiplerinden hangisinde probiyotik kullanmanın faydası gösterilmiştir?

tipiktir. Prekanseröz değildir. Fankoni anemisi 2020 Sleisenger'de ösefagus SCC si için risk faktörü olarak belirtilm iştir.

A) Crohn remisyon tedavisi B) Crohn indüksiyon tedavisi C) Ülseratif kolit poşit tedavisi D) Ülseratif kolit remisyon tedavisi 15. Altmış yedi yaşında erkek hasta renal hücreli karsimnom

E) Crohn cerrahisi sonrası nüks önleme

nedeni ile takip edilirken, ateş kilo kaybı nedeniyle Cevap C

başvuruyor. Yapılan kan tetkiklerinde ALP normalin 4 katı yüksek, INR 3. 5, albümin 2. 6 gr/dl, trombosit 10000 mm3,

Probiyotikler son yıllarda barsak hastalıklarında oldukça

nötrofil 700 mm3 ve biluribin normalin 2. 5 katı yüksek

yaygın konuşulan alternatif tedavi stratejileridir. Ülseratif kolitli

saptanıyor. Karaciğer metastazı ön tanısı ile yapılan batın USG

hastalarda poş enfeksiyonu geliştiği zaman etkili oldukları

ve batın BT'de karaciğerde metastatik odak izlenmiyor.

gösterilmiştir.

Bu hastada en olası tanı aşağıdakilerden hangisidir? A) KT'ye bağlı sekonder lenfoma B) Stauffer sendromu C) Kolestaz D) Budd -Chiari sendromu

13.

Kronik kabızlık tedavisinde kullanılan hangi ilaç ince

E) Otoimmün hepatit

bağırsaktaki klorür kanallarını açarak etki eder?

Cevap B

A) Garnisetron

Stauffer sendromu

B) Lubiprostone



C) ALosetron

Karaciğerin fonksiyonlarını kaybettiği renal hücreli karsinomda görülen paraneoplastik sendromlardan bir

D) PrucaLopride

tanesidir.

E) BisacodyL

• Cevap B

Tanı için hepatik metastazın olmadığı gösterilmelidir. Alkalen fosfataz, sedim, GGT ve alfa globulin düzeylerinde artma söz konusudur.

Lubiprostone; en yeni konstipasyon iLaçLarından birisidir. Pg E1 derivesi olup CL kanallarını açarak kronik kabızlık hastalarında tedavide kuLLanıLabiLir.

DENEME S IN A V I- 85 K a z a lı rtr...

16.

ELLi yedi yaşında mide adenokarsinomu ve periton metastazı

18.

Otuz dokuz yaşında bir kadın hasta ani bilinç bozukluğu ile

olan kadın hastaya kemoterapi uygulanmaktadır. Hastanın

acil servise başvuruyor. Hastada fizik muayenede yaygın

kemoterapi rejimi doxetaksel, sisplatin ve 5-FU şeklindedir.

ekimozlar ve skleralarda ikter tablosu izleniyor. Yapılan

Tedavi sırasında ataksi gelişen bu hastada akut serebellar

laboratuvar incelemelerinde serum laktik dehidrogenaz

sendrom saptanmıştır.

düzeyi yüksek, trombosit sayısı 20000 mm3, retikülosit sayımı

Bu hastada klinik tabloya y o l açabilecek aşağıdaki

% 10 ve hemoglobin düzeyi 8 g/dL olarak bulunuyor.

etiyolojilerden hangisi öncelikle düşünülmelidir?

Bu hasta için en olası tanı aşağıdakilerden hangisidir?

A) 5-FU

A) HELLP sendromu

B) Sisplatin tedavisi

B) Yaygın damar içi pıhtılaşması

C)Doxetaksel tedavisi

C) Sepsis

D) SSS metastazı

D) Trombotik trombositopenik purpura

E) Paraneoplastik sendrom

E) Antifosfolipid sendromu Cevap D

Cevap A TTP: Tanı Kriterleri 5-FU en önemli yan etkileri; koroner vasospasm, serebellar



Mikroanjiopatik hem olitik anemi (MAHA)

sendrom (nörolojik), mukozit



Trombositopeni



Nörolojik semptomlar



Böbrek bozukluğu



Ateş



Tanı Kriteri: Trombositopeni ve MAHA (şistozit, yüksek LDH) •

DAT: Negatif ve diğer MAHA nedenleri yoksa TTP/HUS düşünülür.

17.

Aşağıdaki kemik tümörlerinin hangisinin tedavisinde Tirozin Kinaz İnhibitörleri kullanılır?



vWF: metalloproteinaz antikoru



PT ve aPTT: Normal

A) Kordoma B) Kondrosarkoma C) Ewing sarkoma D) Kemiğin dev hücreli tümörü

19.

48 yaşında akut lösemi tanısı alan bir hastada yapılan fizik muayenede diş eti hiperplazisi; kemik iliği incelemesinde ise

E) Osteosarkoma Cevap A Tiroizn kinaz inh. leri; KML ve GIST dışında HCC RCC

atipik eozinofiller saptanıyor. Bu hastadaki sitogenetik bozukluğun aşağıdakilerden hangisi gibi olması beklenir? A) t(4; 11)

Tiroid kanserleri

B) t(9; 22)

Akciğer kanseri

C) inv16

Kolon kanseri Ve kemik tm lerinden kordoma tedavisinde kullanılabilir.

D) deL20q E) t(1; 19) Cevap C

Akut myelositik lösemi sınıflama AML M0: Uniform, undifferansiye, MPO(-) AML M1: Minimal differansiye myeloblastik lösemi: MPO (-/+), matüre olmamış, AML M2: Differansiye myeloblastik lösemi: En sık görülen tiptir. MPO (+++), granüllü, auer rod'lar +++ (en sık M2'de görülür) t(8; 2 1)+

TUS'U KAZANDIRAN DOĞRU ADRES K a z a l ı rtr...

M4: Myelomonositik lösemi: Myeloperoksidaz (++), Nonspesifik

22.

Aşağıdakilerden hangisi adaptif immün sistem

esteraz (+++) (M4-eo: inv 16+)

yetmezliğine bağlı gelişmez?

M5: M5A = A. monoblastik lösemi, M5A= A. Monositik Lösemi:

A) Hiper Ig M sendromu



Nonspesifik esteraz (+++)

B) DOCK 8 eksikLiği



Diş eti hiperplazisi ile gidebilir.

C) Wiskott-ALdrich sendromu



AML M4-M5'de idrar ve serumla lizozom (muramidaz)

D) Yaygın değişken immün yetmezLik

artmıştır. MDS en çok AML M5'e dönüşür.

E) Kronik granuLamatöz hastaLık

M 6 : A. Eritrolösemi PAS (++), diğer boyalar (-)

Cevap E

M7: A. Megakaryoblastik lözemi: Kİ fibrozis olabilir.

Primer İmmün Yetersizlik Sınıflama Doğal immün Sistem Yetersizliği Fagositik hücreler Ciddi konjenitaL nötropeni AspLeni

20.

Aşağıdaki özelliklerden hangisi megaloblastik anemi hastalarında beklenen bir bulgu değildir?

Lökosit adezyon defekti Kronik granuLamatöz hastaLık

A) Alkalen fosfataz yüksekliği



Sinyal trandüksiyon ve reseptör bozukluğu

B) MCV artışı



ToLL benzeri reseptör sinyaLindeki kusurLar

C) Nötrofillerde hiperpigmentasyon



MikobakteriyeL hastaLığa MendeL duyarLığı

D) Peteşi ve purpuralar



Kompleman eksiklikleri

E) Retikülositopeni

• Cevap A

Megaloblastik anemilerde bütün kan hücreleri azalması dışında hemoliz olmasına rağmen retikülositopeni olması önemlidir



KLasik, aLternatif ve Lektin yoLakLarı Litic faz eksikLikLeri

Adaptif İmmün Sistem Yetersizliği •

ALP konuyla ilgisi yoktur. Megaloblastik anemide hemolize bağlı

T lenfosit Bozulmuş gelişim

LDH artabilir.

Ciddi kombine immün yetmezLik DiGeorge Bozulmuş fonksiyon Kombine immün yetmezLikLer Hyper-IgE sendromu (otozomaL dominant)

21.

Aşağıdaki hastalıkların hangisinde periferdeki yıkıma

DOCK8 eksikLiği

bağlı nötropeni görülmesi beklenir?

CD40 Ligand eksikLiği

A) Polianjitisli granülomatoz (Wegener sendromu).

Wiskott-ALdrich sendromu

B) Chediak —Higashi sendromu

Ataksi-teLanjiektazi ve diğer DNA onarım eksikLikLeri

C) Siklik nötropeni



D) Kardiyopulomer bypass cerrahisine bağlı geçici nötropeni E) Klozapin kullanımına bağlı nötropeni

B lenfosit Bozulmuş gelişim X'e bağLı yada OR AgamagLobuLinemi

Cevap A

Bozulmuş fonksiyon Hiper-IgM sendromu

Aslında cevabı içinde saklı zor gözüken basit bir soru. Wegenerdeki antikorlar c-ANCA; antinötrofilik sitoplazmik antikorlar dolayısı ile periferde nötrofil yıkımına bağlı nötropeni izlenebilir.

Yaygın değişken immün yetmezLik (CVID) IgA eksikLiği

Düzenleyici defektler



Bazofillerden CD 63/203 artışı

Doğuştan



C5a artışı ve buna bağlı bazofillerden antikor salınımı-



OtoinfLamatuar



Şiddetli kolit



Hemofagositik lenfohistiyositoz (HLH)

tiroid peroksidaz— hashimatoya y o l açabilir.

ÜRTİKER ANJİOÖDEM -tedavi

Adaptif



H1 antagonistler

Otoimmün lenfoproliferasyon sendromu (ALPS)



Sistemik steroid allerjen indükleyen , idiopatik ve fiziksel ürtiker kaçın.

Otoimmünite ve enflamatuar hastalıklar (IPEX, APECED) •



APECED, autoimmune polyendocrinopathy candidiasis



Topikal steroid?

ectodermal dysplasia;



LT ant.

IPEX, immunodysregulation polyendocrinopathy



Omalizumab

enteropathy.



Herediter anjioödemde akut atakta; bradikinin2 receptor antagonist (Icatibant), kallikrein inh. (Ecallantidej



Herediter anjioödemde (HAE) -Androjenler



HAE-Taze donmuş plasma ve aminokaproik asitle preope hazırlık.

23.

Aşağıdakilerden hangisi ürtiker/anjioödem için yanlıştır? A) KadınLarda daha sık gözLenir. B) PoListemi hastaLarında aquajenik ürtiker gözLenebiLir C) İzoLe anjioödem ACE inh. aLan gençLerde daha sık gözLenir.

24.

Aşağıdaki ilaçlardan hangisi şarbon tedavisinde kullanımı yoktur?

D) Vibratuar ürtiker mesLekseL maruziyet sonrası yıLLar sonra

A) Raxibacumab

ortaya çıkabilir.

B) Kinolon

E) Latekse bağLı kontakt ürtiker gözLenebiLir.

C) Ampisilin

Cevap C

D) Klindamisin E) Sefepim

ACE İNH. / ANJİOÖDEM RİSKİ

Cevap E

Siyahi ırk Organ transpLantLı

ŞARBON TEDAVİSİ

YaşLı



DERI-Penisilin V veya Ampisilin

Sigara içen



Sipro veya Doksisiklin

Kadın



Sistemik tutulum — İv. kinolon +klindamisin/linezolid



Raxibacumab-Obilotoxaximab-toksin reseptör blokeri

İzole-Herediter anjioödem • •

CL inhibitör eksikLiğine bağLı gözLenir. SERPING gen mutasyonu



TİP1 eksik



Tip2 fonk. BozukLuğu



CL yüksek

25.

Aşağıdaki amiloid prekürsör proteinlerin hangisinin birikimi ile oluşan klinik bulgular böbreği daha az tutar?



C2 ve 4 eksik

A) Apolipoprotein AI



Bradikinin artmış.

B) Sistatin C C) Fibrinogen Aa

ÜRTİKER ANJİOÖDEM -LAB •

Ig Eyüksek



KriyogLobuLinemi



KoLinerjik ürtikerde histamin artışı

D) Lökosit kemotaktik faktör 2 E) Gelsolin

Cevap B

TUS'U KAZANDIRAN DOĞRU ADRES K a z a l ı rtr...

28.

AL primer -h e r yer

Önceki sorudaki hastada ilk tercih tedavi aşağıdakilerden

AA sekonder -böbrek, kalp

hangisidir?

Gelsolin -kornea, deri, böbrek

A) iv. steroid

Lökosit kemotaktik faktör-2-renal

B) iv. L-tiroksin

Apo A1 hepatik, renal

C) Sodyum replasmanı

Sistatin C -SSS ve vasküler

D) Dışardan ısıtmaü E) Potasyum iyodür Cevap B

Özellikle miksödem komasında dışardan ısıtma yapılmaması 26.

Kırk beş yaşında erkek hasta poliüri ve polidipsi yakınması

gerekir.

nedeniyle başvuruyor. Hastanın yapılan tetkiklerinde serum

İlk tercih iv, L tiroksindir .

Na: 145 mEq/L ve idrar dansitesi 1005 olarak ölçülüyor.

Sonrasında steroid ve semptomatik tedavi yapılır.

Bu hasta için aşağıdakilerden hangisi yanlıştır? A) Tanıda co-peptin düzeyleri kullanılabilir. B) Susuzluk testi sonra vasopressin yanıtı değerlendirilebilir. C) Demeklosiklin, Lityum, Sisplatin, Metoksifluran gibi ilaçlar

29.

Kırk sekiz yaşında erkek hasta halsizlik, yorgunluk yakınması ile başvuruyor. Öyküsünden altı y ıl önce antitüberküloz tedavi

nefrojenik Dİ yapabilir. D) Tanıda susuzluk testi yapılmalıdır.

aldığı öğreniliyor. Hastanın fizik muayenesinde anormal

E) Nefrojenik Di tedavisinde furosemid etkili bir ilaçtır.

bulgu izlenmiyor. Hastanın laboratuvarında sodyum: 123 mEq/L, potasyum: 5. 6 mEq/L bulunuyor.

Cevap E

Bu hastada en olası tanı aşağıdakilerden hangisidir? A) Cushing sendromu

Nefrojenik DI 'da tuz kısıtlaması ve tiazid grubu diüretikler

B) Primer hiperaldesteronizm

kullanılr.

C) Feokromasitoma D) Addison hastalığı E) Hipotiroidi Cevap D 27.

Otuzdört yaşında erkek hasta koma ile acil servise getiriliyor. Fizik muayenede tansiyon arteryel 90/60 mmHg, nabız hızı

Tbc'un ekstrapulmoner tutulum larından biride adrenal

40/dk ritmik, ateşi 35. 6°C olarak bulunuyor. Ölçülen serum

yetersizliktir.

sodyumu 124 mEq/L olarak bulunuyor.

Hipnatremi ve hiperkalemi de özellikle kortizol ve aldesteron

Bu hastada en olası tanı aşağıdakilerden hangisidir?

eksikliğini destekler. Tanı için ACTH uyarı testi yapılır.

A) Septik şok, hipotermik şekli B) A-V tam blok C) Miksödem koması D) Addison krizi 30.

E) Water Hause Frederikson sendromu Cevap C

Aşağıdakilerden hangisi bikabonatlı dializat sıvısında bulunmaz? A) Albümin

Hipotermi ve hiponatremi akla hemen miksödem komasını getirir

B) Kalsiyum

ki bradikardi de bunu destekler.

C) Magnezyum D) Glukoz E) Potasyum Cevap A

DENEME S IN A V I- 85 K a z a lı rtr...

33. Otuz sekiz yaşında kadın hasta ayda 4 kez olan, aniden

STANDART DİALİZ SIVISINDA OLMAYANLAR Üre

başlayan, tek taraflı, zonklayıcı, şiddetli başağrısı tarifliyor.

Kreatinin

Ağrı sırasında bulantı kusma mevcut ve sesten aşırı rahatsız olduğunu belirtiyor. Ağrı öncesinde herhangi bir görme

Fosfor

şikayetinin olmadığını söylüyor.

Albümin

Bu hastada akut tedavide en etkili ilaç aşağıdakilerden

Vitamin B12 bulunmaz

hangisidir? A) İndometazin 31.

Otuz yaşında erkek hasta geçirdiği trafik kazası sonucu yapılan refleks muayenesinde Babinksi(+)ve patellar refleksi

B) İbuprofen C) Zolmitriptan

hiperaktif olarak saptanıyor.

D) Propranolol

Lezyonun en olası yeri aşağdakilerden hangisidir?

E) Verapamil Cevap C

A) İkinci motor nöron B) Bazal ganglionlar C) Piramidal traktus

Soruda en sık migren formu olan aurasız migren sorulmuş. Akut

D) Serebellar hemisfer

atakta en etkii ilaç grubu triptanlardır.

E) Ekstrapiramidal sistem

Migren tedavisi: Migren tedavisi temel olarak, akut atak tedavisi Cevap C

ve profilaksi olmak üzere iki komponentten oluşur. Akut atak tedavisi (TUS):

Patolojik refleksler ve Derin tendon refleks artışları; üst motor

Basit analjezikler; aspirin, parasetamol, naproksen, ibubrofen. Sık alındığında gastrik semptomlara ve rebound ağrıya neden

nöron (piramidal traktus) hasarı sonucu görülür.

olurlar. Ergo alkaloidleri (Ergotamin-cafein, dihidroergotamin): Belirgin bulantı ve kusma yaptıklarından tedaviye antiemetik eklenmelidir. 5-HT (serotonin) agonistleri (triptanlar) (Sumatriptan, 32.

Bu hastanın kafa travması sonrası akut bilinç kaybı veya fokal norolojik defisiti de bulunmaktadır. İlk istenilmesi gereken görüntüleme yöntemi aşağıdakilerden hangisidir?

Zolmitriptan): Gebelik, koroner hastalık, periferik vaskuler hastalıkta kontrendikedir. Narkotik analjezikler (Kodein, meperidin, butarfanol)

A) Direk kafa grafisi B) BT

Profilaktik Tedavi:

C) Dopler USG

Trisiklik Antidepresanlar (Amitriptilin)

D) MR Beta blokörler (Propranolol gibi) (ilk tercih) E) SPECT Antikonvulzanlar (Fenitoin, valproat) Cevap B

Kalsiyum kanal blokörleri (Verapamil) Siproheptadin

Eğer nörolojik bulgu olmasa direk kafa grafisi istenmesi gerekiyordu. Ancak fokal nörolojik belirtileri ve bilinç bozukluğu olanlarda CT ilk başvurulacak görüntüleme yöntemidir.

Metiserjid, Pizotifen (Metiserjid retroperitoneal fibrozise neden olur) Profilakside kullanılan metizerjid ve pizotifen akut atak tedavisinde kullanılırsa ağrıyı artırırlar. Akut atakta kontrendikedir. Gebelik sırasındaki migren sadece opioid ajanlarla (meperidin) tedavi edilir.

34. ELLi iki yaşında bilinen hipertansiyon öyküsü olan erkek

SOMATOFORM BOZUKLUKLAR

hastanın düzensiz irbesartan kullanımı mevcut. Sabah 9: 30

Somatoform bedensel görünümlü anlamına gelen bir sözcüktür

civarında ani gelişen sol taraflı kuvvetsizlik gelişen hastanın

ve bu başlık altında bir takım bedensel belirtilerin ön planda

acil kraniyel bilgisayarlı tomografi incelemesinde 16 cm3 sağ

olduğu psikiyatrik hastalıklar sınıflandırılmaktadır. Ancak bu

talam ik hematom izleniyor.

bedensel belirtilere y o l açan herhangi bir organik etken yoktur.

Kan basıncı 190/125 mm-Hg olarak ölçülen normal sinüs

Somatizasyon Bozukluğu Tanı: 4 yerde ağrı: en az 4 ayrı ağrı

ritmindeki hasta için kan basıncı hedefinim en uygun

(sırt, karın, eklem, ekstremite, göğüs, dizüri, ) 2 GIS bulgusu (ağrı

düzenleyen basamak aşağıdakilerden hangisidir?

dışında ör; bulantı, kusma, şişkinlik, ishal gibi) 1 cinsel semptom

A) Oral doksazosin ile %10 oranında düşürülmesi

(libido kaybı, ereksion veya ejekulasyon bozukluğu, aşırı menstral

B) Oral amlodipin ile %25 oranında düşürülmesi

kanama gibi) 1 pseudonörotik semptom (baş dönmesi, sağırlık,

C) İntravenöz esmolol ile 160/90 mm-Hg sınırına acil olarak

disfaji, ses kısıtlılığı, çift görme, güç kaybı gibi) Bu semptomlar kasıtlı olarak çıkarılmaz veya varmış gibi davranılmaz.

düşürülmesi D) İntravenöz esmolol ile 140/90 mm-Hg sınırına acil olarak düşürülmesi E) İntravenöz nitroprussid ile 120/85 mm-Hg sınırına acil

36. Gebelikte majör depresyonun en uygun tedavisi aşağıdakilerden hangisidir?

olarak düşürülmesi

A) Trisiklik antidepresanlar Cevap D

B) Antipsikotikler C) Elektrokonvülsif terapi

Son akut hemorajik inme kılavuzuna göre parenkimal

D) Paroksetin

intrakraniyel hematomlarda kan basıncı ivedilik ile intravenöz

E) Monoamin oksidaz inhibitörleri

bir anti-hipertansif ile 140/90 mm*Hg sınırına indirilm elidir.

Cevap C

Başka ajan yokluğunda nitroprussid, kafa içi basıncını artırabilme riskine karşın, kullanılabilmektedir.

37. Aşağıdakilerden hangisi saçlı, kıllı bölgelerde eritem ve squamasyonla giden hastalıktır? A) Akne vulgaris

35. Kırk yaşındaki kadın hastada, yılla rd ır süren somatik

B) Seboreik dermatit

yakınm alar mevcut. Ayrıntılı incelemeler yapılmış ve

C) Akne Rosea

birçok uzman doktor tarafından tedavi edilmiş. Ancak,

D) Pitriazis Rosea

dalgalanmalar halinde seyreden halsizliği ve somatik sıkıntısı

E) Hidroadenitis süpürativa

geçmemiş. Şu anki yakınmaları şöyle sıralanabilir; düzensiz ve Cevap B

ağnlı adetler, sırt ağrısı, göğüs ağnsı, sersemlik hissi, bulantı, pirozis, dizüri, güçsüzlük atakları ve bacaklarda seyirme. Fizik muayene ve laboratuvar testlerinde hiçbir anorm allik yok.

Pitriazis Rosea papüllü skuamlı hastalıklar grubundandır. Deri eklerinin hastalıkları; Akne vulgaris: Pilosabese biriminin

Bu hastada en olası tanınız nedir?

inflamasyonudur.

A) Munchausen sendromu

Seboreik dermatit: Saçlı, kıllı bölgeleri eritem ve squamasyonla

B) Vücut dismorfik bozukluğu

giden hastalıklarıdır.

C) Hasta takliti yapma

Akne Rosea: Yüzde eritem ve telenjiektaziler üstünde papul ve

D) Hipokondriazis

pustul gelişimi ile karakterize bir hastalıktır. E) Somatizasyon bozukluğu Hidroadenitis süpürativa: Aksilla ve anogenital bölgedeki apokrin glandlarm enfeksiyonunu tanımlar; en sık etken Cevap E

Bu soruda Munchausen sendromu ile somatizasyon bozukluğu arasındaki farka dikkat edelim.

stafilokoklardır.

bir yapıya sahiptir. Gerçek deridir, deri ekleri olan kıl follükülü,

38. Aşağıdakilerden hangisi epidermiste bulunmaz? A) MeLanosit

ter, yağ bezleri gibi yapıları, sinir ve kan damarlarını içerir.

B) Langerhans hücresi

Stratum papillare: Elastik lifler, papillalar ve kan damarları, sinir uçları içerir. Bazal membrana doğru papiller çıkıntılar yapar.

C) KıL foLiküLü

Dermisdeki papillaların epidermis içine doğru aşırı uzaması

D) MerkeL hücresi

papillomatozdur. E) Desmozom Stratum reticulare: Fibroelastik bantları nedeniyle deriye Cevap C

sağlamlık sağlar. SUBKUTAN DOKU (PANNİKULUS) yağ dokusundan oluşur.

EPİDERMİS: St Corneum, St. Lucidium, St. GranuLosum, St. spinosum, St BasaLe tabakalarından oluşur. St. Korneum: Dudakta olmayıp, avuç içinde ve ayak tabanında kalın olan bu tabaka, ölü çok sayıda yassı keretositLerin Lameller

39.

Aşağıdakilerden hangisi “Belirli bir bölgede belirli bir zaman dilimi içerisinde gelişen vaka sayısının risk

şeklinde oluşturduğu en üst tabakadır. Bu öLü ve keratinize oLmuş

altındaki kişi sayısına oranı" olarak tanımlanır?

hücrelerin zamanLa desquame oLurLar.

A) insidans

Bu tabaka hücreLerinin döküLmeyip kaLınLaşması Keratoz

B) PreveLans

(Hiperkeratoz) (TUS), granüLer tabakada çekirdeklerini kaybetmeyerek bu tabakada çekirdekLi hücreLerin buLunmasına

C) RöLatif risk

da Parakeratoz denir. ŞekiLde hiperkeratoz ve parakeratoz

D) AtfediLen risk

izLenmektedir.

E) FataLite hızı Cevap A

St. Lucidum: Sadece avuç içinde ve ayak tabanında buLunan tabakadır ve eLeidin maddesinden zengin oLması nedeniyLe şeffaftır.

MORBİDİTE (hastalık) DÜZEYİNİ BELİRLEYEN ÖLÇÜTLER

St. Granülosum: Spinosumdan geLen hücreLer bu tabakada

1.

İnsidans: SağLam kişiLerin beLirLi bir hastaLığa beLLi bir

stopLazmaLarını ve nükLeusLarını (canLıLıkLarını) kaybederek

süre içinde (hafta, ay, yıL) yakaLanma ihtimaLidir. O haLde

küçüLür yassıLaşır ve iğ şekLini aLırLar. Bu iğ şekLini aLan

insidans hesabında bir süreç (zaman diLimi) beLirtiLmeLidir

hücreLerin içLerinde bazofiLik keratohyaLin granüLLeri geLişir,

(1 yıLLık insidans, 6 ayLık insidans gibi). BeLLi bir zaman

tabaka ismini bu granüLLerden aLır.

diLiminde yeni vaka sayısı l duyarLı (sağLam) topLum

St. Spinosum: BazaL tabakadan köken aLan 6-7 sıra

nüfusu

desmosomLarLa birbirLerine bağLı poLigonaL hücreLerden oLuşur.

2. Prevalans: topLumda bir hastaLığın o anki görüLme

Bu tabakada ayrıca antijenLeri tanıyan ve T LenfositLere sunan

sıktığıdır. HastaLığın başLangıç tarihi biLinmez, yani bir

Langerhans h. Leri buLunur.

süreç yoktur. BeLLi bir zaman kesitinde eski ve yeni tüm

Spinozum hücreLerinin kaLınLaşmasına Akantoz (psöriazis

vaka sayısı l topLum nüfusu

ve nörodermatitde görüLür). Spinozum hücreLeri arasındaki desmosomLarın parçaLanması iLe hücreLerin birbirLerinden ayrıLmasına AkantoLiz (pemfigusta görüLür).

40.

Retinitis pigmentozanın yeni geliştirilen tedavisinde

Spinozum hücreLeri arasında ödem var ise Spongioz (egzema da

hedeflenen gen aşağıdakilerden hangisidir?

görüLür) oLuşur.

A) RPE65

St. Basale: Tek sıraLı siLindirik keratinositLerin aLtındaki bazaL membrana hemidesmosomLarLa bağLanmasıyLa oLuşan epidermisin en aLt tabakasıdır. Bu tabakadaki hücreLerin % 5'i meLanositLerdir ve uLtravioLeye karşı koruyucu meLanin sentezini yaparLar, ayrıca dokunma reseptörü oLarak biLinen merkeL hücresi bu tabakadadır.

B) CDH23 C) USH1C D) SMN1 E) FMR1 Cevap A

BazaL tabaka, epidermisi dermise bağLar. BazaL tabakanın en önemLi fonksiyonu m itotik faaLiyet göstererek diğer

Retinitis pigmentozanın tedaviLeri en az SMA tedavisi kadar

epidermistabakaLarını oLuşturan hücreLeri üretmesidir ve bu

günceL ve önemLi bir konu. Şu an RPE65, CEP290, ABCA4 genLeri

nedenLe germinativum tabakası adını da aLır.

için tedaviLer FDA tarafından onay aLdıLar (EyLüL 2020 itibariyLe)

DERMİS: Derinin esas böLümünü oLuşturur ve sağLam ve eLastik

TUS'U KAZANDIRAN DOĞRU ADRES K a z a l ı rtr...

41. Aşağıdaki has hastalıklarından hangisi otozomal resesif

42.

Aşağıdakilerden hangisi paraplejik hastalarda

kalıtım paterni gösterir?

gelişebilecek komplikasyonlardan biri değildir?

A) Duchenne muskuler distrofi

A) Kaslarda atrofi, spastisite ve koordinasyon bozukluğu meydana gelir

B) Becker muskuler distrofi C) Myotonik distrofi

B) Kemiklerde osteoporoz, heterotopik ossifikasyon gelişebilir

D) Limb-Girdle muskuler distrofi

C) Ciltte; dekübit ülserleri, cilt enfeksiyonları gelişir

E) Facioscapulohumeral muskuler distrofi

D) Mesane-barsaklarda; retansiyon, inkontinans, iştahsızlık, kabızlık, üriner enfeksiyon ve taşlarmeydana gelir.

Cevap D

E) Bilişsel algıda azalma gözlenir Cevap E

MUSKULER DİSTROFİLER Kas liflerinin histopatoljik tekrarlayıcı—yıkım ve rejenerasyon süreçleri sonucunda gelişen endomizyalyağ ve bağ dokusu artışı ile karakterize hastalık grubudur.

Paraplejik hastalarda •

meydana gelir

Duchenne muskuler distrofi Duchenne muskuler distrofi en sık görülen müsküler distrofi

Kaslarda atrofi, spastisite ve koordinasyon bozukluğu



Kemiklerde osteoporoz, heterotopik ossifikasyon gelişebilir

tipidir. X'e bağlı resesif geçişi vardır. Erkeklerde görülür. Güçsüzlük 5 yaşından önce başlar. Çocuk oturur vaziyetteyken



Ciltte; dekübit ülserleri, cilt enfeksiyonları gelişir

ayağa kalkmak için kendi üzerinde tırm anır (Govvers arazı). Alt



Mesane-barsaklarda; retansiyon, inkontinans, iştahsızlık,

extremite distallerinde pseudohipertrofi vardır. Dejenere olan kas liflerinin yerini fibroz doku ve yağ dokusu alır. 9-12 yaşlarında

kabızlık, üriner enfeksiyon ve taşlarmeydana gelir. •

yatağa bağımlı hale gelir. 16yaştan sonra hemen tüm hastalarda kardiyomyopati gelişir. 25 li yaşlarda solunum yetmezliğinden

ven trombozları, ortostatik hipotansiyon •

dolayı ölürler. Patolojide distrofin denilen sarkolemmal protein hiçyoktur. Hikaye, kreatinin fosfokinaz (bu enzim muskuler distrofilerde 15-200 kat artmıştır), EMG anorm alliği tanıda yardımcıdır. Kesin tanı mutant distrofin geninin gösterilmesiyle konur. Prednizon progresyonuyavaşlatır. Becker muskuler distrofi Duchenne göre daha hafiftir. Distrofin vardır. Ama defektiftir. X'e bağlı geçer. 10yaşından sonra başlar. 30-35yaşlarına kadar ayakta kalabilir. Limb-Girdle distrofi Limb-Girdle OR geçişli kas hastalığıdır. Yalnızca proksimal kasları tutar. Pelvis kuşağı kaslardan başlar. Myotonik distrofi Otozomal dominant geçişli, trinukleotid tekrar sendromlarındandır. Kasılan kas gevşeyemez (aksiyon myotonisi) Facioscapulohumeral muskuler distrofi omuz kuşağını etkileyen muskuler distrofidir. OD kalıtılır.

Kardiovasküler sistemde, kalp rezervinde azalma, derin

Solunum sisteminde, vital kapasitede azalma, özellikle pulmoner enfeksiyonlar



Santral sinir sisteminde, parasteziler, duyu azalması, ağrı eşiğinde azalma, motor fonksiyonda azalma, emosyonel bozukluklar, otonom disreflexisi meydana gelir.

KLİNİK BİLİMLER AÇIKLAMALI CEVAP ANAHTARI 3

Bu testte sırasıyla Dahiliye Grubu, Pediatri, Cerrahi Grubu ve Kadın Hastalıkları ve Doğum Bilgisi soruları ve açıklam aları bulunmaktadır. 1.

Aşağıdakilerden h angisi koroner arter h astalığı olan

K ro n ik k a lp ye tm e zliği tedavisinde:

hastalara y a p ıla c a k önerilerden biri değildir?

"İvabradin kronik kalp yetersizliği tedavisinde, ejeksiyon

A)

Hipotansiyondan kaçınılması

fraksiyonu % 4 5 'in altında olan ve

B)

Hipertansiyondan kaçınılması

C)

DM hastalarında çok sıkı kan şekeri kontrolü yapılm ası (H b A lc 5 cm H 2 O olması E)

H ava yo lu düz kasları hipertrofi ve hiperplaziye uğrar

-

Kollabs veya nodül ile açıklanam ayan bilateral opasite

Subepitelyal bazal m em bran kalın laşır (lam ina retikülarisin kalın laşm ası vardır, lam ina rara ve lam ina densa değişm ez-lam inin ve tip IV kollajen

Cevap D

dağılım ı değişmez). -

H ava yo lu duvarı kalınlaşır.

-

H ava yo lu duvarında m atriks birikir.

yeni veya kötüleşen solunum

-

M ikro dam arlarda ve sinir ağların d a artış olur.

sıkıntısı

-

Subm ukozal bezlerde artış olur (mukus, serum

A R D S Tanısında Berlin Kriterleri Z am an lam a

1 hafta içinde ortaya çıkan

Akciğer Görüntülemesi

Ödem kaynağı

Efüzyon, kollabs veya nodül ile

proteinleri, inflam atuvar hücreler ve hücre

açıklanm ayan bilateral opasite

artıklarından (nükleik asit, eozinofil granül

So lu n u m sıkıntısının kalp

proteinleri)

yetmezliği veya hipervolemiye

-

Hafif

yardım cı hücre tip 2 (Th^)

objektif ölçütlere gösterilm esi

sonucudur)

• 200 m m H gValproik asit Depresyonu önlemede lam otrijin>Lityum =Valproik asit Lamotrijin depresyonun ön

B)

C)

Karbamazepin: Etkinliği lityum yada valproik asit

D)



Subakut dönemde direk grafide benekli osteoporoz görülebilir.

kadar değildir. Agranülositoz, glokom krizi, sedasyon ve antikolinerjik ya n etkileri vardır.

Subakut dönemde ağrı şiddetlenir, ödem daha yaygınlaşır.

Bipolar bozuklukta kullanılan diğer duygudurum dengeleyiciler: •

Erken dönemde sintigrafi ve direk grafide bulgu saptanmaz.

planda olduğu bipolar bozukluklu hastalarda tercih edilir ve bir sonraki depresif atağı geciktirir (öteler).

Erken dönemde lokalize yanıcı ağrı, hiperestezi, lokalize ödem görülür.

E)

Kronik dönemde fleksor tendon kontraktürleri gelişir.

Valproik asit: Li ile yakın etkinliktedir. Lityum kullanam ayan hastalarda tercih edilir. Hepatotoksiktir.

Cevap B

Teratojendir. 1. Dönem: Ortalam a 3 ay sürer. •

Lokalize yanıcı ağrı, Hiperestezi, Lokalize ödem, Kas spazmı, Başlangıçta sıcak ve kuru deri, daha sonra siyanoze soğ u k ve ıslak deri görülür. *Direkt grafi normal, bu dönemde kemik sintigrafisi istenir.

2. Dönem: 3-6 ay arası Kan akımı azalmış, cilt soğuk, Hiperhidroz Ağrı şiddetlenir, ödem daha yaygın

Kıllarda seyrelme tırnaklarda kırılganlık Benekli osteoporoz (Direk grafide) Artm ış eklem kısıtlılığı B.

Dönem

Şiddeti azalm ış ağrı, Kas atrofisi belirginleşir, Fleksor tendon kontraktürleri gelişir. Tanıda direk grafi ve kemik sintigrafisi kullanılır.

39.

A şağıdakilerden hangisi v itiligo tarifine uygun elementer lezyondur? A)

M akü l

B)

Papül

C)

Patch

D)

Plak

E)

Purpura Cevap A

Derinin elementer lezyonları: Deri ile aynı seviyede olanlar, M akül: Deri ile aynı seviyede, 1 cm 'den az çaplı, çevresindeki deriden sadece renk bakım ından farklı nonpalpabl lezyon: hipohiperpigmente vs. Patch: 1 cm üstündeki m aküler lezyon Deriden kabarık olanlar; Papül: Çapı 1 cm 'den küçük sert solid lezyonlardır. Nodül: Çapı 1 cm üzerindeki sert solid lezyonlardır. Vertikal seyreder. Plak: Birleşm iş papüler lezyonlardır. İçerisinde sıvı birikimi olanlar: Vezikül: Çapı 0, 5 cm altındaki intra veya subepiderm al su toplanmasıdır. Bül: Çapı 0, 5 cm üzerindeki intra veya subepiderm al su toplanmasıdır. Püstül: Püy içeren su toplanmasıdır. Vezikül veya bül olabilir.

40.

A ltm ış beş y a şın d a erkek hastada, s o l gözünde pitozis, m idriazis ve sa ğ taraflı hemipleji gelişiyor. Yapılan görüntülem elerle serebrovasküler olay tanısı konulan h astad a a şa ğ ıd a verilen arterlerden hangisinin d alla rın d a oklüzyon ge lişm iş o lm ası en olasıdır? A)

A.cerebri media

B)

A.cerebri posterior

C)

A.carotis interna

D)

A.com m unicans anterior

E)

A.cerebri anterior Cevap B

Yukarıda tarif edilen vakada 3. kranial sinir lezyonuna ait bulguların olm ası ve hemipleji gelişm esi bize W EB ER sendrom unu düşündürür. Bu sendrom da tutulan arter A.cerebri posteriordur. Aynı şekilde 3.sinir tutulum u+kontralateral ekstremitede tremor görülm esi ise B EN ED İC T sendrom unu ve yine A.cerebri posteriorun tutulduğunu bize gösterir.

41. A şağıdakilerden hangisi Psöriazis tedavisinde ku lla n ılm a z ? A)

Deksametazon

B)

infliximab

C)

Etanercept

D)

Secukinum ab

E)

Metotreksat Cevap A

Psöriazis tedavisinde topikal steroidler kullanılırken sistem ik steroidler tercih edilmez. A. Topikal 1.

Nemlendirici ve keratolitikler Asit salisilik asit.

2.

Antralin (cynyolin), antimitatik etkilidir

3.

Steroidler

4.

Kalsipotriol (pusarkütan) ^

D3 vitamini. Epiderm al proliferasyon ve diferansiasyonunu düzenler.

B. Fototerapi PUVA (psöralen ve ultraviole A); antijen salan Langerhans hücreleri ve natürel killer hücreler üzerinde im m ünosupresif etki gösterir. Gebe, emziren kadınlarda, fotosensitif kişilerde, SLE, porfirililerde verilmez. Katarakt ve deri kanserleri özellikle skuam oz hücreli karsinom, fotoyaşlanm a bulantı, kusm a ya n etkileridir. C. Sistem ik Tedavi 1.

Metotrexat - Antimitotik etkilidir. Kemik iliği ve karaciğer'e toksiktir.

2.

Sentetik retinoikler (Vit A deriveleri) - Epitel proliferasyonu ve diferansiasyonunu düzenler.

3.

Etretinat

4.

Asitretin (Neotigasan)

5.

Siklosporin

42. Bir gü n lü k erkek bebeğin yenidoğan ünitesindeki fizik muayenesinde, idrarının glan s penis yerine penisin ventral yüzündeki daha proksim al bir delikten geldiği tespit ediliyor. Ayrıca peniste ventrale doğru bir eğimin olduğu ve prepusyum un sadece penis dorsalinde yerleşik olduğu, ventralde bulunm adığı gözleniyor. Bu bebek için en o lası tanı aşağıdakilerden hangisidir? A) Epispadias B) Vezikoüreteral reflü C) Kriptorşidizm D) Kısa üretra E) Hipospadias Cevap E

Kolay bir eski Tus sorusu. Penisin ventral yüzeyine açılm asına hipospadias denir. Fizik muayene ile tanı konulabilir.

T U S 'İüİ T' IKazakulınr... ME

88. DENEME SINAVI AÇIKLAMALI CEVAP KİTAPÇIĞI OCAK 2021

K LİN İK B İL İM L E R CEVAP ANAH TARI

T E M E L B İL İM L E R CEVAP A N AH TARI 1-

B

31-

A

61-

B

91-

C

1-

C

31-

D

61-

B

91-

E

2-

D

32-

C

62-

E

92-

A

2-

C

32-

B

62-

B

92-

E

3-

D

33-

C

63-

B

93-

B

3-

C

33-

B

63-

C

93-

C

4-

E

34-

B

64-

A

94-

D

4-

B

34-

C

64-

B

94-

A

5-

B

35-

A

65-

B

95-

D

5-

B

35-

A

65-

A

95-

B

6-

B

36-

C

66-

E

96-

B

6-

D

36-

D

66-

E

96-

B

7-

E

37-

D

67-

A

97-

A

7-

C

37-

C

67-

B

97-

C

8-

C

38-

C

68-

A

98-

B

8-

E

38-

B

68-

A

98-

D

9-

B

39-

D

69-

C

99-

E

9-

C

39-

B

69-

E

99-

C

10-

C

40-

E

70-

E

100-

E

10-

D

40-

D

70-

C

100-

B

11-

D

4 1-

A

71-

B

101-

C

11-

D

41-

B

71-

A

101-

E

12-

E

42-

B

72-

A

102-

C

12-

B

42-

E

72-

E

102-

E

13-

A

43-

B

73-

E

103-

B

13-

A

43-

B

73-

D

103-

D

14-

A

44-

D

7 4-

E

104-

C

14-

A

44-

D

74-

B

104-

D

15-

E

45-

C

75-

A

105-

E

15-

C

45-

A

75-

E

105-

D

16-

D

46-

D

76-

A

106-

C

16-

E

46-

E

76-

C

106-

C

17-

A

4 7-

E

77-

C

107-

B

17-

D

47-

E

77-

A

107-

B

18-

D

48-

A

78-

E

108-

C

18-

E

48-

A

78-

B

108-

E

19-

C

49-

E

79-

B

109-

C

19-

A

49-

D

79-

C

109-

D

20-

D

50-

D

80-

D

110-

B

20-

A

50-

C

80-

E

110-

B

21-

A

51-

E

8 1-

A

111-

B

21-

B

51-

E

81-

D

111-

B

22-

C

52-

E

82-

E

112-

A

22-

E

52-

E

82-

E

112-

C

23-

D

53-

B

83-

E

113-

D

23-

A

53-

D

83-

D

113-

A

24-

D

54-

A

84-

E

114-

B

24-

C

54-

C

84-

E

114-

A

25-

D

55-

D

85-

B

115-

B

25-

A

55-

D

85-

D

115-

A

26-

A

56-

B

86-

D

116-

C

26-

E

56-

C

86-

C

116-

D

27-

C

57-

C

8 7-

A

117-

D

27-

A

57-

C

87-

D

117-

B

28-

C

58-

D

88-

E

118-

B

28-

C

58-

D

88-

E

118-

E

29-

B

59-

E

89-

D

119-

A

29-

C

59-

C

89-

E

119-

D

30-

A

60-

A

90-

E

120-

A

30-

A

60-

C

90-

B

120-

A

İSTANBUL-MERKEZ

PENDİK

KAYSERİ

A k sa ra y M ah . C e rra h p a şa Cad

Fevzi Ç a k m a k M ah. Ç ın a r S o k a k

Y enid oğan M a h a lle si Ç im enli Cad

No: 59

No: 2 D: 9

D e m a B B lo k No: 35/E

H aseki-Fatih / İS T A N B U L

Pendik / İS T A N B U L

Talas / KA YSER İ

KADIKÖY

ANKARA BEŞEVLER

RİZE

İb ra h im a ğ a Zaviye Sok. Kat 1

M e re şa l Fevzi Ç a k m a k Caddesi

İsla m p a şa M a h a lle si Şe h itle r Cad.

B a ğım sız B ö lü m No:8

No:4 D: 3

No:35, 53020

K o şu yo lu - Kadıköy / İS T A N B U L

B e şe vle r / A N K A R A

M erkez / RİZE

ANKARA

MANİSA

KARABÜK

M a m a k Cad. Dikim evi P osta h a n e si Yanı

U ncub ozköy M ah . 5 5 0 4 Sok. 13/A/25

Yenişehir M ah . Z ü m rü t Sk. K u le P a rk B

D ik im e v i-M a m a k / A N K A R A

M A N İS A

B lo k No:1 K A R A B Ü K

İZMİR

BALÇOVA

DÜZCE

C u m h uriye t Bulvarı No: 99/A Kat: 2

Poyraz Sok.N o: 4/A B a lço va / İZ M İR

O rh a n ga zi M ah. K o n u ra lp 435. Sok. Tekin Apt. No: 14 Z em in Kat

M. Rıza İş Merkezi (A n a d o lu b a n k Üstü)

BORNOVA

P asa p o rt / İZ M İR

Kazım Dirik M ah. 185 Sk. N:2/A B o rn o v a / İZ M İR

ISPARTA

M erkez / D Ü ZC E

SİVAS Yenişehir K a rd e şle r Cad. T h e M o o n

Gazi K e m al M ah. 1317 Sok.

YEŞİLYURT

H ende n Ap. No: 11 D: 9/10

9 0 4 9 S o k a k Vatan M a h a lle si

M erkez / ISP A R T A

K a ra b a ğ la r / İZ M İR

AYDIN

KIRIKKALE

H asanefend i M ah . Kızılay Cad. No:34

Yenişehir M ah. 263. Sok.

Sitesi D B lo k 2/ D:10 M erkez / S İV A S

SAKARYA G ü llü k M ah. O rh a n ga zi Cad. Belde Sok.

M erkez / A Y D IN

No:16 Ad apazarı / S A K A R Y A

D o ğ a n Apt. Kat:1

TOKAT

Y a h şih a n / K IR IK K A L E

Se m e rka n t M ah. G a zio sm a n p a şa Bulvarı

GAZİANTEP

10.

Yeditepe M a h a lle si 85061 N olu S o k a k

ESKİŞEHİR

No: 7/1 Şa h in b e y / G A Z İA N T E P

B üyükd ere M a h a lle si A daklı Sok.

S o k a k No:23 TO K AT

BALIKESİR

No: 9/A

K a sa p la r m ah 11003 so k a k 14/2 Altıe ylü l

O d u npa za rı / E S K İŞ E H İR

/ B A L IK E S İR

KONYA / SELÇUK

SAKARYA

B o s n a H e rse k M ah. S o n e r Sk. 3 C

G ü llü k M ah. O rh a n ga zi Cad. Belde Sok.

S e lçu k lu / KO N Y A

No:14/1 Ad apazarı / S A K A R Y A

M e vh ib e Erkek Ö ğrenci Yurdu Altı

KONYA / MERAM

VAN

Battalgazi / M A L A T Y A

B e yşe h ir Cad. No: 220

Van Y ü zü n cü yıl Üniversitesi D u rsu n

M e ra m / KO N Y A

O dabaşı Tıp M erkezi Karşısı,

MUĞLA Kötekli M a h a lle si 274 S o k a k No: 12/5 M enteşe / M U Ğ L A

MALATYA B u lg u rlu M ah. İnönü Üniversitesi

M erkez / V A N

www.tustime.com

0850 840 7834

KLİNİK BİLİMLER AÇIKLAMALI CEVAP ANAHTARI 3 Bu testte sırasıyla Dahiliye Grubu, Pediatri, Cerrahi Grubu ve Kadın Hastalıkları ve Doğum Bilgisi soruları ve açıklam aları bulunmaktadır. 1.

A şa ğıd a k i fizik m uayenelerden h angisi triküspit darlığı ile

Soru zorlaştırılm ası uzun uzadıya yazıldı. Bilinmesi gereeken

m itral darlığını birbirinden ayırt etmede ku llan ılır?

dilate kardiyomiyopati başlığının genel bir başlık olduğudur.

A)

Hill belirtisi

Geçirilmiş viral enfeksiyon, kalp yetmezliği bulguları, kalp

B)

Carvello belirtisi

büyümesi....Viral miyokardit.

C)

K u ssm all belirtisi

koroner anjiyografinin norm al olm ası koroner arter hastalığını

D)

Landolfi bulgusu

ekarte eder.

E)

Dans des arter bulgusu

FM'de üfürüm olm am ası kapak hastalıklarını ekarte eder. Sonuç Cevap C

Soruyu okuyunca ilk akla gelen carvello belirtisini işaretleyen

viral miyokardite bağlı dilate kardiyomiyopati.

3.

Aşağıdakilerden h angisi kalp yetm ezliğinde sekonder gelişen durum lardan biri değildir?

çoktur. Fakat soruları dikkatli okum ak çok ya ra r sağlar. Carvello

A)

Koroner kan akımın azalm ası

bulgusu bilinmesede olabilir. Soruyu zorlaştırm ak için şıkka

B)

Sistem ik konjesyonun olması

konulm uştur.

C)

Pulm oner kanlanm anın azalm ası

H ill belirtisi: Alt ekstremite kan basıncının üst ekstremite kan

D)

Kalp duvarlarında hipertrofi olması

basıncından 40 m m H g ve daha fazla olması

E)

Oksijen gereksinim inin artması

triküspit yetm ezliğinde görülür. Landolfi ve dans des arter

Carvello belirtisi: inspiryum la sağ kalp üfürüm lerinin artmasıdır.

Cevap C

M itral yetmezliği ve triküspit yetm ezliğinin ayırımında kullanılır. K u ssm a ll belirtisi: inspiryum la beraber boyun venöz

Önce ufak bir düşünm e, sonra çoğu kursiyerimizin yapabileceği

dolgu n lu ğu n u n artm ası (paradoks olarak) (TD'de görülür)

soru... Kalp yetm ezliğinde boşluk boyutları artabilir, duvar

Landolfi bulgusu: Her sistolde pupillada kasılm a (Ciddi AY'de

kalınları artabilir. Debi azalm asına bağlı olarak koroner

görülür)

kanlanm a ve periferik kanlanm a bozulur. Oksijen gereksinimi

Dans des arter bulgusu: Karotis arter pulsasyonlarının görülm esi

artar. Pulm oner yatakta ise göllenm e olur, konjesyon m eydana

(Ciddi AY'de görülür)

gelir, kanlanm a azalmaz.

2.

42 yaşın d a erkek hasta artan nefes darlığı şikayeti ile başvuruyor. Geceleri nefes darlığı nedeniyle üç yastıkla

4.

Aşağıdakilerden hangisi iskem ik kalp hastalıklarının

yatabildiğini, bacaklarında şişlik olduğu nu söylüyor. 1 ay

sekonder ko m plikasyo n ların dan biridir?

önce koroner anjiyografi yapılm ış ve norm al bulunmuş.

A)

Ani ölüm

B)

Miyokard rüptürü

C)

Transm ural infarkt

D)

Asem ptom atik kalp hastalığı

E)

A n gina pektoris

Birkaç hafta önce grip benzeri hastalık geçirmiş. Herhangi bir ilaç kullanm ıyorm uş. Tansiyonu 110/70, nabız 110/dk, juguler venöz basınç 14 m m Hg, her iki akciğer bazallerinde rall, dinlemekle S3 ve S4 var idi. Ek ses ve üfürüm saptanmadı. Hafif karaciğer büyüm esi vardı. Ekstremiteler soğ u k idi.

Cevap B

EKG norm al sinüs ritminde idi. Akciğer grafisinde bilateral m inim al plevral efüzyon ve kardiyom egalisi vardı.

Kulağı tersten tutan bir soru...

Bu hastad a sem ptom ların en o la sı sebebi aşağıdakilerden

İskemik kalp hastalıklarının primer kom plikasyonları;

hangisidir?



Asem ptom atik iskemi

A)

Ciddi mitral darlık



Stabil angina pektoris

B)

Ciddi aort darlığı



U nstabil angina pektoris

C)

İdiyopatik dilate kardiyomegali



Akut miyokard infarktüsü

D)

Koroner arter hastalığı



Ani ölüm

E)

Ciddi mitral yetmezliği Cevap C

D EN E M E S IN A V I- 88 Kazandırır...

5.

A şa ğıd a k i h astalardan h angisi için byp ass am eliyatı daha

8.

45 ya şın d a hasta acil servise akut akciğer ödemi ile geliyor.

uygundur ve yararlıdır? (A şa ğıd a verilenler hariç diğer

İlk değerlendirm e sonrası hasta m ekanik ventilasyona

tüm özellikler aynı kab u l edilm elidir)

bağlanıyor. Hasta 800 cc tidal volüm, 16/dk hızla yardım cı

Diyabeti olan + EF >50 + RCA ciddi darlık + CX ciddi darlık

kontrol ventilasyona bağlanıyor. Yarım saat sonra alınan kan

B)

Diyabeti olan + EF 50 + RCA ciddi darlık + CX ciddi darlık

A)

D)

Hastanın aşırı ajite olduğu görülüyor. Bu h astada respiratuvar alkalozun kontrolü için hangisi

Diyabeti olm ayan + EF >50 + LAD ciddi darlık + RCA

ya p ılm a lıd ır?

ciddi darlık E)

Diyabeti olmayan + EF 200 pg/ml (genellikle ektopiktir)



> 500 pg/ml (= Ektopik)

CD19, CD20 ve CD22 pozitif, CD5, CD10 ve CD23 negatifdirler.

2. Yüksek doz DXM supresyon testi (2 gün 8 mg veya tek doz 8 mg)

Splenik m arjinal zon lenfom a (SM ZL)

3. gün sabah (08-09°°) Kortizol % 5 0 den fazla baskılanm a % 8 5 CH

1.



HCV İLİŞKİSİ



7q32 delesyonu



Ayırıcı tanı



KLL -C D 20 d üşük



3. IP S S (İnferior petrozal sinüs örneklemesi) 0.dk (ACTH için kan alınır) ardından 100 |ig C RH -İV yapılır.

POZİTİF TEDAVİ 1.SPLENEKTOM İ



2.SP LE N İK RT





2., 5., ve 10.dk'larda ACTH ölçülür



Santral/Perifer ACTH oranı

titre CD 23 pozitif

Mantle hücreli lenfoma -C D 5, CD 43 VE SİK L İN D1



ve 2. gün: 6 saatte bir 4 tbl Dekort verilir.

3.Klorombusil, Fludarabin, Rituksimab

0.

saat 2 altı CRH sonrası 3 altı ise ektopik

0.

saat 2 üzeri CRH sonrası 3 üzeri ise CH

4.Periferik CRH testi: 0. dk (ACTH ve kortizol için kan alınır) ardından 100 |ig i.v. CRH yapılır. 15., 30. ve 60. dakikalarda kan alınarak ACTH ve kortizol düzeylerine bakılır. Bazale göre ACTH > % 3 0 - 50, kortizol > % 2 0 artış (+yanıtlı) CH % 9 0 (+ yanıtlı) 5. Hipofiz M R

TUS'U KA ZAND IRAN DOĞRU ADRES

23. A şa ğıd a k i ifadelerden h angisi im m ün sistem için y a n lıştır? A)

B)

C)

A N A F İLA K Sİ

Antijen için T hücre reseptörleri (TCR ), CD 7 üzerinden



M ast hc, eozinofil, bazofiL patolojide önemli.

sinyale y o l açar.



Histam in(ilk yükselen), proteaz, LT, PAF, Pgler

Folliküler T helper lenfositler, lenf nodunda germ inal



merkezde B lenfosit olgu n la şm a sına katkıda bulunurlar.



Histamin flushing, ürtiker, kaşıntı,hipotansiyon

Th 1 lenfositler transkripsiyon faktör beta ile uyarılıp IF



LT ve P g D2 -bronkokonstriksiyon



Pg D2 kutanöz flushing



PAF ciddiyetle korele.



C3 a ve C 5a yüksek.

gam a üretirler. D)

Th 2 lenfositler STAT6 ve GATA3 transkripsiyon faktörleri ile uyarılıp, IL-13 üretip antikor ve eozinofil yanıtını

Triptaz 60-9 0 dk artar 5 saat yü k se k kalır tanıda

düzenlerler. E)

Regülatuar T lenfositler FOXp3 transkripsiyon faktörü ile uyarılıp, zararlı im m ün yanıtı önlerler.

25.

77 yaşın d a kadın hasta 2 yıld an beri evre 0 kronik lenfositer lösemi tanısı ile tedavisiz izleniyor. Hasta ani gelişen halsizlik

Cevap A

ve yo rg u n lu k yakınm aları ile acil servise başvuruyor. Yapılan fizik muayenede sarılık, lenfadenopati ve dalağın

Sınavın immünoloji sorusu ile T lenfositler ve özelliklerine

büyük olduğu saptanıyor. Laboratuvar incelemelerinde

bakm akta fayda var.

hemoglobin: 7.4 g/dl, ortalam a eritrosit hacmi (MCV): 104 fL,



TCR; CD 3 üzerinden etki gösterir.

lökosit: 80.000/m m 3, m utlak lenfosit: 68 000/m m 3, trombosit:



Foll. T helperlar; düzenleyici -bcl-6;IL-4 ve IL-21 üretir.

175 000/m m 3, retikülosit % 6 bulunuyor.





Lenf nodunda y a da dalakta germ inal merkezde B

Bu h asta için en olası öntanı ve tam koyabilm ek için

lenfosit olgu n la şm a sına katkıda bulunurlar.

y a p ılm a sı gereken tetkik aşağıdakilerden hangisinde

Th1 ler; Transkripsiyon faktör düzenleyicisidir. IFN)-

verilm iştir?

gam m a, IL-2, TN F-beta salgılayarak hücresel yanıtı

A)

Otoim m ün hemolitik anemi —

düzenler.

B)

KLL progresyonu-------- Kemik iliği incelemesi

Th 2 ler;Düzenleyici STAT-6, GATA 3. IL-4, IL-5, IL-6, IL-9,

C)

Pernisiyöz a n e m i

D)

Hipersplenizm —

E)

Richter tra n sfo rm a sy o n u

IL-10, IL-13 salgılar, antikor ve eozinofil yanıtı düzenler. •

T reg; CD4 ve CD 8 lerin Fox p 3 ile düzenlenm esi ile

Direkt Coom bs testi

Folat ve B12 tayini Portal basınç ölçüm ü Lenf nodu biyopsisi

oluşur ve zararlı im m ün yanıtı önlerler. •

Cevap A

Th9 T hücreleri; Transkripsiyon faktörü PU.1 tarafından düzenlenen CD4 T hücreleri. IL-9 salgılayan ve atopik hastalıkta enflam asyonu artırırlar ve inflam atuar barsak hastalığının yanı sıra, antitüm ör imm ünitesine aracılık

Soruda hem retikülositoz, hem sarılık hem olitik anemi için

ederler. •

KLL, lösem iler içerisinde otoim m ün hem olitik anemi ve trombositopeni en sık yapandır.

önem li işarettir. Otoimm ün hem olitik anem iler için Direkt

Th17 T hücreleri; RORyt transkripsiyon faktörü tarafından düzenlenen CD4 T hücreleri. IL-17, IL-22 ve IL-

coom bs testi yapılm alıdır.

26 salgılayan ve otoim m ünde rol oynarlar ve bakteriyel ve fu n gal patojenlere karşı im m ün yanıt olu şturu rlar. 26. 24. Aşağıdakilerden hangisinin düzeyi a n afilak si hastalarının

sayısı normaL sınırLarda buLunuyor.

ciddiyeti ile ilgilidir? A)

Histamin

B)

Triptaz

C)

Platelet aktive edici faktör -P A F

D)

Pg D2

E)

T N F alfa

Hemorajik diatez nedeniyLe başvuran bir hastada protrombin zamanı, aktive parsiyeL trombopLastin zam anı ve trombosit

Bu hastad a öncelikle düşünülm esi gereken aşağıdakilerden hangisidir?

Cevap C

Son sınavda anafiLaksi tedavisi im. adrenaLin sorguLandı.

A)

von WiLLebrand hastaLığı tip III

B)

Faktör VII eksikLiği

C)

Faktör VIII inhibitörü varLığı

D)

Faktör IX inhibitörü varLığı

E)

Faktör XIII eksikLiği Cevap E

AnafiLaksi aciL hastaLar içinde önemLi konuLardan biridir.

D EN E M E S IN A V I- SS Kaztmdınr...

Von W illebrand T ip i ve Faktör 13 eksikliği tüm testLer normaL

29.

A k u t bakteriyel menenjit am pirik tedavisi için

oLup kanam a bozukLuğu yapabiLir. Tanıda faktör düzeyLeri

aşağıdakilerden h angisi yan lıştır?

bakıLmaLıdır.

A)

YenidoğanLarda ampisiLin+sefotaksim veriLebiLir.

B)

immünitesi sağLam 40 y hastaya Seftriakson veriLebiLir.

C)

55 y üzeri erişkinde tek başına seftriakson yeterLidir.

27.

Böbrek naklinde rejeksiyon açısından aşağıdakilerden hangisi y ü k se k riskli bir durum değildir? A)

CanLı donör

B)

Genç aLıcı yaşı

C)

Donör spesifik antikor vaLığı

D)

PaneL reaktif antikor pozitifLiği(PRA)

E)

Gecikmiş greft fonksiyonu

D)

Hastane kökenLi hastada ampisiLin +seftaz,d,m veriLebiLir.

E)

Nöroşirürjik operasyon sonrası Meropenem +vankom isin veriLebiLir. Cevap C

Cevap A

A K U T B A K T ER İYEL M EN E N JİT -A M P İR İK TEDAVİ Endikasyon

Antibiyotik

Yüksek Risk

Preterm ve 1 aya kadar yeni

AmpisiLin + sefotaksim

• H LA - M İSM A T C H NEGATİF

• H LA M M sayısının artması

doğanlarda

• Canlı donör

• Genç alıcı yaşı

1-3 ay arası yenidoğanlarda

AmpisiLin + sefotaksim ya da

• Beyaz ırk

• Siyah ırk İmmünitesi - sağlam 3 aydan

Sefotaksim, seftriakson y a da

büyük yenidoğan - 55 ya ş

sefepim + vankom isin

Düşük Risk

• Düşük PRA

• Yüksek PRA

• D SA un olm am ası

• D SA varlığı

seftriaksiyon

• Kan grubu uyum u

• Kan grubu uyum suzluğu

• Kısa soğ u k iskemi süresi

• Uzun soğuk iskemi süresi

• İlk Nakil

• Retransplant

arası yetişkinlerde 55 yaş üzeri yetişkinlerde ve

• Gecikmiş greft fonksiyonu

28. Kırk aLtı yaşın d a erkek hasta ya n ağrısı yakınm ası iLe

AmpisiLin + sefotaksim,

alkoliklerde ve ek hastalığı

seftriakson ya da sefepim +

olanlarda

vankom isin

Hastane kökenli menenjit,

AmpisiLin + Seftazidim y a da

travm a sonrası veya

meropenem + vankom isin

nöroşirurji operasyonu

başvuruyor. YapıLan USG'de her 2 böbrekte muLtıpL taş iLe

sonrası menejit, nötropenik

renaL medüLLer ve intrapapiLLer topLayıcı kanaLLarda 1-8

hastalar veya hücresel im m ün

m m arasında değişen muLtipLe küreseL ovaL kistLer ve Lineer

yetersizliği olanlarda

bantLar saptanıyor. Bu h astada en olası tanı aşağıdakilerden hangisidir?

30.

Klorokin dirençli P. falciparu m sıtm asın da aşağıdakilerden

A)

MedüLLer kistik hastaLık

h angisi ku llan ılabilir?

B)

OtozomaL resesif poLikistik böbrek hastaLığı

A)

Atovaquan proguaniL

C)

MedüLLer sü nge r böbrek

B)

KLorokin +mefl.okin

D)

HipokaLemik kistik hastaLık

C)

KLorokin +prim akin

E)

TüberozskLeroz

D)

Primakin

E)

Kinin

Cevap C M E D Ü L L E R SÜ N G E R BÖ BR EK •

RenaL medüLLer ve intrapapiLLer topLayıcı kanaLLarda 1-B m m arasında değişen muLtipLe küreseL ve ovaL kist

Cevap A S IT M A TEDAVİSİ •

mevcuttur. •

Tanı IVP iLe konur - kontrast m addenin genişLemiş

lumefantrine •

topLayıcı kanaLLarda birikimi - küreseL kistLer ve Lineer bantLar şekLinde görünüm e neden oLur. •

oLarak adLandırıLır.

TekrarLayan FaLsiforum -Artem ether/lum efantrine + Prim akin y a d a Kinin



Lineer bantLar papiLLaya doğru karakteristik bir görünüm oLuşturur - çiçek demeti y a da boya fırçası gö rünüm ü

KompLikasyonsuz FaLsiforum - Artemether I

Birinci trimesterdeki gebeLerde kom pLikasyonsuz P. faLciparum enfeksiyonLarı oral kinin ve klindam isin



Şiddetli ise ilk tercih iv.Artesunate;



Profilaksi -prim akin

TUS'U KA ZAND IRAN DOĞRU ADRES K a z a r u iır ır . . .





D İR EN Ç Lİ f a l s i f o r u m VEYA FALCİFORUM DIŞI

V E R T İK A L BAK IŞ BO ZUKLU KLARI

S IT M A TEDAVİ; Atovaquane-proguaniL

A şa ğıya Deviasyon

H İPN O ZO İT E EN ETKİN P R İM A K İN ***



Çocuklarda (batan güneş m anzarası) hidrosefalide görülür.

31.

Bilateral M e d ia l Longitidunal Fasikülüs (M LF) lezyonu saptan an 25 y a şın d a kadın h astada en olası ha stalık

Parinaud Sendromu •

aşağıdakilerden hangisidir?

Akuaduktus silvi ve tektal bölge Lezyonlarında görülür. (mesensefalon)

A)

Locked-in sendrom u



VertikaL bakış parezisi

B)

Fisherin bir buçuk sendrom u



EtiyoLoji: Aquaduktus stenozu, pinealom

C)

MiLLer-fisher sendrom u

D)

MuLtipLe skleroz

E)

Perinaud sendrom u

32. Transtentorial herniasyonda ilk etkilenen kran iyal sinir aşağıdakilerden hangisidir? A)

N. opticus

B)

N. oculom otorius

H ORIZO NTAL O K Ü LER H AREK ET BO ZUKLU KLARI

C)

N. hypoglossus

Tonik Okuler Deviasyon

D)

N. trigem inus

FrontaL Lob 8. aLan LezyonLarında

E)

N. Glossofaringeus

Cevap D



GözLer Lezyon tarafına deviye oLur



KontraLateraL hemiparezi

Bakış felci— Pons (PPRF) lezyonlarında

Cevap B

H ER N İA SY O N SE N D R O M L A R I Subfalxian Herniasyon



Lezyon tarafına bakamaz. Lezyonun karşı tarafına



KontrLateraL hemiparezi

Sin gu lat gyrus karşıya geçer Arterior



İnternükLeer OftaLmopLeji (INO)

Serebrel Arter basısı nedeniyle kontralateral hemipleji-parezi,

deviasyon



Parietal Lob lezyonlarının beynin falx serebri altına sıkıştırmasıdır.

Lezyon tarafında adduksiyon kısıtLıLığı+abdüksiyonda

inkontinans m eydana gelir.

nistagm us

Tonsiller Herniasyon



M L F Lezyonu

Posterior fossada yerleşen bir kitle nedeniyle serebellar



EtiyoLoji: GençLerde muLtipLe skLeroz, yaşLıLarda

tonsillerin foram en m agn um 'd an aşağı herniye olarak m edulla'ya

serebrovasküLer oLay Bilateral Internükleer Oftalmopleji (BİNO)

bası uygulam asıdır. M e d u lla kom presyonu ani bilinç, solunum kaybına y o l açabilir. En sık cerebellar tüm örlerde olur.



İnternükLeer oftaLmopLeji biLateraLdir

Transtentorial herniasyon



Çift tarafLı M L F Lezyonu

1. U ncal herniasyon (lateral transtentorial fıtıklanm a)



EtiyoLoji: MuLtipLe skLeroz için patognom onik

Fisher'in Bir Buçuk Sendromu • •

M L F +Po n s (PPRF) birlikte tutuLur. Bakış feLci ve internükLeer oftaLmopLeji (Lezyon tarafındaki göz ortada sabit karşı taraftaki göz içe bakamıyor)

Kilitlenme (Locked in) •

VentraL Ponsta BiLateraL P P R F +piramidaL yoLLar etkiLenir.



BiLateraL horizontaL bakış feLci +quadriparezi



BiLinç açıktır, vertikaL göz hareketLeri korunmuştur.



EtiyoLoji: SerebrovasküLer oLay

B İN O (bilateral internükleer oftalmopleji) m ultip skleroz için patognom onik bulgudur.

Klinikte en sık karşılaşılan herniasyondur. Supratentorial bir kitle nedeniyle tem poral Lob unkus b ölüm ünün tentorium yanından aşağı herniye olmasıdır. İlk olarak bu bölgede posterior serebral arter ve superior serebellar arter arasında seyreden 3. kranial sinir etkiLenir. İpsilateral midriazis gelişir. Lezyonun karşı tarafında piram idal irritasyon bulguları ve hemiparezi gelişir. Daha sonra A R A S etkiLenir ve bilinç bozukluğu ortaya çıkar. Basının devam etmesi Duret kanam alarına y o l açar. İlerleyen dönemde beyim sapı karşı tentoryum kenarında sıkışır ve lezyonla aynı tarafta hemiparezi yapar. (Kernohan belirtisi) 2. Aksiel (santral): 2 taraflı unkal transtentorial Herniasyondur. İki yanlı yerleşm iş kronik subdural hematom gibi lezyonlarda oluşabilir.

D EN E M E S IN A V I- SS Kaz& ndm r...



Hemiasyon. Diensafalon ve mezensefalon tentoryum açıklığından

Toksik: Alkol, Lityum, C-Dopa, Organofosfatlar, Steroidler, Karbonmonoksit, Ergotamin,

aşağı doğru fıtıklaşm asıyla oluşur. İki yanlı myotik pupil, yukarı bakış kısıtlılığı, cheyne-stokes

Diğer: cerrahi, MI travm a kanama.

solunum u klinik özellikleridir.

Tedavi: Delirium bir psikiyatrik acildir. M utlaka altta yatan neden bulunup ortadan kaldırılmalıdır. Ajitasyonda düşük doz haloperidol verilir. Uykuyu temin etmek

33. Delirium hakkınd a a şağıd ak i ifadelerden hangisi ya n lıştır?

için gereğinde kısa etkili benzodiazepinler kullanılabilir.

Bilinç bozukluğu mevcuttur

Prognoz: Hızlı seyirlidir ve altta yatan neden ortadan kaldırılırsa

B)

Kronik bir tablodur

düzelir. Altta yatan neden düzeltilmezse ölüm veya dem ansla

C)

Ajitasyon görülebilir

D)

H alüsinasyonlar sıktır

E)

Dikkat dağınıklığı vardır

A)

sonuçlanabilir.

34.

Ç ocu ksuluğun ve non sistem atik sanrıların ön planda olduğu, erken y a ş ta başlayan, işlevsellikte bozulm a ile

Cevap B

karakterize şizofreni tipi aşağıdakilerden hangisidir? O R G A N İK M ENTAL BO ZUKLU KLA R

A)

Paranoid şizofreni

B)

Katatonik şizofreni

C)

Hebefrenik (dezorganize) şizofreni

D)

Rezidüel şizofreni

E)

Ayrışm am ış şizofreni

DELİRYUM: Beyin işlevlerinin bedendeki herhangi bir patolojiden etkilenmesi ile ortaya çıkar. Konfüzyon ve bilinç düzeyi bozukluğu ile karakterize akut

Cevap C

başlangıçlı ve genellikle geri d önüşüm lü bir bozukluktur. Kognitif (bilişsel) yetilerde bozulm a eşlik eder (bellek, dikkat ve konsantrasyon, oryantasyon,konuşma).

Şizofreninin klinik alt tipleri:

Em osyonel dalgalanm a, halusinasyonlar ve m antıksız dürtüsel

1- Paranoid: Daha geç yaşta başlar. En sık görülen tiptir.

şiddet içeren davranışlar olabilir.Zaman, m ekan ve kişi



Düşünce içeriği bozukluğu hakimdir.

oryantasyonu genelde bozulmuştur. Dikkat genelde dağınıktır.



Sa n rıla r ve işitsel h alüsinasyonlar tabloya hakimdir.

H alusinasyonlar sıktır ve tipik olarak canlı görsel halusinasyonlar



görülür. Yargılama, bellek ve soyut düşünm e bozulmuştur. Sirkadien ritmde ciddi aksam alar gö rü lü r ve hastaların uyku düzenini sa ğla m a k zor olabilir. Tanı: 1.

Bilinç düzeyinde dalgalanm alar.

2.

Dem ans tablosu ve daha önce olan algı bozukluğu olm aksızın algıda (halusinasyonlar ve iluzyonlar) ve

Persekütif, büyüklük, kıskançlık, ku şkuculuk-alınganlık sanrıları vardır.

2 - Heberfrenik (dezorganize): Daha erken yaşta başlar. •

Kişilik dağılm ası ve yıkım hızlıdır.



Dezorganize konuşm a ve davranış, K ü nt-donuk yada uygunsuz affekt temel özelliğidir.



Çocuksu davranışlar, ekolali, ekopraksi, görülür.



Prognozu en kötü, tedaviye en az yanıt veren tiptir.

bilişsel işlevlerde (bellek, dikkat ve konsantrasyon, dil, oryantasyon) bozukluk. 3.

Akut başlangıç.

4.

Deliryum u destekleyen laboratuvar bulguları. (elektrolit

3 - Katatonik: Şu belirtilerinin biri veya birkaçının tabloya hakim olduğu tiptir. M otor hareketsizlik (katatonik postür-rijidite (katılık), stopor,

imbalansı, enfeksiyon göstergeleri, görüntülem e yöntem leri vs. ) Deliryum nedenleri •

Beyinle ilgili nedenler: Serebrovasküler olaylar, İntrakranial enfeksiyonlar, Travmalar, Tümörleri, Epilepsi, Y oksunluk sendromları.



Metabolik: Karaciğer, böbrek yetmezliği, Sıvı-elektrolit bozuklukları, Vitamin eksiklikleri, Hipertansiyon, Hipo-hiperglisem i, Adrenal patolojiler, Akut pankreatit, Hipertermi.

balm um u katılığı - hastanın bedeni istenilen postüre getirilebilir, bırakıldığında da bir süre bu duruş korunur. Negativizm (hareket ettirmeye yönelik tüm çabalara direnç gösterir, yemez, içmez, uyumaz, uyarılara tepkisizdir. Negativizm veya konuşm az (mutizm). Mutizm en sık bulgudur. Hareket bozuklukları ön plandadır. İşitsel h alüsinasyonlar tipik belirtiler arasında ye r almaz. EKT'ye en iyi yanıt veren tiptir.

4 - A yrışm am ış (undiferantieted): Birden fazla tipin özelliğini

farktır. Range de ortalam a gibi uç değerlerden çok etkilenir.

taşır, ancak hiçbir belirti baskın değildir. (paranoid, dezorganize

S tandart sap m a (Standard Déviation: SD): Her bir verinin

ya da katotonik tipe uymaz) 2. En sık görülen tiptir.

aritmetik ortalam aya olan uzaklığını. yani dağılım ının yaygınlığını

5 - Rezidüel: Bir y a da birkaç atak sonrası, (+) belirtilerin

gösterir.Dağılım eğrisi ne kadar basıksa standart sapm a o kadar

kaybolduğu rem isyon dönemi ve (-) bulguların (künt duygulanım,

fazla. dağılım eğrisi ne kadar sivri ise standart sapm a o kadar

sosyal çekilme gibi) tabloya hakim olduğu tiptir.

azdır. Yani S D büyüdükçe dağılım yaygınlaşır. Varyans: Standart sapm anın karesidir.

35.

Tanım layıcı istatistikte, deneklerin aldıkları değerlerin

Standart hata (Standard error: SE): Örnek ortalam asının evrenin

(verilerin) toplam ının denek sa yısın a bölünm esi ile

ortalam asından ne kadar farklı olduğunu gösterir. Denek sayısı

aşağıdakilerden h angisi hesaplanır?

arttıkça azalır.

A)

Mean

B)

Median

C)

Mod

biridir?

D)

Range

A)

Su ların klorlanm ası

E)

Standart sapm a

B)

Aşı yapm ak

C)

Mem e kanseri taraması

D)

Rehabilitasyon hizmetleri

E)

Kolon ca taraması

36. Aşağıdakilerden hangisi üçüncül korum a stratejilerinden

Cevap A

TAN IM LA YIC I İSTATİSTİKLER

Cevap D

Elde edilen verilerin sınıflandırarak (Frekans dağılımları. ortalam a standart sapm a gibi) sunulm asıdır.

KORUYUCU H İZM ET LER

Tanımlayıcı ölçütler;

Bir hastalık etkeni vücuda girm eden önce hastalığa ait hiçbir

1. Yer gösteren ölçüler

belirti yoktur. Herhangi bir patolojik değişiklik gelişmemiştir.

a) merkezi ölçütler b) çeyrek ve yüzdelikler 2. Yaygınlık ölçüleri a) standart sapm a

Duyarlılık evresi adı verilen bu dönemde verilen koruyucu hizmetlere Birincil Korum a hizmeti denir. Brincil (primer) koruma; •

Su ların klorlanm ası ve bağışıklam a gibi. belirli hastalıklara karşı alınan önlemlerdir.

b) varyans c) varyasyon katsayısı d) standart hata M ER K E Zİ ÖLÇÜTLER



Birincil korum anın amacı belirli hastalıkların ortaya çıkışını (insidansını) önlemektir.

Birincil korum a önlemleri toplum ların sağlık düzeyini yükseltm ede en etkili olan faliyetlerdir.

Aritm etik ortalam a (mean): Deneklerin aldıkları değerlerin (verilerin) toplamının denek sayısına bölünmesidir. Simetrik sayısal dağılım larda tercih edilir. Aşırı uç değerlerden etkileneceğinden aşırı uç değerler varsa tercih edilmez. Ortanca (median): Dağılım ın orta noktasındaki değerdir. Dağılımdaki değerlerin % 5 0 si ortancaya eşit ve/ veya daha küçük. % 5 0 's i ortancaya eşit ve/veya daha büyüktür. Aşırı uç değerlerden etkilenmeyeceğinden. aşırı uç değerler varsa ya da sim etrik olm ayan dağılım larda tercih edilir.

H astalığa ait belirtilrin çıkmadığı. ancak etken nedeniyle hastalık oluşturacak patolojik değişikliklerin olduğu ve presemptomatik evre adı verilen dönemdeki korum aya ise ikincil korum a denir ve erken tanı hizmeti ile ikincil korum a sağlanır. EKG-mamografi. pap smear. Üçüncül döneme ise klinik evre denir. Hastalık aşikar haldedir. Bu dönemde hasta ölüm den korunm aya veya tam olarak tedavi edilmeye çalışılır. Ayrıca sakatlıklar veya kom plikasyonların önüne geçilmeye çalışılır. Bu dönemde verilen koruyucu

Sıra ile dizilen verilerin sayısı tek ise. en ortasındaki. Çift ise en ortasındaki iki değerin ortalamasıdır.

hizmetler yani hastaların tedavisi. rehabilitasyonu ve sakatlıkların önlenmesi üçüncül koruyucu hizmetlerdir.

Tepe değeri (mod): Dağılım da en sık bulunan (tekrarlayan) veridir. YAYG IN LIK G Ö STEREN ÖLÇÜTLER Farklı grupların merkezi ölçütleri aynı olduğu halde. gruplar birbirlerinden çok farklı olabilirler. Dağılım aralığı (range): En büyük ve en küçük değer arasındaki

D EN E M E S IN A V I- 88 Kaz&ndLm r...

37. Bir taram a testi çalışm a kapsam ındaki 500 kişiden 50'sini

38.

A şağıdakilerden h angisi atopik derm atitin major

hasta olarak değerlendirmiştir. İleri incelemeler sonucunda

bu lgu ların dan biri değildir?

bu 50 kişinin 10 tanesinin gerçekte hasta olm adığı tespit

A)

Kaşıntılı olm ası

edilmiştir. Tarama testinin sağlam olarak değerlendirdiği 450

B)

Akut olması

C)

Ailesel veya kişisel atopi öyküsü

kişinin 20 tanesinin de hasta olduğu tespit edilmiştir. Bu taram a testi ile ilgili o larak aşağıdakilerden hangisi

D)

doğrudur?

Küçük çocuklarda ekstremitelerin ekstansor yüzlerine yerleşim göstermesi

I. Pozitif prediktif değeri 40/50 dir.

E)

II. Negatif prediktif değeri 20/450 dir.

A d olesanlarda ekstremitelerin fleksor yüzlerinde yerleşim göstermesi

III. Duyarlılığı 20/60 dır.

Cevap B

IV. Seçiciliği 430/440 dır.

- Atopik Dermatit

A)

I, II

Vakaların % 6 0 'ı 1 ya ş altında başlar. Hastada ve ailesinde atopi

B)

I, III

öyküsü (IgE artışı ve Alerjik hastalığa eğilim), astım, Alerjik rinit

C)

I, IV

olabilir. Antigenin IgE ile etkileşimi sonucu duyarlanm ış olan

D)

II, III

mast hücreleri ve bazofillerden vazoaktif m addelerin salınm ası ile

III, IV

gerçekleşen tip 1 Alerjik reaksiyondur.

E)

Cevap C

Etyoloji: Genetik predispozisyon

Referans test ile hasta tanısı alanlara. yeni yöntem ile hasta tanısı

Besinler: Yumurta, süt, fasülye, balık, unlu m am üller en sık etken

koyma oranı SEN SİTİVİT E-du yarlılıkdir. Yani testin duyarlılığı

İnhalanlar: Aeroallerjenler, toz akarları

toplumdaki gerçek hastaları ortaya çıkarm a özelliğidir.

S.aureus, streptokok, herpes enfeksiyonları Deri kuruluğu

Yeni testin saptadığı h astalar / Gerçek hastalar (referans teste

Hormonal: Hamilelik, m ensturasyon, tiroid hastalıkları

göre)

Mevsim: Genellikle hastalık yazın iyileşir, kışın alevlenir.

Referans test de sağlam çıkanlara. yeni yöntem ile sağlam

Yün giysiler

tanısı koym a oranı SP ESİT İV İT E-se çicilik (özgüllük) dir. Yani

Em osyonel stres

testin seçiciliği ise toplumdaki gerçek sağlam ları ortaya çıkarma özelliğidir.

Lezyon; Deri kuruluğu ve kaşıntıyla giden döküntülü iyi sınırlı olm ayan eritamatöz yamalardır. Bebeklerde; yan a kla rda (ağız çevresi sağlamdır), saçlı deri ve

Yeni testin saptadığı sa ğ la m la r / Gerçek sağlam lar(referans teste göre) False (+) lik: Testin hasta dediği tüm kişilerin gerçekten hasta olan kişilerden farkıdır. False (-) lik: Testin sağlam dediği tüm kişilerin gerçekten sağlam

perinededir. Çocuklarda; ekstremite ekstensör yüzlerinde Erişkinde fleksural bölgelerde (dirsek önü, diz arkası, el ayak bilekleri fleksör yüzü) M ajor kriterler

olan kişilerden farkıdır. Pozitif prediktif değer: Yeni tanı testinde pozitif (hasta) çıkan tüm kişilerin. hangi oranda gerçekten hasta oldu ğu nu gösterir.



Atopik öyküsü (aile veya kendisinde)



Pruritis-kaşıntı



Kronik tekrarlayan dermatit



Tipik morfolojik dağılım

Tanı testinin gerçek ya ka la d ığı pozitifler / tanı testine göre tüm pozitifler Negatif prediktif değer: Yeni tanı testinde negatif (sağlam) çıkan tüm kişilerin. hangi oranda gerçekten sağlam olduğunu gösterir. Tanı testinin gerçek ya ka la d ığı negatifler / tanı testine göre tüm negatifler

39.

A şağıd akilerin hangisinde N ik o lsk y (+)'tir? A)

Seboreik dermatit

B)

Pem figus vulgaris

C)

Büllöz pemfigoid

D)

Tinea versicolor

E)

Leischm ania cutis Cevap B

PEM F İG U S

40.

15 yaşın d a erkek hasta özellikle spor yaptığında ayağında

Pem figus otoim m ün kökenli, kronik, büllü bir hastalıktır. En

ciddi ağrı yakınm ası ile doktora başvuruyor. Ayak bileği

sık orta ya şla rd a başlar, cinsler eşit olarak tutulur. Pemfigusta,

grafisinde kalkaneal apofizin fragm ante ve skleroze olduğu

intersellüler ara maddeye karşı gelişm iş otoantikorlar vardır. IgG

ve manyetik rezonans görüntülem e ile kalkaneus metafizinde

yapısındaki bu otoantikorlar, antijen-antikor reaksiyonu sonunda

ve apofiziyel alanda kemik zedelenmesi ve mikrofraktürle

intersellüler ara maddeyi eritir, sekonder olarak dezmozom lar

uyum lu görünüm elde edildiği saptandı.

kopar (akantolizis) (TUS). Hücreler arasında oluşan boşluğa

Bu h asta için en olası tanı aşağıdakilerden hangisidir?

serözitenin dolm asıyla bül oluşur.

A)

Legg- Perthes hastalığı

B)

O sgood- Schlatter hastalığı

C)

Kienbock hastalığı

D)

Sever hastalığı

E)

Freiberg hastalığı

Klinik: Pem figusun dört klinik tipi vardır: 1.

Pem figus vulgaris: O lguların çoğunda lezyonlar ağız m ukozasından başlar. Ağızdan b aşlam asa bile, her

Cevap D

olguda ağız lezyonu m utlaka görülür. Oral lezyonlar ağrılıdır ve aylarca devam eder. Vulva ve konjonktiva gibi m ukozalar da tutulabilir. Büller genellikle sağlam deri üzerinde yerleşm işlerdir, İyileşen erozyonların yerlerinde hiperpigm entasyon kalır. Kaşıntı yoktur.

PER T H E S CALCE LEGG HASTALIĞI Proksim al fem oral epifizin idiopatik avasküler nekrozudur ve en sık görülen avasküler nekrozdur. En sık 5-7 ya şla rın d a görülür. •

Epidermisin yüzeyel kısm ına uygulanan basınç ile yeni

önem li sebebidir, bilateral avasküler nekroz yapar.

bül olu şu r ve bül kaydırm a ile epidermisin altında ilerler buna N ikolsky belirtisi adı verilir. Tzanck'ın sitolojik muayenesi ile pem figus vulgaris bülleri içindeki akantolitik epidermal hücreler görülebilir. Histopatolojisinde, suprabazal yerleşim li akantolitik bül

2.

Tuberositas T ibia'nın aseptik nekrozudur. Pubertal yaşlarda görülür. KO H LER H ST Tarsal Naviküler Kemiğin aseptik nekrozudur. K İE N B O C K HST: El Kemiği (Lunatum) aseptik nekrozudur.

dokuya karşı o lu şm uş IgG tipinde antikorlar, direkt

SE V E R HST: Kalkaneus posteriyorunun apofiziti.

im m ünfloresan incelemede epidermisde intersellüler

FREIBERG HASTALIĞI

alanlarda IgG birikimi saptanır.

2.

Metastars başının aseptik nekrozudur.

Pem figus vegetans: Daha çok inguinal ve aksillar 41. Önceki soruda bahsedilen hastanın tedavisinde aşağıd aki

Pem figus foliaseu s: Subkorneal yerleşimli, çok yüzeyel

tedavi yöntem lerinden hangisinin ku lla n ılm a sı önerilm ez?

büllerle ile karakterizedir. Histopatolojisinde granüler

A)

İboprufen başlanm ası

B)

Sp o r faliyetlerine ara vermeden devam etmesi

C)

İyontoforez ile lokal kremlerin uygulanm ası

D)

Tabanlık kullanılarak topuğun üzerine basılm asının

tabakanın hemen altında akantolizis görülür. Nikolski fenomeni (+)'tir (TUS) 4.

O SG OO D SCH LATTER

görülür. İndirekt im m ünfloresan inceleme intersellüler

bölgelere yerleşir. 3.

Steroid kullanımı; Erişkinde avasküler nekrozunun en

Pem figus eritematozus (Pem figus seboreik, Senear Usher sendromu): Göğüste ve yüzde görülür, pem phigus

engellenm esi

foliaseusun seboreik bölgelere lokalize formudur. E)

Aralıklı soğuk uygulam a

Tedavi: Cevap B Pem figus vulgaris, prognozu kötü olan bir hastalıktır; tedavi edilmediği takdirde bir y ıl içerisinde ölüm le sonlanır. Fakat sistem ik kortikosteroidler mortaliteyi azaltmıştır. Daha etkin bir tedavi ve kortikosteroidlerin ya n etkilerini azaltm ak için im m ünosupresif ajanlar, steroid tedavisine ilave edilebilir.

Fizik tedavi modalitelerini hatırlatm ak istedim. Genel olarak oral ve lokal ağrı kesiciler kullanılır. Akut travma ağrısında sıcak u ygulam a değil soğ u k uygulam a tercih edilir ve üzerine y ü k binmemesi için topuğun desteklenmesi sağlanır. Ağrı azalıncaya kadar spora devam etmesi önerilmez.

42.

A şağıdakilerden hangisi otosklerozun bu lgularından değildir? A)

işitme kaybı

B)

W illis parakuzisi

C)

Aile hikayesi

D)

Schvvartz belirtisi

E)

Kulak ağrısı Cevap E

OTO SKLEROZ Otik kapsülün (iç kulak ve labirenti saran sert kemik doku) primer hastalığıdır ve patolojik olarak spongiöz kemik oluşum u vardır. Genellikle bilateraldir. En sık oval pencerede yerleşir. Nadiren yu va rla k pencere ve kohleada da görülebilir. Etyoloji tam olarak bilinememektedir. % 5 0 vakada inkomplet otozom al dom inant geçiş vardır. Flor-çinko eksikliği, puberte, m enapoz ve ham ilelik sem ptom ları alevlendirir. Klinik oval pencerede yerleşenler başlangıçta iletim tipi işitme kaybına neden olurken, Kokleayı tutan otoskleroz sensorinoral işitme kaybına y o l açar.

Tanı •

Progresif işitme kaybı (iletim -m ikst-sensörinöral olabilen)



2000 Hz'de kem ikyo lu n d a düşm e olur(charcot çentiği)



Schvvartze işareti (zarda pembelik olması)



Aile hikayesi olması



Parakuzi: gürültülü ortam da daha iyi duym a olur.

Tedavi sem ptom atik tedavi uygulanır. Aktif dönemde flor verilerek inaktif hale getirilir, işitme kaybı 25dB'i geçtiğinde oval pencere otosklerozunda stapedektomi yapılarak protez konur.

T US'U KA ZAND IRAN DOĞRU A D R ES

TUSWORLD •

KLİNİK BİLİMLER

EKG: Özellikle inferior derivasyonlarda testere dişi görünümü görülür. ÖRS ler düzenlidir.

1.

Hipertansiyon yakınması olan bir hastada aşağıdaki fizik muayene bulgularından hangisi sekonder hipertansiyonu düşündürecek bulgulardan biri değildir? A. Sert S2 B. Göbek çevresinde üfürüm C. Alt ve üst ekstremite tansiyon ve nabız farklılığı D. Kan biyokimyasında hipokalemi ve metabolik alkaloz E. Lomber bölgede kitle varlığı CEVAP: A

Tedavi atrial fibrilasyona benzer.

Sert S2 hem esansiyel hem de sekonder hipertansiyonda olabilen bir bulgudur. Göbek çevresinde üfürüm renal arter stenozunu, alt ve üst eksremite tansiyon farklılığı aort koarktasyonunu, hipokalemi ve metabolik alkaloz hiperaldesteronizmi, lomber bölgede kitle varlığı polikistik böbrek hastalığının düşündürür.

3.

56 yaşındaki erkek hasta acil servise çarpıntı yakınması ile başvuruyor. Çekilen elektrografisi yukarıdaki gibi olan hastada en olası tanı aşağıdakilerden hangisidir? A. Atriyal fibrilasyon B. Atriyal flutter C. Atriyal taşikardl D. Atriyoventriküler nodal reentran taşikardl E. İdioventriküler ritim 2.

CEVAP: B Atrial flutter • Atrial makroreentran halka sonucu oluşur. AF ile benzer etyolojiye sahiptir. A tria l flutter

K azananların d l M G j A j

A. B. C. D. E.

Yirmibir yaşında bir hasta futbol oynarken aniden senkop geçiriyor. Daha önce hiçbir yakınması olmayan hastanın öyküsünden amcasının ani kardiyak ölüm ile kaybedildiği öğreniliyor. Fizik muayenesinde kan basıncı: 140/60 mmHg olarak bulunuyor. Kardiyak muayenede tüm prekordiyumda apekse yayılım gösteren 376° kaba sistolik üfürüm duyuluyor ve apekste palpabl S4 hissediliyor. EKG’sinde sol ventrikül hipertrofisini gösteren voltaj artışı, lateral prekordiyal derivasyonlarda ST-T değişiklikleri, derin ve dar Q dalgaları saptanıyor. Bu hastada en olası tanı aşağıdakilerden hangisidir? Hipertrofik kardiyomiyopati Brugada sendromu Restriktif kardiyomiyopati Aritmojenik sağ ventrikül displazisi Dilate kardiyomiyopati

CEVAP: A Hipertrofik Kardiyomiyopati:(HKM) HCM kardiak yada sistemik nedenler (aort stenozu, hipertansiyon vs) dışında hipertrofiye uğramış ve nondilate sol ventrikül ile karakterizedir. Eğer sol ventrikül çıkışında hipertrofiye bağlı obstruksiyon var ise idiopatik hipertrofik subaortik stenoz (İHSS) yada hipertrofik obstruktif kardiyomiyopati (HOCM) olarak da isimlendirilir.

tusworld.com.tr

TUSWÔRLD Normal H eart

HCM / Obstruction

EKG: • Sol atrial genişleme, inferolateral derivasyonlarda Q dalgaları, sol ventrikül hipertrofisi, ST-T değişiklikleri ( lateral derivasyonlarda derin negatif T dalgaları) Ekokardiyografi: • Septal hipertrofi (> 1 3 mm), küçük sol ventrikül kavitesi, sol atrial dilatasyon, mitral kapak sistolik öne hareketi

Note the increased left ventricular (LV) muscle w all thickness decreased LV lum en size, and enlarged left atrium (LAJ of the hearts with HCM.

En sık genetik geçişli kardiyovasküler hastalıktır.

Tedavi: • Tıbbi tedavi: Beta blokör, kalsiyum kanal blokörleri, dizopramid • Cerrahi: Septal miyektomi • Septal alkol ablasyonu Hipertrofik obstruktif KMP de vazodilatör ilaç kullanımı kontrendikedir.

En sık genetik mutasyon beta miyozin ağır zincirinde oluşur. Troponin T mutasyonları yüksek mortalité ile ilişkilidir. Patofizyoloji: Sol ventrikül çıkış yolu obstruksiyonu, diastolik disfonksiyon ve m ikrovaskülerdisfonksiyon Belirti ve bulgular: • Hastalar sıklıkla asemptomatiktirler. • Kalp yetmezliği semptomları: Dispne, yorgunluk, ortopne vs • Miyokardiyal iskemi:Anjian pektoris (ihtiyaç ve sunu dengesizliği ve mikrovasküler disfonksiyon nedeniyle)) • Senkop ve presenkop • Ani ölüm: Gençlerde ve sporcularda ani ölümün en sık nedenidir. Egzersiz sırasında veya hemen ardında ortaya çıkan anjina, dispne ve senkop + ailede ani ölüm öyküsü: Hipertrofik kardiyomiyopati düşün Gençlerde ve sporcularda ani ölümün en sık nedeni hipertrofik obstruktif kardiyomiyopatidir. Tüm toplumda ani ölümün en sık nedeni koroner ar­ ter hastalıklarıdır. Fizik muayne: • Parodoks çiftleşme ve S4 duyulabilir. • Sol sternal kenarda sistolik üfürüm .Üfürüm boyna ve aksillaya yayılmaz. Üfürüm preload ve afterload ile ters orantılıdır. • Karotis nabzı çift tepelidir, (pulsus bisferiensis)

tusworld.com.tr

Ani • • • • • •

ölüm riski yüksek olan hastalara IKD takılmalıdır. Kardiak arrest öyküsü Sürekli(sustained) ventrikülertaşikardi Ailede ani ölüm öyküsü, tekrarlayan senkop öyküsü Masif sol venriküler duvar kalınlığı >30 mm Egzersizde kan basıncında düşme Holter monitorizasyonunda tekrarlayıcı non-sustained VT

4.

İleri mitral darlığı tedavisinde aşağıdaki bulgulardan hangisi mitral balon valvuloplasti yerine cerrahi tedaviyi tercih nedenlerinden biri değildir? A. Orta-ileri mitral yetersizliği B. Ciddi sol ventrikül sistolik disfonksiyonu C. Sol apendikste trombüs D. Ciddi kalsifik-hareketsiz kapak E. Eşlik eden ciddi aort kapak hastalığı CEVAP: B Balon valvuloplastinin kontrendikasyonları: • Orta ileri derece MY varlığı • Sol atrial trombüs varlığı • Uygunsuz mitral kapak morfolojisi ( Yaygın kalsifik, hareksiz, kalınlaşmış vs) Valvuloplasti yapılamayan hastalara cerrahi düşünülür. Ciddi aort kapak hastalığında cerrahi endike olduğu için mitral kapak içinde cerrahi tercih edilir. Aynı operasyonda hem aort hem de mitral kapağa müdahele edilir.

Kazananların

dÜMjCiél

TUSVVORLD 5. 46 yaşındaki erkek hasta acil servise sol kola ve sırta yayılım gösteren baskı tarzında göğüs ağrısı ve nefes darlığı ile başvuruyor. Hastanın fizik muayenesinde kan basıncı 150/90 mmHg ölçülüyor. Dinlemekle S3, S4 ve mitral odakta 3/6 sistolik üfürüm tespit edliyor. Hastanın çekilen elektrokardiyografisinde prekordiyal derivasyonlarda ST depresyonları izlenyor. Troponin düzeyi 3.4 ng/ml olarak ölçülüyor. Bu hastada en olası tanı aşağıdakilerden hangisidir? A. Pulmoner emboll B. Unstabil anjina pektoris C. Non-ST miyokart infarktüsü D. Aort disseksiyonu E. Akut perikardit

6.

A. B. C. D. E.

CEVAP: C Hasta USAP kliniği ile başvurmuş. USAP+ troponin pozitifliği: Non-ST miyokart infarktüsü USAP ve Non-ST miyokart infarktüsü Göğüs ağrısının özellikleri: • Yeni başlangıçlı anjina ( son 2 ay içerisinde) • Var olan anjinanın şiddettinde veya süresinde artış olması • İstirahat anjinası , hafif efor ile tetiklenen anjina • Genellikle 20 dk dan uzun süren anjina (dinlen­ meye yada nitrata yanıt yoktur yada azalmıştır) • Miyokart infarktüsü sonrası anjina (postinfark anji­ na) EKG: ■ ST, T değişiklikleri olabilir. Normal elektrokardiyografigrafi izlenebilir. Ekokardiyografi: ■ Akut iskemi ile orantılı olarak duvar hareket bozukluğu tespit edilebilir.

Elli yaşındaki bir erkek hasta 2 yıldır progresif olarak artan egzersiz dispnesi nedeniyle başvuruyor. Çekilen ekokardiyografide pulmoner arter basıncı 55 mmHg olarak ölçülüyor. Önceden geçirilmiş bir akciğer hastalığı, pulmoner tromboemboli, derin ven trombozu tanımlamayan hastanın yapılan toraks bilgisayarlı tomografi anjiyografisi kronik trom boem bolik pulmoner hipertansiyon ile uyumlu olarak rapor ediliyor. Bu hasta için en uvaun tedavi aşağıdakilerden hangisidir? Antikoagülan tedaviye başlayıp 1 yıl sonra kontrole çağırmak Tedavi vermeden elektrokardiyografi kontrolüyle izlemek Vena cava inferior filtresi taktırıp, medikal tedavisiz izlem Pulmoner tromboendarterektomi yapmak Anjiyografik yolla trombolitik tedavi uygulamak

CEVAP: D Hastanın tanısı kronik tromboembolik pulmoner hipertansiyondur. Tedavisinde tromboendarterektomi uygulanır. 7.

Aşağıdakilerden hangisi kronik obstrüktif akciğer hastalığı gelişiminde rol oynayan risk faktörlerinden biri deöildir? A. Çocukluk çağında geçirilmiş ciddi solunum yolu enfeksiyonları B. İç ve dış ortam hava kirliliği C. Mesleki / çevresel toz ve kimyasal maruziyeti D. Sigara dumanı maruziyeti E. İç ortam allerjen maruziyeti CEVAP: E

Stres testleri (efor .sintigrafi): ■ Düşük riskli hastalarda (12-24 saattir göğüs ağrısı olmayan, troponin negatif, kalp yetmezliği semptomu olmayan) yapılabilir. Diğer hastalarda kontrendikedir. Koroner Anjiografi: ■ Tüm akut koroner sendrom hastaları koroner anjiografi için adaydırlar. Lezyon tespit edilen hastalar stent yada CABG ile tedavi edilebilirler.

K azananların

dİM jO uA

tusworld.com.tr

TUSWORLD 9.

KOAH risk faktörleri Kesinlik derecesi

Çevresel faktörler

Konjenital fak­ törler

Kesin



Sigara (En sık) Mesleksel maruziyet (Kadmiyum, silika) Organik yakıt (biomass)





Çoçukluk çağında pasif içicilik



Düşük doğum ağırlığı



Çocukluk çağında solunum yolu enfeksiyonları



Aile öyküsü



Havayolu aşırı du­ yarlılığı



■ Yüksek olasılık

Düşük olasılık



Düşük so­ syoekonomik durum



Alkol



Hava kirliliği



Adenovirüs enfeksiyonları Vitamin C eksikliği



8.

A. B. C. D. E.

Alfa-1 an­ titripsin eksikliği

Cinsivet farkı aözetmeksizin en sık aörülen akciğer kanseri tipi aşağıdakilerden hangisidir? A. Küçük hücreli akciğer karsinomu B. Adenokarsinom C. Skuamöz hücreli karsinom D. Büyük hücreli karsinom E. Mukoepidermoid karsinom CEVAP: B



A gurubu kan

Acil servise başvuran 70 yaşındaki bir hastada sol alt zonda pnömoni saptanıyor. Bilinci iyi olan hastaya yapılan fizik muayenede solunum sayısı 25/dakika, arteryal kan basıncı 85/50 mmHg olarak ölçülüyor. Laboratuvar incelemelerinde kan biyokimyası normal, lökosit sayısı 22 000/mm3 olarak bulunuyor. CURB-65 skorlamasına göre, bu hastanın skoru kaçtır? 1 2 3 4 5

CEVAP: B CURB-65 skorlaması ■ Konfüzyon ■ Üre > 42,8mg/dl (BUN>20mg/dl) ■ Solunum sayısı>=30dk ■ Sistolik kan basıncı 7 yaş

K azananların

dünyası

39

RLD 29. A ş ağ ıd a ki ko n v a n s iy o n e l o s teo sa rkom a tip le rin d e n h an g isi ra d y o lo jik g ö rü n ü m ü nedeniyle anevrizm al kem ik k is tle ri ile ka rışır? A. Osteoblastik B. Fibroblastik C. Kondroblastik D. Telenjitazik E. Parosteal CEVAP: D O STEO SARKO M A TİPLERİ A. Konvansiyonel osteosarkom (Yüksek grade) 1) Osteoblastik 2) Fibroblastik 3) Kondroblastik 4) Telenjitazik: Kemiğin diafizinden medullar boşluğa doğru ilerler.Litik görünümü nedeniyle radyografik görünümü ane vrizm al kem ik k is tle ri ile karışır. B. Diğer tipler (Düşük grade) 1) Parosteal: Genellikle posterior distal femura yüzeyel olarak yerleşir. Periferik (yüzeyel) yerleştiği için, te d a vid e sadece ce rrah i e k zisyo n ile tedavi edilebilir. 2) Periosteal: Parosteal osteosarkomalar gibi yüzeyel yerleşir ancak daha sık metastatik yayılım gösterir. DENEME

30. Dow n s e n d ro m lu la rd a g ö rü le n lö sem i ile ilg ili aşağıda ki ifadelerden h a n g is i d o ğ ru d u r? A. ALL/AML oranı genel populasyonla aynıdır GATA-1 proteini kötü prognos göstergesidir En sık görülen AML tipi M3’tür Geçici lösemi olanların % 50’si ileride tipik lösemi olur E. Lösemi riski normal popülasyona göre 10 kat artmıştır CEVAP: A 1- DOWN SENDROMU ve A K U T LÖSEMİ ve MİYELOPROLİFERASYON Down sendromlu hastaların lösemi geliştirme sıklığı normal popülasyona göre 14 kat artmıştır. Down s e n d ro m lu h astalarda A L L /A M L oranı genel po p u la s y o n la a y n ıd ır. Down ALL’lerin tedaviye yanıtı diğer ALL’ler gibidir, ancak Dow n A M L ’nin tedaviye yanıtı ve p ro g n o zu Down olmayan AML’ye göre çok daha iy id ir ve uzun süreli sağkalım oranı % 80’nin üzerindedir. Downlu çocuklarda sık lık la gözlenen AM L tip i M7 (m e g a k a ry o s itik lösem i)dir. 2- DOWN SENDROMU ve GEÇİCİ LÖSEMİ Yenidoğan ve bebeklerin ko n je n ita l lö s e m iy i andıran tra n s ie n t m iy e lo p ro life ra tif se n d ro m olarak bilinen durumları olabilir. Bu tür hastaların HSM, anemi, trombositopeni ve ağır lökomoid reaksiyonları ve periferde blastları vardır. Genellikle bu anormallikler birkaç gün ile hafta içinde kendiliğinden düzeldiğinden kemoterapi gerektirmezler.

40

K azananların

dünyâsı

G eçici lösem i ile iliş k ili p ro te in : GATA-1 Bu çocukların % 20-30’u sonraki birkaç yılda tipik lösemi geliştirebildiğinden yakın takibi gerekir. 31. A şa ğ ıd a kile rd e n han g isi ç o c u k lu k çağı lö se m ile r iç in ta n ım la n m ış ris k fa k tö rle ri arasında yer alm am aktadır? A. Nörofibromatozis-1 B. Kostman C. Turner D. Sotos E. Paroksismal noktürnal hemoglobinüri CEVAP: D

Çocukluk çağı lösem isinde risk faktörleri G e n e tik d u ru m la r

Çevresel faktörler

• Down sendromu (1/95)

• Iyonizan radyasyon

• Turner sendromu * Fankoni anemisi (1/12) • Bloom sendromu (1/8) ■ Nörofibromatosls tip 1 * Ataksl telenjiektazl * Diamond-Blackfan anemisi * A ğır kombine im mün yetmezlik

• Alkile edici ajanlar • Nitrosüre • Epipodofilotoksin • Benzen • ilaçlar

• Schawacm ann sendromu • Paroksis. noktürnal hemoglobinüri * Klinefelter sendromu * Li-Fraumenl sendromu • Kostman sendromu

NOT: İla çle r ve K ostm an se n d ro m u N elson 2020’ye yeni eklenen b ilg ile rd e n birid ir. K ostm an S endrom u: • Konjenital nötropeni • Ağır nötropeni, otit, pnömoni, tekrarlayıcı cilt enfeksiyonları ve sepsis ile seyreder • Kemik iliği incelendiğinde nötrofil gelişiminde promiyelosit/miyelosit evresinde duraksama olduğunun görülmesi tipiktir • Tedavide granülosit stimüle edici faktör (G-CSF) kullanılmaktadır 32. A şa ğ ıd a kile rd e n han g isi safra kanalı azlığı, iske le t d e fo rm ite le ri ve 20p12 m ikro d e le syo n u g ö rü le n s e n d ro m d u r? A. Potocki-Shaffer B. Jacobsen C. Langer-Giedon D. Axenfeld-Rieger E. Alagille CEVAP: E

tu sw orld.com .tr

RLD CEVAP: B

A LAG İLLE SENDROMU A lagille otozom al dom inant geçişli b ir sendrom dur. Fa llot

2 0 p l2 kromozomunda

T e troloiisi

mikrodelesyon bulunm aktadır. Hastalığın karakteristik bulguları, ince yüz anom alisi (üçgen yüz),

m

Si

sem er burun, kelebek verte bra . göz

X ’e 1. 2. 3. 4. 1. 2.

bağlı P rim er İm m ün Y etm ezlikler: X ’e bağlı agammaglobülinemi Hiper IgM sendromu Ağır kombine immün yetmezlik Wiskot-Aldrich sendromu Kronik granülomatöz hastalık X ’e bağlı lenfoproliferaif hastalık

anom alisi (pos te rio r

vertebra

embriyotokson

karaciğer hipoplazisi.safra kanalı azlığı/yokluğu büyüm e ve gelişm e geriliğidir.

33. Prematüre bir bebeğin transfontanel USG’sinde lateral ventrikülün %40’ını dolduran ancak ventrikülde dilatasyon yapmayan intraventriküler hemoraji saptanıyor. Bu bebeğin in tra v e n trik ü le r hem oraji evresi aşağıd a kilerde n h a n g is id ir? A. Evre 1 B. Evre 2 C. Evre 3 D. Evre 4 E. Evre 5 CEVAP: B İntraventriküler kanamada tanı USG ile konur. USG ile hem kanamanın varlığı hem de yaygınlığı gösterilir. Papile ve Volpe evrelendirmesine göre: • Evre I: Kanama subepandimal bölgeye sınırlı veya ventrikül %10’dan azı dolmuş • Evre II: İntraventriküler kanama, ventrikülün %10-50’si dolu, dilatasyon yok • Evre III: Ventriküler dilatasyonla birlikte %50’den fazla kanama • Evre IV: Evre III’ e ilaveten periventrikiler intraparenkimal kanama Lütfen IVK ve Subdural kanamanın farklarını irdeleyen aşağıdaki tabloyu da inceleyeniz. İVK

Subdural Kanama

Risk

Prematürite

Term LGA

Etiyoloji

Germinal matrix

Travma

Klinik

Letarji, konvülziyon. Emme ve Moro kaybı, Kabarık fontanel

Asemptomatik —►Benzer

Tanı

KraniyalUSG

KraniyalBT

Tedavi

Medikal, destek

Cerrahi

34. A şa ğ ıd a kile rd e n ha n g isi X ’e bağlı p rim e r im m ün y e tm e z lik le r arasında v e r alm az? A. Hiper IgM sendromu B. Hiper IgE sendromu C. Ağır kombine immun yetmezlik D. Kronik granülomatöz hastalık E. Wiskot-Aldrich sendromu

tu s w o rld .c o m .tr

35. A şağıdaki yö n te m le rd e n hangisi gastroözefa geal reflü ta nısında ku llanılm am akta dır? A. Özefageal pH monitörizasyonu B. Kontrastlı grafi C. Batın ultrasonografi D. Bronkoskopi E. Proton pompa inhibitörü CEVAP: C GÖR tanısında hafif olgularda klinik değerlendirme tanı için yeterli olabilir ve tedaviye yanıt (antireflü tedavisi) tanıyı doğrular. Ağır olgularda tanı özefageal pH probu (en kesin tanı aracı. Ancak artık olmazsa olmaz değil) çalışmaları ve baryumlu floroskopik özefagografi ile doğrulanabilir. Özefajit şüphesinde (gaitada gizli kan pozitif olur) özefagoskopi ile biyopsi tanısal bir tekniktir. Bunun dışında solunum yolu problemleri (vokal kord ödemi, kronik farenjit vs.) için tanıda bronkoskopiden de yararlanılmaktadır.

I

GÖR ta nısında ku lla n ıla n yö n te m le r: •Özefageal pH monitarizasyonu •Endoskopi (görüntüleme&biyopsi) •Kontrastlı grafi (baryum) •Çok kanallı intraluminal impedans •Bronkoskopi (Laringotrakebronkoskopi) •Ampirik antireflütedavisi (PPI) 36. A şa ğ ıd a kile rd e n ha n g isi özefageal atrezinin iliş k ili o ld u ğ u d u ru m la rd a n b iri d e ğ ild ir? A. Şişmanlık B. Sigara C. İleri anne yaşı D. Asya kökeni E. Düşük sosyoekonomik durum CEVAP: D Özefageal atrezinin gerçek nedeni bilinmemektedir ama ilişkili durumlar arasında ile ri anne yaşı, A vrupa kökeni, şişm anlık, d ü ş ü k s o s y o e k o n o m ik d u ru m ve s ig a ra içilmesi bulunmaktadır.

K a za n a n la rın

dünyası

41

DENEME

e m b riy o to k s o n ), kalp anom alisi,

K e lebe k

P o ste rio r

RLD 37. A ş ağ ıd a ki ifadelerde n h a n g is i d e m ir e k s ik liğ i a nem isi te d a v is i ile ilg ili ya n lış b ir ifa d e d ir? A. 2 mg/kg/günden Ferröz (++) dem ir kullanılır B. Hastanın günlük süt tüketimi en çok 500 ml olacak şekilde sınırlandırılmalıdır C. Aneminin nedeni malabsorbsiyon ise parenteral dem ir (dekstran) verilir D. Tedaviye yanıtı değerlendirmek için hasta 3-7. günler arası kontrole çağırılır E. Tedaviye yanıtı değerlendirmede retikülositoz yüzdesinden faydalanılabilir CEVAP: A

DENEME

D em ir e k s ik liğ i a n e m isind e te da vi ilke le ri: • Ferröz (++) tuzların (sülfat, glukonat, fumarat) oral verilmesi ucuz ve tatm inkâr tedavi sağlar. ■ 6 m g /kg/g ün dem ir preparatı verilir. • Aneminin nedeni malabsorbsiyon ise parenteral dem ir (dekstran) verilir. • Hastanın günlük süt tüketimi en çok 500 ml olacak şekilde sınırlandırılmalıdır • Demir profilaksisi: Miadında doğan normal doğum ağırlıklı bebeklere 4 aylıktan itibaren 1 mg/kg/gün (max 15 mg); düşük doğum ağırlıklı bebeklere 2 aylıktan itibaren 2 mg/kg/gün (max 15 mg/gün) Fe profilaktik olarak başlanmalıdır • Tedaviye yanıtı değerlendirmek için hasta 3-7. günler arası kontrole çağırılır • Tedaviye yanıtı değerlendirmede retikülositoz yüzdesinden faydalanılabilir

39. A şa ğ ıd a kile rd e n han g isi Fanconi se n d ro m u n u n se ko n d e r h e re d ite r nedenleri arasında ye r alm am aktadır? A. Wilson Hastalığı B. Sistinozis C. Homosistinüri D. Tirozinemi Tip-1 E. Dent Hastalığı CEVAP: C F a n co n i’de S e konde r H erediter N edenler 1. Sistinozis (en sık) 2. Lowe (oküloserebrorenal) sendromu 3. Galaktozemi 4. Tirozinemi 5. Herediter fruktoz intoleransı 6. Fanconi - Bickel sendromu 7. Wilson hastalığı 8. Mitokondriyal hastalıklar 9. Dent hastalığı (X’e bağlı nefrolitiazis) 40. A şağıdaki hastalıklarda n hangisin de H o m o g e n tisik a s it oksid a z e k s ik liğ in e bağlı olarak k lin ik b u lg u la r g ö rü lü r? A. Homosistinüri B. Fenilketonüri C. Alkaptonüri D. Hawkinsüri E. Metilmalonik asidüri CEVAP: C

38. A ş ağ ıd a ki ifadelerde n h a n g is i A lfa-Talasem i ta şıyıcılığ ı ile ilg ili ya n lış b ir ifa d e d ir? A. Demir tedavisinde dirençli hipokrom mikrositer anemi yapar B. Yenidoğan döneminde Bart hemoglobin seviyeleri % 3 -1 0 bulunur C. Sessiz taşıyıcılıkta tek bir a-globin gen delesyonundan kaynaklanır D. Özellikle Akdeniz ülkeleri, Güneydoğu Asya, Afrika, Ortadoğu, Güney Çin ve Hindistan’da görülür E. Tüm yaş gruplarında hemoglobin elektroforez ile tanı konur CEVAP: D

Phenylalanine hydroxylase Tyrosine Tyrosine aminotransferase

Y enidoğ an dö ne m i dışında Hb e le k tro fo re z i norm al o lu p DNA an alizi dışında tanı im kânı yoktur. HbH hastalığının ayrımı Hb elektroforezi ile yapılabilir.

K a z a n a n la rın

dünyası

k

4-Hydroxyphenylpyruvic acid 4-hydroxyphenylpyruvic acid dioxygenase Homogentisic acid Homogentisate 1,2-dioxygenase

Alfa talasemi taşıyıcılığı: Demir tedavisinde dirençli hipokrom mikrositer anemi yapar. Yenidoğan döneminde Bart hemoglobin seviyeleri % 3-10 bulunur.

B

Phenylalanine

)'

ALKAPTONURIA

Maleylacetoacetic acid

ALKAPTONÜRİ Homogentisik asit oksidaz eksikliğine bağlı, vücutta aşırı miktarda homogentisik asit birikimi ve idrarda fazla atılımı olur. Çocukluk çağında tek bulgu idrarın bekleyince siyah renk almasıdır. Ochronosis (dokuların renginin koyulaşması), artrit, kalp hastalığı (mitral ve aortik valvülit, kapak kalsifikasyonları ve miyokard enfarktüsü görülür.

tu s w o rld .c o m .tr

RLD 41. Aşağıdakilerden hangisi lens subluksasyonlarının olası nedenleri arasında ver almaz? A. B. C. D. E.

Sülfit oksidaz eksikliği Ehler-Danlos sendromu Sturge-W eber sendromu Klippel-Feil sendromu Okuloserebrorenal sendrom

CEVAP: E Lens Subluksasvonu Nedenleri: * * * * * *

Marfan sendromu (superotemporal) Homosistinüri (inferior) Sülfit oksidaz eksikliği Ehler-Danlos send. Sturge-W eber send. Klippel-Feil send.

42. Aşağıdaki organik asidemi nedenlerinden hangisi dermatit, alopesi ve “erkek kedi idrarı’ kokusu ile karakterizedir? A. İzovalerik asidemi B. Multiple karboksilaz eksikliği C. Akçaağaç şurup idrar hastalığı D. Glutarik asidüri tip-1 E. Glutarik asidüri tip-2

/T

CEVAP: B Hastalık

Enzim / Vitam in

Özellik

A k ç a a ğ a ç ş u ru b u idrarı hastalığı (M S U D )

a-ketoasid dehidrogenaz / B1 (tiyamin)

N e onatal te ta n o z ile k a rış a n hiperton i

izo va le rik asidem i

izovaleril CoA dehidrogenaz

Terli ayak kokusu

M ultip le k a rb o ksila z e ksikliğ i

Holokarboksilaz sentetaz (erken form < 1 ay) Biotinidaz (geç) / Biyotin

Erkek kedi idrarı D erm atit. Alopesi

p -ke to th io la z e ksikliğ i

-

R e y e -b e n z e ri k linik A s p irin intox, ile karışır

P ro p iyo n ik asidem i

Propionil C o A karboksilaz / Biyotin

H ip e rg lis in e m i

M e tilm a lo n ik asidem i|

Metilma. CoA mutaz B12: adenozilkobalamin

H ip e rg lis in e m i

G lu ta rik a sid ü ri tip 1 (lizin)

Glutaril CoA dehidrogenaz

M a k ro s e fa li, > 2 ya ş E n fe k s iy o n -> A ta k

G lu ta rik a sid ü ri tip 2 (E T F )

Elektron transport flavoprotein/ B2:riboflavin

M u ltip le A ç il K o A DH eksikliğ i, K e toz yok.

H id ro k s im e tilg lu ta rik asidem i

HMG KoA liyaz

K e toz yok.

I

Ç em en kokusu

tu sw o rld .co m .tr

N ö ro m o to r gerilik

Kazananların d ü /H (jQ A j

43

KLİNİK BİLİMLER 1. Aşağıdakilerden hangisi uzun Q T’ye bağlı torsades de pointes gelişimine neden olabilecek ilaçlardan biri değildir? A) Amiodoron B) Eritromisin C) Prokainamid D) Fenitoin E) Terfenadin

3. Sol kalp yetmezliği öyküsü olan bir hastada aşağıdakilerden hangisinin varlığı, hastada sağ kalp yetmezliğinin geliştiğini düşündüren bulgulardan biri değildir? A) Parasternal lift B) Hepatomegali C) Hepatojuguler reflü D) Kussmaul bulgusu E) Apikal vurunun laterale ve inferiora kayması CEVAP: E

CEVAP: D Apikal vurunun laterale ve inferiora kayması sol ventrikülün dilate olduğunun göstergesidir. Sağ yetmezlikle ilişkili değildir.

İlaçlar: ■ Sınıf 1a ve 3 antiartimik ilaçlar, ■ Makrolidler, trimetoprim sulfmetaksazol ■ Terfenadin ■ Trisiklik antidepresanlar ■ Fenotiazinler, haloperidol ■ Klorokin Fenitoin 1b gurubu bir antiaritmik ilaçtır. 2. 27 yaşında bayan hasta nefes darlığı, ortopne ve yorgunluk yakınmaları üzerine hastaneye başvuruyor. Hastanın öyküsünden romatizmal mitral kapak nedeniyle takipli olduğu fakat kontrollerine düzenli gelmediği öğreniliyor. Hastanın fizik muayenesinde kan basıncı 130/85 mmHg olarak ölçülüyor. Dinlemekle S1, S2 sert ve apekste middiastolik rulman ve pansistolik üfürüm duyuluyor. Çekilen elektrokardiyografide atriyal fibrilasyon izleniyor. Bu hasta için yapılan ekokardiyografide aşağıdaki bulgulardan hangisinin görülmesi en az olasıdır? A) Sol atriyal dilatasyon B) Mitral yetersizliği C) Pulmoner hipertansiyon D) Düşük ejeksiyon fraksiyonu E) Mitral darlığı CEVAP: D Mitral darlığında sol ventrikül sistolik ve diyastolik fonksiyonlarının etkilenmesi beklenmez. Romatizmal mitral kapak hastalığında erişkinde en sık mitral darlığı görülür, fakat hastanın apeksinde pansistolik üfürüm duyulması yetmezliğinde eşik ettiğini gösteriyor. S2 sertleşmesi pulmoner hipertansiyon veya sistemik hipertansiyonda olur.

tu sw o rld .co m .tr

Sol kalp yetm ezliğinde:



Akciğerde krepitan raller



Mitral yetersizlik üfürümü( sol ventrikül genişlemesine bağlı fonksiyonel)

Sağ kalp yetmezliğinde:



Hepatomegali, Juguler venöz dolgunluk



Bilateral gode bırakan ödem,



Hepatojuguler reflü,



S3 ve S4



Pulsus alternans



Parasternal lift



Apeks vurusunun aşağıya ve yana yer değiştirmesi



Protein kaybettiren enteropati

I

\

4. 40 paket yıl sigara kullanım öyküsü olan 55 yaşındaki erkek hasta bilateral bacak ağrısı yakınması ile başvuruyor. Hasta ağrısının yürümekle olduğunu ve kalçadan itibaren ayak bölgesine kadar hissettiğinin belirtiyor. Hastanın anemnezinden impotans yakınmasının da eşlik ettiği öğreniliyor. Fizik muayenede her tarafta femoral ve ayak nabızları alınamıyor. Bu hasta için en olası tanı aşağıdakilerden hangisidir? A) Leriche sendromu B) Abdominal aort anevrizması C) Tip B aort disseksiyonu D) Burger hastalığı E) Polimiyozit

dünyası

K azananların

29

DENEME

Etyoloji:

RLD CEVAP: A Leriche sendromu Aortun iliak arterlere ayrılmadan önce ateroskleroza bağlı tıkanması durumudur. Hastakarda impotans+ bilateral alt eksremite arteryel tıkanıklık bulguları izlenir. Leriche sendromu.

6. 12 saat önce aniden başlayan nefes darlığı nedeniyle acil servise getirilen 50 yaşındaki erkeke hastada takipne, taşikardi ve hipotansiyon saptanıyor. Çekilen kontrastlı toraks bilgisayarlı tomografisinde, sol ana pulmoner arteri tama yakın tıkayan pulmoner emboli ile uyumlu dolma defekti tespit ediliyor. ProBNP düzeyi: 2000 pg/ml bulunana hastanın ekokardiyografisinde sistolik pulmoner arter basıncı yüksek saptanıyor. Herhangi bir kanama diyatezi öyküsü bulunmayan bu hasta için en uygun tedavi yaklaşımı aşağıdakilerden hangisidir? A) Standart heparin infüzyonu B) Fondaparinuks C) Dabigatran D) Cerrahi tedavi E) Trombolitik tedavi CEVAP: E

Leriche sendromu. Alt eksremite kollateraller ile beslenmekte.

Pulmoner Emboli Tedavisi: Yüksek riskli pulmoner emboli: Hipotansiyon, şok Orta riskli pulmoner emboli: Sağ ventrikül işlev bozukluğu( EKO,BNP), miyokart hasarı (troponin) Düşük riskli pulmoner emboli: Sağ kalp yetmezliğin ve miyokart hasarının olmaması

DENEME

5. Pulmoner arter kateterizasyonu sırasında aşağıdakilerden hangisi doğrudan ölcülemeven bir parametredir? A) Santral venöz basınç B) Pulmoner arter basıncı C) Pulmoner kapiller kama basıncı D) Sistemik vasküler direnç E) Sağ ventrikül basıncı ■ CEVAP: D Pulmoner arter kataterizasyonu ile sırasıyla, santral venöz basınç, sağ atriyal basınç, sağ ventrikül basıncı, pulmoner arter baıncı ve pulmoner wedge kapiller basınç ölçülebilir. Sistemik vasküler direncin ölçülebilmesi için kalp debisinin ve ortalama sistemik kan basıncının bilinmesi gerekir. Kan basıncı: Kalp debisi * sistemik vasküler direnç



Tüm pulmoner emboli hastalarında en az 3 ay antikoagülasyona devam edilmelidir. Eğer neden düzeltilemiyorsa (örn: hemofili durumu) ya da emboli antikoagülasyon kesildikten sonra tekrarlamışsa antikoagülasyon ömür boyu sürmelidir. (Warfarin INR 2-3 aralığı) Antikoagüle edilemeyen yada antikoagülasyona rağmen emboli atakları olan hastalara vena kava filtresi takılır.

Pulmomer embolide trombolik tedavi 7-14 güne kadar verilebilir.

30

K azananların dünyası

tusw orld.com .tr

CEVAP: A Akut alevlenmede KOAH’ta mekanik ventilasyon Endikasyonları: ■ pH 45 mmHg ve ■ Solunum sayısı > 24 olmasıdır. o Öncelikli olarak non-invaziv mekanik ventilasyon (NIMV) uygulanmalıdır, o Kardivasküler instabilite (hipotansiyon, miyokard infarktüsü vs ) ve bilinç bulanıklığı NIMV için kontrendike durumlardır. Bu hastalara başlangıcta invaziv mekanik ventilasyon uygulanmalıdır. o Non-invaziv mekanik ventilasyona yanıt alınamayan hastalara invaziv mekanik ventilasyon uygulanmalıdır. 8. Plevral efüzyonu olan bir hastadan plevral ponksiyon ile alınan sıvının incelenmesinde glukoz düzeyi 25 mg/ dL, pH 7.15, LDH düzeyi 850 IU/L, ADA düzeyi 30 U/L olarak bulunmuştur. Bu hasta için en olası tanı aşağıdakilerden hangisidir? A) Tüberküloz plörezisi B) Malign plevral efüzyon C) Sistemik lupus eritematozusa bağlı plevral efüzyon D) Romatoid artrite bağlı plevral efüzyon E) Mikst konnektif doku hastalığına bağlı plevral efüzyon CEVAP: D Tüberküloz plörezisinde pH genelde < 7.3 olur, 7.4’ün üzeri tbc’yi ekarte ettirir. Adenozin deaminaz düzeyi yüksektir (>40 IU), glukoz düşük olur.

tu sw o rld .co m .tr

Kollajen doku hastalıklarında eksüda vasfında sıvı olur. Kollejen doku hastalığında ayırıcı tanı Plevra sıvısı

SLE

RA

Glukoz

> 80 mg/dl

< 30 mg/dl

LDL

< 500 IU/L

> 7 0 0 IU/L

pH

< 7.35

< 7.20

9. Hastane dışında gelişmiş ağır pnömoni nedeniyle yoğun bakıma yatırılan 70 yaşında bir erkek hastanın akciğer filminde bilateral ve yama tarzında, çok sayıda infiltrasyonlar gözleniyor. Hastadan bronkoskopiyle alınan solunum sekresyonunun mikroskobik incelemesinde bol miktarda parçalı lökosit, çok az sayıda bakteri hücresi görülüyor.Kan biyokimyasında hiponatremi ve hipofosfatemi tespit ediliyor. İdrarda spesifik bir mikroorganizma varlığı için bakılan antijen testi pozitif bulunuyor. Bu hastadaki klinik tabloya aşağıdaki mikroorganizmalardan hangisinin neden olduğu düşünülmelidir? A) Staphylococcus aureus B) Mycoplasma pneumoniae C) Moraxella catarrhalis D) Chlamydia pneumoniae E) Legionella pneumophila tip 1

S

CEVAP: E

I Legionella Pnömonisi: ■ Atipik pnömoni etkenidir. ■ Pnömoni eşlik etmeyen Lejyoner Hastalığına Pontiac ateşi adı verilir. ■ Sularda yaşayan hücre içi bir patojen olduğu için klimalardan ve duş başlıklarından bulaşabilir. ■ Bilinç durumunda değişme, baş ağrısı, relatif bradikardi, karın ağrısı, ishal, karaciğer enzimlerinde yükselme, hiponatrem i, hipofosfatemi, LDH ve kreatininde yükselme gibi ekstrapulmoner belirtiler ve laboratuar anormallikleri saptanabilir. ■ İdrarda antijen pozitifliği ile tanı konur. ■ Tedavide birinci tercih olarak azitromisin önerilir.

\

10. Prostatik kalp kapağı olan bir hastaya diş çekimi planlanıyor. Profilaksi için en uygun yaklaşım aşağıdakilerden hangisidir? A) Herhangi bir profilaksi uygulanmasına lüzum yoktur B) İşlemin bir saat öncesinden 2 gr amoksisilin C) İşlemden bir saat önce 1 gr amoksisilin + 1 gr gentamisin D) İşlemden bir saat önce 1 gr vankomisin E) İşlemden yarım saat önce 1 gr seftriakson

dünyası

K azananların

31

DENEME

7. Altmış yedi yaşında bir erkek hasta nefes darlığında artış ve şuur bulanıklığı şikâyetleriyle getiriliyor. Hastanın kronik obstrüktif akciğer hastalığı tanısıyla düzenli olarak ilaç kullandığı öğreniliyor. Fizik muayenede kan basıncı 80/60 mmHg, solunum sayısı 35, konfüze ve ajite olarak saptanıyor; iki taraflı ronküsler duyuluyor. Arter kan gazları analizinde pH 7.2, PaCO2 75 mmHg, PaO2 40 mmHg olarak bulunuyor. Hastaya endotrakeal entübasyon yapılarak mekanik ventilasyon desteği veriliyor. Bu hastada noninvaziv mekanik ventilasyon tercih edilmemesinin nedeni aşağıdakilerden hangisidir? A) Hastanın şokta olması B) PaCO2 nin 45 mmHg nin üzerinde olması C) Solunum sayısının 24 ün üzerinde olması D) pH nin 7.35 in altında olması E) Dispnede artma olması

TUSWORLD CEVAP: B

CEVAP: A

İnfeksitf endokarditte antibiyotik profilaksisi: 1- Protez kapakğı ya da kalp kapağı onarımında protez materyali kullanılmış olanlar 2- Daha önce İE geçirmiş olanlar 3- Doğumsal kalp hastalığı olanlar • Cerrahi uygulanmamış ya da rezidü defekti, palyatif şantı ya da kondüitleri bulunan siyanotik DKH • Konjenital kalp hastalıklarının protez materyal ile onarımında sonra ilk 6 ay • Kardiak cerrahi sonrası rezidü defektin sürmesi durumlarında kullanılır. Am oksisilin/ampisilin işlemden 30-60 dk önce tek doz 2 gr Penisilin alerjisi olanlarda klindamisin 600 mg şeklinde uygulanır.

Hayvancılık ve bel ağrısı varsa ilk aklımıza gelecek olan brusellozdur. Bruselloz insanlara direkt veya indirekt olarak özellikle enfekte domuz ve geviş getiren hayvanlardan bulaşan bir zoonozdur. Remittan karakteri nedeniyle ondülan ateş olarak bilinir. Semptomatik hastalığın en yaygın nedeni B. melitensis ‘tir. Koyun keçi ve develerden bulaşır. B. abortus genellikle sığır ve bufalolardan geçmektedir. B. suis genellikle domuzlardan bulaşmakla birlikte geyik ve kemirgenlerden de bulaşabilir. B. canis ise çoğunlukla köpeklerden bulaşmaktadır.

DENEME

11. Altmış beş yaşında erkek hasta bir yıldır süren bel ağrısı ile doktora başvuruyor. Anemnezinde mesleğinin çobanlık olduğu öğreniliyor. Hastanın gece terlemesi ve ateş öyküsüde mevcut. Çekilen lomber grafide L3 vertebrada destürüksiyon gözleniyor. Hastanın Rose­ Bengal testi pozitif sonuçlanıyor. Aşağıdaki öncüllerden hangisi ya da hangileri en uygun tedavidir? I. Doksisiklin II. Rifampisin III. Streptomisin IV. Gentamisin A) B) C) D) E)

I I,III I,IV II,III I,N,IV

CEVAP: B Hastanın ateşi, gece terlemesi, bel ağrısı, vertebra destrüksiyonu ve rose bengal testinin pozitif sonuçlanması bizi bruselloz tanısına yaklaştırıyor. Bruselloz tedavisinde çeşitli protokollerde tedavi uygulanmaktadır. Ancak altın standart tedavi 6 hafta doksisiklin + 3 hafta streptom isindir. Brusella endokarditinde ise en az üç antibiyotikle tedavi edilmelidir (aminoglikozid- tetrasiklin-rifampin). 12. Ateş, gece terlemesi, kilo kaybı, bel ağrısı ile başvuran hastanın fizik muayenesinde hepatosplenomegali saptanıyor. Hastanın mesleğinin hayvancılık olduğu öğreniliyor. Aşağıdakilerden hastalıklardan hangisi ilk önce düşünülmelidir? A) Bruselloz B) Sıtma C) Şarbon D) Tularemi E) Enfektif endokardit

32

K azananların dünyası

13. Aşağıdakilerden hangisi organa spesifik otoimmün bir hastalıktır? A) Behçet hastalığı B) Ankilozan spondilit C) Romatoid artrit D) Goodpasture sendromu E) Sistemik skleroz CEVAP: D Behçet hastalığı ve ankilozan spondilit otoinflamatuar süreçler ile seyreder. Romatoid artirit ve sistemik skleroz sistemik tutulum lar ile seyreder. Goodpasture akciğer ve böbreği tutan organa özgü hastalıktır. 14. Üç haftadır bilateral pretibial ödem öyküsü bulunan 60 yaşında bir kadın hastada hepatosplenomegali, diyare, göz etrafında ekimoz, makroglossi, kalpte büyüme ve efor dispnesi saptanıyor. Laboratuar incelemelerinde BUN ve kreatinin değerleri normal olarak bulunuyor. 24 saatlik idrarda 5 gram proteinüri saptanıyor. Kemik iliği aspirasyonunda plazma hücrelerinde artış olduğu görülüyor. Hasta için en olası tanı aşağıdakilerden hangisidir? A) Sistemik lupus eritamatozus B) Primer amiloidoz C) Diabetik nefropati D) İntertisyel nefrit E) Kresentrik glomerülonefrit CEVAP: B Yaygın ödem, rakun gözü görüntüsü, organomegali, restirktif kardiomiyopati ve nefrotik düzeyde proteinüri olan hastanın kemik iliğinde plazma hücrelerinde artış saptanmış tanı primer amiloidozdur.

tu sw orld.com .tr

CEVAP: E Hastanın kliniği ve pANCA pozitifliği bizi PAN yada Churg-Strauss sendromuna yönlendirir. Ancak bu iki hastalıktada böbrek tutulumu RPGN(Kresentik glomerülonefrit) şeklinde olur. 16. Diabetik ketoasidoz teşhisiyle tedavi edilmeye başlanan tip 2 diabetus mellitus olan 55 yaşında bir kadın hastada, tedaviye başlanılıyor. Tedavinin 4. saatinde yapılan laboratuar tetkiklerinde kan şekeri 100 mg/dl, sodyum düzeyi 135 mEq/L, potasyum düzeyi 4,5 mEq/dl, idrar ketonu negatif bulunuyor. Kan pH’ı 7.30 iken hastanın bilinci kapanıp koma geliştiği gözleniyor. Bu hastada en olası tanı aşağıdakilerden hangisidir? A) Ketoasidozun derinleşmesi B) Santral pontin demiyelizasyonu C) Beyin ödemi D) Laktik asidoz E) Non-ketotik hiperosmolar koma CEVAP: C Diabetik ketoasidozlu bir hastanın laboratuar değerleri düzelmişken bilinci bozulmaya başlamışsa ilk düşünmemiz gereken beyin ödemidir. Beyin ödemi gelişmemesi için hipergliseminin çok hızlı düzeltilmemesi gerekir. Bu yüzden ilk 24 saatte genellikle 200-250 mg/dl arasında tutulmalıdır. 17. Kırk yaşında kadın hasta hiperkalsemi etyolojisi araştırmak için boyun ultrasonografisi isteniyor. Paratiroid hiperplazi mevcut ve beraberinde tiroid nodülü gözleniyor. Kan tetkiklerinde dikkat çekici olarak CAE (Karsino embriyonik antijen) yüksek sonuçlanıyor. Aşağıdakilerden hangisi tanıda yardımcıdır? A) Anti-troglobulin B) Kromogranin-A C) Metanefrin D) Reverse triiodotironin(rT3) E) Kalsitonin

tu sw o rld .co m .tr

CEVAP: E Paratiroid hiperplazi ve tiroit nodülünün beraber gözlenmesi ve CAE yüksekliği m edüller tiroid kanserini düşündürür. Bu bulgular ile hastada MEN-2a araştırılmalıdır. Medüller tiroid kanser takibinde kalsitonin yardımcıdır. Hastadaki hiperkalsemi muhtemelen paratiroit hiperplazisine bağlı olarak gelişmiştir. 18. Aşağıdakilerden hangisi böbrekten glukozun reabsorbsiyonunu engelleyerek etki yapar? A) Akarboz B) Muraglitazar C) Dapagliflozin D) Pramlintid E) Metformin CEVAP: C SGLT-2 inhibitörlerinin etki mekanizmaları böbrekteki sodyum glukoz tranporter (SGLT) inhibe ederek böbrekten glukozun geri emilimini engellemektir. Örneğin: Dapagliflozin, remogliflozin, sergliflozin.Son dönemlerde SGLT 2 inhibitörlerinin diabetes mellitusun kardiak komplikasyonu(Koroner arter hastalığı, kalp yetmezliği) gelişmiş ise hastanın sağ kalım süresini arttırdığı gösterildiği için sorulma ihtimalleri yüksektir.

S I

19. Yaz tatili dönüşünde gövdesinde kahverengi lekeler gelişmesi nedeniyle hastaneye başvuran 26 yaşındaki kadın hastanın muayenesinde yuvarlak, kaşıntılı, ağrısız, hiperkeratotik lezyonlar saptanıyor. Bu tabloya yol açan en olası etken aşağıdakilerden hangisidir? A) Cryptococcus neoformans B) Malassezia furtur C) Trichophyton rubrum D) Trichosparon beigeleii E) Microsporum canis

\

CEVAP: B Pitriazis versicolor etkeni M .furfur’dur. Gövdenin üst kısmında sırt ve boyunda kahverengi yuvarlak - oval kaşıntılı lezyonlar vardır. Lipidden zengin parenterel beslenmelerde katater kaynaklı invazif hastalık tablolarına yol açabilir. Yuvarlak maya formları ve kısa hifler halinde ürer. (Spagetti ve köfte görünümü)

dünyası

Kaza.nanların

33

DENEME

15. Elli yaşında kadın hasta son 3 aydır devam eden halsizlik, 10 kg kilo kaybı, bacaklarda döküntü şikayeti ile başvuruyor. Fizik muayenede kan basıncı 170/110 mmHg ve alt ekstremitelerde palpabl purpura tespit ediliyor. Laboratuvar tetkiklerinde hemoglobin 10.4 g/ dl, BUN:20 mg/dl, Kreatinin: 2 mg/dl, idrarda 12-14 eritrosit, pANCA + saptanıyor. Hastanın böbrek biyopsisinde saptanması beklenen tanı aşağıdakilerden hangisidir? A) Membranöz glomerülonefrit B) Fokal segmental glomerülonefrit C) Fokal segmental glomerüloskleroz D) Granülomatöz vaskülit E) Kresentik glomerülonefrit

TUSWORLD 20. Antibiyotik ilişkili diyareler için aşağıda verilen şıklardan hangisi doğrudur? A) Büyük üçlüğü “imipenem, metronidazol, nafsilin’’ oluşturur B) Psödomembranöz enterokolitin tedavisinde metroni­ dazol kullanılamaz C) Psödomembranöz enterokolitin önemli bir kısmında C. difficile saptanmaz D) Antibiyotik ilişkili diyarelerin çoğunda C. difficile tok­ sinine rastlanamaz E) Psödomembranöz enterokolitin asıl etkeni S. aureus’tur

22. Geçirilmiş emboli nedeniyle warfarin kullanan hasta burun ve dişeti kanaması şikâyetleri ile doktora başvuruyor. Laboratuvar testlerinde INR:7.8 olarak bulunuyor. Bu hastada en uygun tedavi aşağıdakilerden hangisidir? A) İlacı kesmek yeterlidir. B) Taze donmuş plazma verilir. C) Faktör replasmanı yapılır. D) İlacı kesilir ve oral 1 mg K vitamini verilir. E) Taze donmuş plazma ve oral 1 mg K vitamini verilir. CEVAP: D

CEVAP: D

DENEME

Psödomembranöz enterokolit En sık Klindamisin, ampisilin, sefalosporin sonrası akut diyare gelişir. C.difficile aşırı çoğalır. Antibiyotik esnasında ya da 6 hafta sonra gelişebilir. Hafif bir diyare ile başlar.Tanıda gaitada C.difficile toksinine bakılır.Ama her antibiyotik ilişkili kolitte de C.difficile toksini saptanmaz. Clostridium difficile dışı antibiyotik ilişkili diyare, antibiyotiğin kolon mikroflorasını baskılamasına bağlı SCFA (kısa zincirli yağ asitleri)miktarının azalması C. difficile sorumlu olduğundan, birçok antibiyotik ilişkili diyarenin olası nedeni SCFA üretiminin göreceli azalmasıdır. 21. Aşağıdaki hasta gruplarının hangisinde gastrik karsinom görülme riski en düşüktür? A) Yüksek miktarda tütsülenmiş et-balık tüketenler B) Kan grubu A olanlar C) Duodenal ülser nedeniyle gastrik rezeksiyon yapılmış hastalar D) Pernisiyöz anemili hastalar E) Morbid obezite nedeniyle gastrik by-pass uygulanan hastalar CEVAP: E Mide kanseri: Düşük sosyoekonomik düzey, besinlerin uygunsuz saklanması ve diğer diyetle ilgili ve lokal gastrik faktörler hastalıkla ilişkili bulunmuştur. Diyetle ilgili faktörler yağ protein ve A - C vitaminleri eksikliği, salamura et ve balık ve nitratlardır. Atrofik gastrit, postgastrektomi durumları, aklorhidri, pernisyöz anemi, adenomatöz polipler ve M enetrier’s hastalığı gibi altta yatan durumlar da insidansı artırabilir. Dünya Sağlık Örgütü Helikobakter Pilori’yi bir kanserojen olarak sınıflar ve epidemiyolojk olarak gastrit adenokarsinom ile ilişkili olarak bildirir. Bununla birlikte, H. Pilori saptanan hastaların çok küçük bir kısmında gastrik adenokarsinom gelişmektedir.Geçirilmiş mide cerrahilerinden 10-15 sene sonra gastrik karsinomlar gelişebilir.

34

K azananların dünyası

Warfarinin antidotu K vitaminidir. Hafif kanama ^ Ekimoz, peteşi, traş olan yerde kanama. INR Beta h c g 2- İlaç >• Anam nez 3- Prolaktinoma — > Hipofiz m r 4- Hipotiroidi — > T3 T4 t s h 5- M akroprolaktinem i— > Diiüsyon 6 - SLE — > ANA

Testi

30. Hipertrofik obstrüktif kardiyomiyopatili hastada atriyal fibilasyon tedavisinde antiaritmik olarak kullanımı riskli olan ajan hangisidir? A) Beta bloker B) Verapamil C) Digoksin D) Disopromid E) Amiodaron CEVAP:C

0 ------20 --------- 100 u g r/lt------- 2 0 0 u g r/lt............ N orm al insan

İLA Ç

M İK R O A D E N O M

MAKROADENOM

28. Aşağıdakilerden hangisi diyabetik ketoasidoz için yanlıştır? A) Hiperfosfatemi gelişir B) Kan glukozunun yükselmesi serum osmolaritesinin yükselmesi ile sonuçlanır C) Anyon açığı yükselir D) Osmotik diürez gelişir E) Hepatik glukoz outputu artar

31. 58 yaşındaki erkek hasta acil servise yaklaşık bir saattir devam eden sol sola ve sırta yayılım gösteren baskı tarzında göğüs ağrısı yakınması ile başvuruyor. Çekilen elektrokardiyografisi aşağıdaki gibi izleniyor.

11Â

L

-i'"'

"N



-fv



r J. - '- ■ - iİVJ I

;

c M I

DENEME

CEVAP:A DKA’da hiperglisemiye bağlı osmotik diürez gerçekleşir, osmotik diürez ile fosfor kaybedilir.Hiperfosfatemi görülmez. DKA, yüksek anyon açıklı metabolik asidozdur. İnsülin eksikliği ve glukagon fazlalığından dolayı hepatik glukoz outputu artar.

1 ! ı. 1 .. " i '■ ' u 'i" - | 1 i-A ı 'M I- - y i r 1 |p i j,.- - 1

29. Akromegali hastalarda hiperfosfatemi görülmesine rağmen normokalsemi görülmesinin sebebi aşağıdakilerden hangisidir? A) Artmış aktif D vitamini senteziyle bağırsaktan Ca emiliminin artışı B) Artmış PTH senteziyle bağırsaktan Ca emiliminin artışı C) Artmış kalsitonin senteziyle bağırsaktan Ca emilimi­ nin artışı D) Artmış IGF- 1 senteziyle bağırsaktan Ca emiliminin artışı E) Artmış Nörotensin2 senteziyle bağırsaktan Ca emili­ minin artışı

Soruda belirtilen hastada miyokart infarktüsünden sorumlu, tıkanmış olması en olası koroner arter aşağıdakilerden hangisidir? A) Sol ana koroner arter B) Sağ koroner arter C) Sol ön inen arter D) Sirkumfleks arter E) Septal arter

CEVAP:A Akromegalili hastalarda özellikle iki mekanizma üzerinden fosfor emilimi artar ve hiperfosfatemi meydana gelir: 1. artmış tübüler reabsorbsiyon 2. Büyüme hormonunun renal 1-alfa hidroksilazı aktiflemesiyle aktif D vitamini sentezi artışı ve D vitamininin barsaklardan Ca ve P emilimini artırması. Kanda artan Ca poliüri ile atılarak dengelenir. Sonuç olarak kanda hiperfosfatemik normokalsemi olur.

36

HOKMP hastalarında gradienti azaltan beta bloker,disopramid ve bazen verapamil kullanılır. Ancak digoksin gibi gradenti arttıran ilaçlardan kaçınılır.Amiodaronun olumsuz etkisi yoktur.

dünyası

K azananların

• ...............

,

V

1 ■" ^ 4 .

|

TV.

-

-

:

i

ı

r 13

J U -v . ■ _

1 1 -i1 "■ V - J" 1■-■ ■n I ■-ı .„■ ■ .

ı

.-

CEVAP:D Derivasyonlarla ilişkili anatomik bağlantılar: inferior duvar

DII, DIII ve aVF

Septum

V1-V2

Anteroseptal duvar

V1-V4

Anterior duvar

V1-V6

Lateral duvar

I, aVL, V5, V6

Yüksek lateral

I, aVL

Anterolateral (yaygın an­ terior)

V1-V6, I, aVL

Posterior duvar

V7-V9 derivasyonları

Sağ ventrikül

Sağ taraftaki göğüs derivasyonları ( V1R-V6R En spesifik V4R)

tu sw orld.com .tr

J_ t -

32. Fluorourasil’e dönüşerek etki eden, metastaz yapmış kolerektal kanser tedavisinde oral kullanılan ilaç aşağıdakilerden hangisidir ? A) Sitarabin B) Fludarabin C) Kladribin D) Doksorubisin E) Kapesitabin

"K.Ok1BaonKT=W tf BS5 TDPENIH 3URFURA (T lP i „

HT R A ------ N

ttü'iLn.Ju

t. Jt-ifci

T

>l£jrtıll h

w :u l

r-EIIJJ. r b iı iL i- h

T îÜ H B ÎH lr jP C N l

H E I/D JT lK ÜREUİK SEHDRÜM {HÜSj

CEVAP:E Kapesitabin: 5- fluorourasile dönüşerek etki gösteren ön ilaçtır. Metastatik kolorektal kanserlerinin tedavisinde oral kullanılan bir ilaçtır.

.

/S

*

t

4

E F K F IIA

Ü Ö L £ H s r -1-3 ü J ı i g n i i k t r k î * , : :EM î J i :

rTP

A U A flflT t

Hû -

A

33. Herediter mide kanserinde, hasta asemptomatik ve genç ise aşağıdaki gen mutasyonlarından hangisinin varlığında, profilaktik gastrektomi önerilir? A) P53 gen mutasyonu B) NOD2 gen mutasyonu C) E-cadherin gen mutasyonu (CDH1) D) SPINK1 gen mutasyonu E) CFTR gen mutasyonu

35. Asbeste bağlı en sık gelişen kanser aşağıdakilerden hangisidir ? A) Mezotelyoma B) Küçük hücreli akciğer kanseri C) Akciğer adenokanser D) Büyük hücreli akciğer kanseri E) Squamöz hücreli akciğer kanseri

CEVAP: C

CEVAP: C

Profilaktik ameliyatlar: • RET nokta mutasyonu varlığında— bilateral total tiroidektomi • E-Cadherin mutasyonu---total gastrektomi • APC gen mutasyonu---total kolektomi • HİF onkogeni---sistektomi • MLH1MSH2— TAHBSO+kolektomi

Asbeste bağlı en sık gelişen kanser akciğer adenokanserdir. Mezotelyomanın ise en sık sebebi asbesttir.

34. 33 yaşında kadın hasta ateş, bacaklarında döküntü ve idrarda azalma olduğunu söyleyerek başvuruyor. Fizik muayenesinde ateş yüksekliği, ciltte ateşi, purpura ve bilinç değişikliği saptanan hastanın labaratuvarında: hemoglobin: 8,2 g/dL, trombosit 18.700/mm3 ve lökasit 8.500mm3 bulunuyor. Periferik yaymada fragmente eritrositler ve her alanda tekli trom bositler görülüyor. PT, aPTT, Fibrinojen düzeyi ve D-dimer tetkikleri normal saptanıyor. Coom b’s testi negatif bulunuyor. Bu hastanın en olası tanısı aşağıdakilerden hangisidir? A) Hemolitik üremik sendrom B) Yaygın dam ar içi pıhtılaşma C) Trombotik trom bositopenik purpura D) İmmün trombositopenik purpur E) HELLP sendromu

Selim hastalıklar

Malign hastalıklar

Plevral plaklar (en sık)

Akciğer kanseri (adenokanser, skuamoz),

/r \

i

Plevral efüzyon, Mezotelyoma Perikardiyal efüzyon, Diffüz plevral fibrozis, Diffüz interstisyel fibrozis 36. Primer biliyer sinozlu 48 yaşında kadın hasta kaşıntı şikâyetleriyle polikliniğe başvuruyor. Bu hastanın ilk başta en olası tedavisi aşağıdaki ilaçlardan hangisi olmalıdır? A) Kortikosteroid B) Azotiopürin C) Siklosporin D) Ursodeoksikolik asit E) Metotreksat CEVAP: D

CEVAP: C Trombotik trombositopenik purpurada; hemolitik anemi,trombositopeni,renal yetmezlik,ateş ve nörolojik bulgular gözlenir. ADAMTS-13 metalloproteazi Normal insanda % 30’un üzerinde fonksiyonel çalışır ancak bu MP’az T TP’de % 10’un altına düşmektedir.

tu sw o rld .co m .tr

Ursodeoksikolik asit: Primer safra asitlerinin toksik etkisini azaltır. Hepatositler üzerinde koruyucu etkisi vardır. Dolayısıyla genelde kolestatik hastalıkların tedavisinde kullanılır. Primer bilier sirozda hastalığın gidişini yavaşlatır, semptomları azaltabilir. Ancak küratif değildir.

dünyası

K azananların

37

RLD K a şın tı

K ilo kaybı

K ırk y a ş K adın

\

\

/

.77 K o le s te ro l!

K a n s e rle ş m e z

R in a "

( K J ll€ r

KCFTî

S h o z

AM A M2 +

K C T x (K ü ra tif te d a v i)

A L P îf K C Bx ' kırm ızı sa fra y o lu

K o le stira m in (P a lya tif)

Ağırlık artışı: İlk günlerde görülen ağırlık kaybı doğum kilosunun %510’u kadardır Doğum ağırlığına dönüş: 7-10. günlerde olur Doğum ağırlığının 2 katı: 4-5 ay Doğum ağırlığının 3 katı: 1 yaş Doğum ağırlığının 4 katı: 2 yaş Günlük ağırlık artışı; İlk 3-4 ayda 20-30 g/gün İlk yılın geri kalanında 15-20 g/gün Boy artışı: 1 yaşında doğum boyunun 1.5 katına 4 yaşında 2 katına 13 yaşında 3 katına çıkar İlk 6 ayda 2.5-4 cm/ay, ikinci 6 ayda 1 .5-3 cm/ay uzar.

KSANTOM

DENEME

37. Enteropatik artrit ile ilgili aşağıdakilerden hangisi yanlıştır? A) Periferik artrit şiddeti inflamatuvar barsak hastalığı aktivasyonu ile orantılıdır B) Ülseratif kolite kıyasla periferik artrit crohn hastalığında daha çok görülür C) Sakroileit şiddeti inflamatuvar barsak hastalığı aktivastonu ile orantılıdır D) İnflamatuvar barsak hastalığında görülen sakroileit genellikle simetriktir E) Aksiyel iskelet tutulumun ankilozan spondilitten ayırımı yapılamayabilir CEVAP: C İnflamatuvar barsak hastalığında görülen sakroileitin şiddeti inflamatuvar barsak hastalığı aktivasyonu ile bağlantılı değildir. 38. Yenidoğan bir bebek kaçıncı ayında doğum kilosunun 3 katına ulaşmaktadır? A) 10 B) 12 C) 15 D) 18 E) 24

Baş Çevresi artışı: 3. ayda 40.5 cm 6. ayda 43 cm 1 yaşında 46 cm dir Baş çevresindeki artış ilk 1 yıl 1 cm/ay (ilk 3 ay için 2 cm/ay, daha sonra yavaşlar) Yaşamın 1. yılından sonra artış toplamı 10 cm ’dir. 39. Bir bebekte ilk çıkması beklenen süt dişleri aşağıdakilerden hangisidir? A) Üst santral kesici diş B) Alt santral kesici diş C) Alt lateral kesici diş D) Üst lateral kesici diş E) Alt köpek dişi CEVAP: B Diş gelişimi ile ilgili bilinmesi gerekenler: • Genellikle ilk süt dişleri 5 - 10. ayda çıkar. İlk çıkan süt dişi alt orta kesicilerdir. • İlk dişin çıkması 14 -15. aya kadar uzayabilir. • 12 aylık çocukta 8 süt dişi vardır ve 2.5-3 yaşında 20 süt dişini tamamlamıştır. • Kalıcı dişler 5 - 7 yaşında ilk molarlar olacak şekilde çıkar. • Kabaca bebekte diş sayısı= Ay yaşı - 6 • Dişin çıkmasında en önemli iki husus genetik ve tiroid hormonudur.

CEVAP: B Yenidoğan bebekler kabaca yaşamlarının 12. ayında doğum kilosunun 3 katına ulaşırlar. Yenidoğanların vücut parametrelerindeki değişimler genel olarak şöyledir:

38

dünyası

K azananların

tu sw orld.com .tr

TUSWORLD 40. Dört yaşındaki erkek çocuk, ebevenylerine istediğini yaptıramadığında bağırmaya başladığı, elindekini fırlattığı, etrafa saldırdığı ve kendine zarar vermeye çalıştığı için polikliniğe getiriliyor. Bu çocuğa yaklaşım ile ilgili aşağıdakilerden hangisi yanlıştır? A) Çocuk öfke davranışları göstermeye başladığında dikkatini başka şeylere yönlendirmek B) Çocuğa daha az «hayır» demek doğrultusunda ön­ lemler almak C) Çocuğun kendine ve çevreye zarar vermeyeceği şekilde fiziksel ortam düzenlemesi yapmak D) Çocuğa yetenekleri dahilinde yapabileceği alternatif şeyleri göstermek E) Öfke nöbeti sırasında çocuğu yalnız bırakmak CEVAP: E Öfke patlamaları yaşamın ilk yıllarından itibaren görülebilmektedir ve hayal kırıklığı ya da öfkenin yaşa özgü ifadeleridir. Oyun çocuğunun karşı gelmelerine cezalandırıcı bir öfkeyle karşılık veren ebeveynler karşıt olma davranışını pekiştirebilirler. Ebeveynlere çocuğun karşı gelme davranışı ile başa çıkabilmesi için çocuğa seçenekler vermeleri önerilir; çocuğa yetenekleri dahilinde yapabileceği alternatifler gösterilebilir. Ebeveynlere çocuğu sakinleştiğinde öfkeye neden olan hususun giderilmeye çalışılacağını ancak aksi ve öfkeli davranışların kabul edilemeyeceğini söylemeleri tavsiye edilir. Ebeveynlere karşı gelme davranışlarında çocuğa 2-3 dakika sakin bir yere koyarak karşı gelme davranışının önüne geçmesi tavsiye edilmelidir. Öfke nöbeti sırasında çocuğu otoriter bir ses tonuyla uyarmak çocuğun aksi davranışını daha da şiddetlendirebilir. Ebeveynlere basit seçenekleri sakin bir şekilde sunmalarını tavsiye etmek, çocuğun daha kontrollü hissetmesine ve otonomi duygusunu geliştirmesine yardımcı olur. Ebeveynlere karşı gelme davranışlarında çocuğa 2-3 dakika sakin bir yere koyarak karşı gelme davranışının önüne geçmesi tavsiye edilmeli, çocuğu tamamen yalnız bırakmamalarını öğütlemelidir. 41. Aşağıdaki genetik hastalıklardan hangisinde, otizm spektrum bozukluklarında görülen davranış bozukluklarının görülmesi olası değildir? A) Frajil X sendromu B) Rett sendromu C) Tuberoskleroz D) Akondroplazi E) Angelman sendromu

tu sw o rld .co m .tr

CEVAP: D Otizm spektrum bozuklukları , sosyalleşme, iletişim ve davranış problemleri ile karakterize hastalıklardır. İletişimde bozulmalar, atipik sosyal etkileşim, dikkat eksikliği ve 3 yaşından önce başlangıç otizmin temel özellikleridir. Otizmin tanısı klinik muayene ile konulur. Aşağıdaki yer alan otizmin sık görüldüğü genetik hastalıklar tablosu iyi bilinmelidir. Otizmin sık görüldüğü genetik hastalıklar HASTALIK

SIKLIK

Frajil X sendromu

% 21-50

Rett sendromu

Çoğunlukla otizm eşlik eder

Tuberoskleroz

% 24-60

Down sendromu

% 5-39

Fenilketonüri

% 5-20%

CHARGE sendromu

% 15-50

Angelman sendromu

%50-81

Timothy sendromu

% 60-70

Joubert sendromu

~%40

42. Aşağıdakilerden hangisinin düzeyi olgun anne sütünde kolostruma oranla daha yüksektir? A) Yağ B) Protein C) Çinko D) İmmünglobulinler E) Yağda eriyen vitaminler CEVAP: A Kolostrum gebeliğin son 3 aylık döneminden itibaren meme alveolleri dolduran ve doğumdan birkaç gün sonraya kadar salgılanmaya devam eden önemli besleyici bir sıvıdır. Olgun anne sütüne göre laktoz, yağ, karbonhidrat ve suda eriyen vitaminler kolostrumda daha az iken; protein, yağda eriyen vitaminler (A, D, E, K), immünglobulinler ve bazı mineraller (özellikle Na, Fe, Zn, Cu) bakımından daha zengindir.

K azananların

dünyası

RLD 64. DENEMEKLINIK BİLİMLER AÇIKLAMALI CEVAPLARI 1. Aşağıdaki kardivovasküler hastalık nabız bulgusu eşleştirmelerinden hangisi yanlış olarak verilmiştir? Hastalık Nabız bulgusu A) Kalp yetmezliği Pulsus alternans B) Aort yetersizliği Pulsus parvus et tardus C) Hipertrofik KMP Pusus bisferiensis D) Atriyal fibrilasyon Pulsus defisit E) Kardiyak tamponad Pulsus paradoksus CEVAP:B Anormal Nabızlar Pulsus filiformis: Atım hacminin azalmasına bağlı zayıf amplitüdlü nabızdır. Şokta görülür. Pulsus alternans: Nabız amplitünün atımdan atıma düzenli değiştiği nabız tipidir. Pulsus alternans sol kalp

Pulsus bisferiensis: Sistolde çift tepeli nabız. Ateş ve egzersiz sonrası normal insanlarda palpe edilebilir. Ciddi ileri aort yetersizliğinde nonkomplian (genişlemeyen, sertleşmiş) vasküler yatak varlığında görülür. Hipertrofik kardiyomyopatide nadiren görülebilir.

Dikrotik nabız: Sistolik dalganın ardından abartılı dikrotik dalganın gelmesi ile oluşan iki vurumlu nabız. Dilate kardiyomyopayi, intraaortik balon pompası uygulaması ve sepsis te izlenebilir.

yetmezliğinde görülür

DENEME

Pulsus paradoksus: Normal insanlarda inspirasyon esnasında sistolik basınçta 10 mm Hg’ ye kadar bir azalma olabilir. Sistolik kan basıncının sistolde 10 mm Hg’den fazla düşmesine pulsus paradoksus denir. Gebelik ve obezitede fizyolojik olarak görülebilir. Görüldüğü durumlar: ■ Perikardiyal tamponad • Masif pulmoner emboli • Ciddi obstruktif akciğer hastalığı • Tansiyon pnömotoraks • Hemorajik şok Pulsus parvus et tardus: Nabız amplitünün küçük ve uzamış olduğu nabız tipidir. Yavaş bir yukarı çıkış kolu vardır. Aort stenozunda görülür.

Pulsus defisit: Kalp tepe atım sayısının, periferden ölçülen nabız sayısından fazla olması durumudur. Atrial fibrilasyonda görülür. Pulsus bigeminus: Normal nabızdan hemen sonra gelen küçük bir nabız vardır. Bigemine ventriküler erken atıma bağlı oluşur. 2. Yetmiş yaşında bir erkek hastanın öyküsünden 5 yıl önce akut miyokart enfarktüsü ve son bir yılda 2 kez akut akciğer ödemi atağı geçirdiği öğreniliyor. Bu hastada son üç haftada tekrarlanan ölçümlerde kan basıncı 190/115 mmHg civarında bulunuyor. Bir hafta önce ACE inhibitörü tedavisi başlanan hastanın kan basıncının 130/85 mmHg ye düştüğü gözleniyor. Laboratuvar incelemelerinde serum kreatinin düzeyinin 1.1mg/dl den 2.1 mg/dL yükseldiği bulunuyor. Bu hastada en uygun yaklaşım aşağıdakilerden hangisidir? A) ACE tedavisinin sürdürülmesi B) Tedaviye tiazid grubu diüretik eklenmesi C) Tedaviye kalsiyum kanal blokörü eklenmesi D) Kalp kateterizasyonu yapılması E) Renal anjiyografi yapılması

Sıçrayıcı (Corrigan) nabız: Hızlı çıkış ve iniş ile karakterize nabızdır. Water-hammer nabzı olarakta bilinir. Kronik aort yetersizliği , PDA ve hiperkinetik durumlarda görülür.

34

Kazananların

dünyası

tusworld.com.tr

CEVAP: C

Renal arter stenozu ■ Fibromüsküler displazi (gençlerde) veya aterosklerotik lezyona ( yaşlılarda) bağlı oluşur. • Hastaların klinik öyküsünde ciddi, ani başlangıçlı , renal fonksiyon bozukluğunun (kendiliğinden yada RAAS blokerlerine yanıt olarak) eşlik ettiği hipertansiyon izlenir. • Karında üfürüm duyulabilir. ■ Birinci basmak testi renal doppler USG dir. Kesin tanı anjiografi ile konulur. • Tedavide stent yada cerrahi uygulanabilir.

Atrial fibrilasyonda oral antikoagülanlar: Warfarin, dabigatran, rivaroksaban, apixaban • Warfarin kullanımında INR 2-3 arasında düzenlenmelidir. • Oral antikoagülan kullanamayacak hastalara aspirin+klopidogrel birlikte verilmelidir. • Dabigatran( oral direkt trombin inhibitörü), rivaroksaban ve apixaban (oral faktör Xa inhibitörleri) yeni oral antikoagülanlardır. INR takibi gerektirmezler. KBY hastalarında (GFR